You are on page 1of 194

Introduction

A 24-item NCLEX style exam all about the ADPIE or the Nursing Process.

Topics

Nursing Process

Guidelines

Read each question carefully and choose the best answer.

You are given one minute per question. Spend your time wisely!

Answers and rationales are given below. Be sure to read them.

If you need more clarifications, please direct them to the comments section.

Questions
1. Once a nurse assesses a clients condition and identifies appropriate nursing diagnoses,
a:
1. Plan is developed for nursing care.
2. Physical assessment begins
3. List of priorities is determined.
4. Review of the assessment is conducted with other team members.
2. Planning is a category of nursing behaviors in which:
1. The nurse determines the health care needed for the client.
2. The Physician determines the plan of care for the client.
3. Client-centered goals and expected outcomes are established.
4. The client determines the care needed.
3. Priorities are established to help the nurse anticipate and sequence nursing
interventions when a client has multiple problems or alterations. Priorities are determined
by the clients:
1. Physician
2. Non Emergent, non-life threatening needs
3. Future well-being.
4. Urgency of problems
4. A client centered goal is a specific and measurable behavior or response that reflects a
clients:
1. Desire for specific health care interventions
2. Highest possible level of wellness and independence in function.
3. Physicians goal for the specific client.
4. Response when compared to another client with a like problem.
5. For clients to participate in goal setting, they should be:
1. Alert and have some degree of independence.
2. Ambulatory and mobile.
3. Able to speak and write.
4. Able to read and write.
6. The nurse writes an expected outcome statement in measurable terms. An example is:
1. Client will have less pain.
2. Client will be pain free.

3. Client will report pain acuity less than 4 on a scale of 0-10.


4. Client will take pain medication every 4 hours around the clock.
7. As goals, outcomes, and interventions are developed, the nurse must:
1. Be in charge of all care and planning for the client.
2. Be aware of and committed to accepted standards of practice from nursing and other disciples.
3. Not change the plan of care for the client.
4. Be in control of all interventions for the client.
8. When establishing realistic goals, the nurse:
1. Bases the goals on the nurses personal knowledge.
2. Knows the resources of the health care facility, family, and the client.
3. Must have a client who is physically and emotionally stable.
4. Must have the clients cooperation.
9. To initiate an intervention the nurse must be competent in three areas, which include:
1. Knowledge, function, and specific skills
2. Experience, advanced education, and skills.
3. Skills, finances, and leadership.
4. Leadership, autonomy, and skills.
10. Collaborative interventions are therapies that require:
1. Physician and nurse interventions.
2. Nurse and client interventions.
3. Client and Physician intervention.
4. Multiple health care professionals.
11. Well formulated, client-centered goals should:
1. Meet immediate client needs.
2. Include preventative health care.
3. Include rehabilitation needs.
4. All of the above.
12. The following statement appears on the nursing care plan for an immunosuppressed
client: The client will remain free from infection throughout hospitalization. This statement
is an example of a (an):
1. Nursing diagnosis
2. Short-term goal
3. Long-term goal
4. Expected outcome
13. The following statements appear on a nursing care plan for a client after a
mastectomy: Incision site approximated; absence of drainage or prolonged erythema at
incision site; and client remains afebrile. These statements are examples of:
1. Nursing interventions
2. Short-term goals
3. Long-term goals
4. Expected outcomes.
14. The planning step of the nursing process includes which of the following activities?
1. Assessing and diagnosing
2. Evaluating goal achievement.
3. Performing nursing actions and documenting them.
4. Setting goals and selecting interventions.
15. The nursing care plan is:

1. A written guideline for implementation and evaluation.


2. A documentation of client care.
3. A projection of potential alterations in client behaviors
4. A tool to set goals and project outcomes.
16. After determining a nursing diagnosis of acute pain, the nurse develops the following
appropriate client-centered goal:
1. Encourage client to implement guided imagery when pain begins.
2. Determine effect of pain intensity on client function.
3. Administer analgesic 30 minutes before physical therapy treatment.
4. Pain intensity reported as a 3 or less during hospital stay.
17. When developing a nursing care plan for a client with a fractured right tibia, the nurse
includes in the plan of care independent nursing interventions, including:
1. Apply a cold pack to the tibia.
2. Elevate the leg 5 inches above the heart.
3. Perform range of motion to right leg every 4 hours.
4. Administer aspirin 325 mg every 4 hours as needed.
18. Which of the following nursing interventions are written correctly? (Select all that
apply.)
1. Apply continuous passive motion machine during day.
2. Perform neurovascular checks.
3. Elevate head of bed 30 degrees before meals.
4. Change dressing once a shift.
19. A clients wound is not healing and appears to be worsening with the current
treatment. The nurse first considers:
1. Notifying the physician.
2. Calling the wound care nurse
3. Changing the wound care treatment.
4. Consulting with another nurse.
20. When calling the nurse consultant about a difficult client-centered problem, the
primary nurse is sure to report the following:
1. Length of time the current treatment has been in place.
2. The spouses reaction to the clients dressing change.
3. Clients concern about the current treatment.
4. Physicians reluctance to change the current treatment plan.
21. The primary nurse asked a clinical nurse specialist (CNS) to consult on a difficult
nursing problem. The primary nurse is obligated to:
1. Implement the specialists recommendations.
2. Report the recommendations to the primary physician.
3. Clarify the suggestions with the client and family members.
4. Discuss and review advised strategies with CNS.
22. After assessing the client, the nurse formulates the following diagnoses. Place them in
order of priority, with the most important (classified as high) listed first.
1. Constipation
2. Anticipated grieving
3. Ineffective airway clearance
4. Ineffective tissue perfusion.

23. The nurse is reviewing the critical paths of the clients on the nursing unit. In
performing a variance analysis, which of the following would indicate the need for further
action and analysis?
1. A clients family attending a diabetic teaching session.
2. Canceling physical therapy sessions on the weekend.
3. Normal VS and absence of wound infection in a post-op client.
4. A client demonstrating accurate medication administration following teaching.
24. The RN has received her client assignment for the day-shift. After making the initial
rounds and assessing the clients, which client would the RN need to develop a care plan
first?
1. A client who is ambulatory.
2. A client, who has a fever, is diaphoretic and restless.
3. A client scheduled for OT at 1300.
4. A client who just had an appendectomy and has just received pain medication.

Answers and Rationale


1.

2.

3.

4.

5.

6.

7.

8.

9.

10. D
11. D
12. B
13. D
14. D
15. A
16. D. This is measurable and objective.
17. B. This does not require a physicians order. (A & D require an order; C is not appropriate for a
fractured tibia)
18. C. It is specific in what to do and when.
19. B. Calling in the wound care nurse as a consultant is appropriate because he or she is a specialist
in the area of wound management. Professional and competent nurses recognize limitations and seek
appropriate consultation. (a. This might be appropriate after deciding on a plan of action with the
wound care nurse specialist. The nurse may need to obtain orders for special wound care products.
c. Unless the nurse is knowledgeable in wound management, this could delay wound healing. Also, the
current wound management plan could have been ordered by the physician. d. Another nurse most
likely will not be knowledgeable about wounds, and the primary nurse would know the history of the
wound management plan.)
20. A. This gives the consulting nurse facts that will influence a new plan.
(b, c, and d. These are all subjective and emotional issues/conclusions about the current treatment
plan and may cause a bias in the decision of a new treatment plan by the nurse consultant.)

21. D. Because the primary nurse requested the consultation, it is important that they communicate
and discuss recommendations. The primary nurse can then accept or reject the CNS
recommendations. (a. Some of the recommendations may not be appropriate for this client. The
primary nurse would know this information. A consultation requires review of the recommendations,
but not immediate implementation. b. This would be appropriate after first talking with the CNS about
recommended changes in the plan of care and the rationale. Then the primary nurse should call the
physician. c. The client and family do not have the knowledge to determine whether new strategies are
appropriate or not. Better to wait until the new plan of care is agreed upon by the primary nurse and
physician before talking with the client and/or family.)
22. C, D, A, B.
23. B.
24. B. This clients needs are a priority.

Introduction
An NCLEX practice exam dedicated to Coronary Artery Disease and Hypertension. This exam contains
50 items about the two diseases.

Topics

Coronary Artery Disease

Hypertension

Guidelines

Read each question carefully and choose the best answer.

You are given one minute per question. Spend your time wisely!

Answers and rationales are given below. Be sure to read them.

If you need more clarifications, please direct them to the comments section.

Questions
1. A client is scheduled for a cardiac catheterization using a radiopaque dye. Which of the
following assessments is most critical before the procedure?
1. Intake and output
2. Baseline peripheral pulse rates
3. Height and weight
4. Allergy to iodine or shellfish
2. A client with no history of cardiovascular disease comes into the ambulatory clinic with
flu-like symptoms. The client suddenly complains of chest pain. Which of the following
questions would best help a nurse to discriminate pain caused by a non-cardiac problem?
1. Have you ever had this pain before?
2. Can you describe the pain to me?
3. Does the pain get worse when you breathe in?
4. Can you rate the pain on a scale of 1-10, with 10 being the worst?

3. A client with myocardial infarction has been transferred from a coronary care unit to a
general medical unit with cardiac monitoring via telemetry. A nurse plans to allow for
which of the following client activities?
1. Strict bed rest for 24 hours after transfer
2. Bathroom privileges and self-care activities
3. Unsupervised hallway ambulation with distances under 200 feet
4. Ad lib activities because the client is monitored.
4. A nurse notes 2+ bilateral edema in the lower extremities of a client with myocardial
infarction who was admitted 2 days ago. The nurse would plan to do which of the following
next?
1. Review the intake and output records for the last 2 days
2. Change the time of diuretic administration from morning to evening
3. Request a sodium restriction of 1 g/day from the physician.
4. Order daily weights starting the following morning.
5. A client is wearing a continuous cardiac monitor, which begins to sound its alarm. A
nurse sees no electrocardiogram complexes on the screen. The first action of the nurse is
to:
1. Check the client status and lead placement
2. Press the recorder button on the electrocardiogram console.
3. Call the physician
4. Call a code blue
6. A nurse is assessing the blood pressure of a client diagnosed with primary hypertension.
The nurse ensures accurate measurement by avoiding which of the following?
1. Seating the client with arm bared, supported, and at heart level.
2. Measuring the blood pressure after the client has been seated quietly for 5 minutes.
3. Using a cuff with a rubber bladder that encircles at least 80% of the limb.
4. Taking a blood pressure within 15 minutes after nicotine or caffeine ingestion.
7. IV heparin therapy is ordered for a client. While implementing this order, a nurse
ensures that which of the following medications is available on the nursing unit?
1. Vitamin K
2. Aminocaproic acid
3. Potassium chloride
4. Protamine sulfate
8. A client is at risk for pulmonary embolism and is on anticoagulant therapy with warfarin
(Coumadin). The clients prothrombin time is 20 seconds, with a control of 11 seconds. The
nurse assesses that this result is:
1. The same as the clients own baseline level
2. Lower than the needed therapeutic level
3. Within the therapeutic range
4. Higher than the therapeutic range
9. A client who has been receiving heparin therapy also is started on warfarin. The client
asks a nurse why both medications are being administered. In formulating a response, the
nurse incorporates the understanding that warfarin:
1. Stimulates the breakdown of specific clotting factors by the liver, and it takes 2-3 days for this to
exert an anticoagulant effect.
2. Inhibits synthesis of specific clotting factors in the liver, and it takes 3-4 days for this medication to
exert an anticoagulant effect.

3. Stimulates production of the bodys own thrombolytic substances, but it takes 2-4 days for this to
begin.
4. Has the same mechanism of action as Heparin, and the crossover time is needed for the serum level
of warfarin to be therapeutic.
10. A 60-year-old male client comes into the emergency department with complaints of
crushing chest pain that radiates to his shoulder and left arm. The admitting diagnosis is
acute myocardial infarction. Immediate admission orders include oxygen by NC at
4L/minute, blood work, chest x-ray, an ECG, and 2 mg of morphine given intravenously. The
nurse should first:
1. Administer the morphine
2. Obtain a 12-lead ECG
3. Obtain the lab work
4. Order the chest x-ray
11. When administered a thrombolytic drug to the client experiencing an MI, the nurse
explains to him that the purpose of this drug is to:
1. Help keep him well hydrated
2. Dissolve clots he may have
3. Prevent kidney failure
4. Treat potential cardiac arrhythmias.
12. When interpreting an ECG, the nurse would keep in mind which of the following about
the P wave? Select all that apply.
1. Reflects electrical impulse beginning at the SA node
2. Indicated electrical impulse beginning at the AV node
3. Reflects atrial muscle depolarization
4. Identifies ventricular muscle depolarization
5. Has duration of normally 0.11 seconds or less.
13. A client has driven himself to the ER. He is 50 years old, has a history of hypertension,
and informs the nurse that his father died of a heart attack at 60 years of age. The client is
presently complaining of indigestion. The nurse connects him to an ECG monitor and
begins administering oxygen at 2 L/minute per NC. The nurses next action would be to:
1. Call for the doctor
2. Start an intravenous line
3. Obtain a portable chest radiograph
4. Draw blood for laboratory studies
14. The nurse receives emergency laboratory results for a client with chest pain and
immediately informs the physician. An increased myoglobin level suggests which of the
following?
1. Cancer
2. Hypertension
3. Liver disease
4. Myocardial infarction
15. When teaching a client about propranolol hydrochloride, the nurse should base the
information on the knowledge that propranolol hydrochloride:
1. Blocks beta-adrenergic stimulation and thus causes decreased heart rate, myocardial contractility,
and conduction.
2. Increases norepinephrine secretion and thus decreases blood pressure and heart rate.
3. Is a potent arterial and venous vasodilator that reduces peripheral vascular resistance and lowers

blood pressure.
4. Is an angiotensin-converting enzyme inhibitor that reduces blood pressure by blocking the
conversion of angiotensin I to angiotensin II.
16. The most important long-term goal for a client with hypertension would be to:
1. Learn how to avoid stress
2. Explore a job change or early retirement
3. Make a commitment to long-term therapy
4. Control high blood pressure
17. Hypertension is known as the silent killer. This phrase is associated with the fact that
hypertension often goes undetected until symptoms of other system failures occur. This
may occur in the form of:
1. Cerebrovascular accident
2. Liver disease
3. Myocardial infarction
4. Pulmonary disease
18. During the previous few months, a 56-year-old woman felt brief twinges of chest pain
while working in her garden and has had frequent episodes of indigestion. She comes to
the hospital after experiencing severe anterior chest pain while raking leaves. Her
evaluation confirms a diagnosis of stable angina pectoris. After stabilization and
treatment, the client is discharged from the hospital. At her follow-up appointment, she is
discouraged because she is experiencing pain with increasing frequency. She states that
she is visiting an invalid friend twice a week and now cannot walk up the second flight of
steps to the friends apartment without pain. Which of the following measures that the
nurse could suggest would most likely help the client deal with this problem?
1. Visit her friend earlier in the day.
2. Rest for at least an hour before climbing the stairs.
3. Take a nitroglycerin tablet before climbing the stairs.
4. Lie down once she reaches the friends apartment.
19. Which of the following symptoms should the nurse teach the client with unstable
angina to report immediately to her physician?
1. A change in the pattern of her pain
2. Pain during sex
3. Pain during an argument with her husband
4. Pain during or after an activity such as lawn mowing
20. The physician refers the client with unstable angina for a cardiac catheterization. The
nurse explains to the client that this procedure is being used in this specific case to:
1. Open and dilate the blocked coronary arteries
2. Assess the extent of arterial blockage
3. Bypass obstructed vessels
4. Assess the functional adequacy of the valves and heart muscle.
21. As an initial step in treating a client with angina, the physician prescribes nitroglycerin
tablets, 0.3mg given sublingually. This drugs principal effects are produced by:
1. Antispasmodic effect on the pericardium
2. Causing an increased myocardial oxygen demand
3. Vasodilation of peripheral vasculature
4. Improved conductivity in the myocardium

22. The nurse teaches the client with angina about the common expected side effects of
nitroglycerin, including:
1. Headache
2. High blood pressure
3. Shortness of breath
4. Stomach cramps
23. Sublingual nitroglycerin tablets begin to work within 1 to 2 minutes. How should the
nurse instruct the client to use the drug when chest pain occurs?
1. Take one tablet every 2 to 5 minutes until the pain stops.
2. Take one tablet and rest for 10 minutes. Call the physician if pain persists after 10 minutes.
3. Take one tablet, then an additional tablet every 5 minutes for a total of 3 tablets. Call the physician
if pain persists after three tablets.
4. Take one tablet. If pain persists after 5 minutes, take two tablets. If pain still persists 5 minutes later,
call the physician.
24. Which of the following arteries primarily feeds the anterior wall of the heart?
1. Circumflex artery
2. Internal mammary artery
3. Left anterior descending artery
4. Right coronary artery
25. When do coronary arteries primarily receive blood flow?
1. During inspiration
2. During diastolic
3. During expiration
4. During systole
26. Prolonged occlusion of the right coronary artery produces an infarction in which of the
following areas of the heart?
1. Anterior
2. Apical
3. Inferior
4. Lateral
27. A murmur is heard at the second left intercostal space along the left sternal border.
Which valve is this?
1. Aortic
2. Mitral
3. Pulmonic
4. Tricuspid
28. Which of the following blood tests is most indicative of cardiac damage?
1. Lactate dehydrogenase
2. Complete blood count (CBC)
3. Troponin I
4. Creatine kinase (CK)
29. Which of the following diagnostic tools is most commonly used to determine the
location of myocardial damage?
1. Cardiac catheterization
2. Cardiac enzymes
3. Echocardiogram
4. Electrocardiogram (ECG)

30. Which of the following types of pain is most characteristic of angina?


1. Knifelike
2. Sharp
3. Shooting
4. Tightness
31. Which of the following parameters is the major determinant of diastolic blood
pressure?
1. Baroreceptors
2. Cardiac output
3. Renal function
4. Vascular resistance
32. Which of the following factors can cause blood pressure to drop to normal levels?
1. Kidneys excretion of sodium only
2. Kidneys retention of sodium and water
3. Kidneys excretion of sodium and water
4. Kidneys retention of sodium and excretion of water
33. Baroreceptors in the carotid artery walls and aorta respond to which of the following
conditions?
1. Changes in blood pressure
2. Changes in arterial oxygen tension
3. Changes in arterial carbon dioxide tension
4. Changes in heart rate
34. Which of the following terms describes the force against which the ventricle must expel
blood?
1. Afterload
2. Cardiac output
3. Overload
4. Preload
35. Which of the following terms is used to describe the amount of stretch on the
myocardium at the end of diastole?
1. Afterload
2. Cardiac index
3. Cardiac output
4. Preload
36. A 57-year-old client with a history of asthma is prescribed propranolol (Inderal) to
control hypertension. Before administered propranolol, which of the following actions
should the nurse take first?
1. Monitor the apical pulse rate
2. Instruct the client to take medication with food
3. Question the physician about the order
4. Caution the client to rise slowly when standing.
37. One hour after administering IV furosemide (Lasix) to a client with heart failure, a short
burst of ventricular tachycardia appears on the cardiac monitor. Which of the following
electrolyte imbalances should the nurse suspect?
1. Hypocalcemia
2. Hypermagnesemia

10

3. Hypokalemia
4. Hypernatremia
38. A client is receiving spironolactone to treat hypertension. Which of the following
instructions should the nurse provide?
1. Eat foods high in potassium.
2. Take daily potassium supplements.
3. Discontinue sodium restrictions.
4. Avoid salt substitutes.
39. When assessing an ECG, the nurse knows that the P-R interval represents the time it
takes for the:
1. Impulse to begin atrial contraction
2. Impulse to transverse the atria to the AV node
3. SA node to discharge the impulse to begin atrial depolarization
4. Impulse to travel to the ventricles
40. Following a treadmill test and cardiac catheterization, the client is found to have
coronary artery disease, which is inoperative. He is referred to the cardiac rehabilitation
unit. During his first visit to the unit he says that he doesnt understand why he needs to
be there because there is nothing that can be done to make him better. The best nursing
response is:
1. Cardiac rehabilitation is not a cure but can help restore you to many of your former activities.
2. Here we teach you to gradually change your lifestyle to accommodate your heart disease.
3. You are probably right but we can gradually increase your activities so that you can live a more
active life.
4. Do you feel that you will have to make some changes in your life now?
41. To evaluate a clients condition following cardiac catheterization, the nurse will palpate
the pulse:
1. In all extremities
2. At the insertion site
3. Distal to the catheter insertion
4. Above the catheter insertion
42. A clients physician orders nuclear cardiography and makes an appointment for a
thallium scan. The purpose of injecting radioisotope into the bloodstream is to detect:
1. Normal vs. abnormal tissue
2. Damage in areas of the heart
3. Ventricular function
4. Myocardial scarring and perfusion
43. A client enters the ER complaining of severe chest pain. A myocardial infarction is
suspected. A 12 lead ECG appears normal, but the doctor admits the client for further
testing until cardiac enzyme studies are returned. All of the following will be included in
the nursing care plan. Which activity has the highest priority?
1. Monitoring vital signs
2. Completing a physical assessment
3. Maintaining cardiac monitoring
4. Maintaining at least one IV access site
44. A client is experiencing tachycardia. The nurses understanding of the physiological
basis for this symptom is explained by which of the following statements?

11

1. The demand for oxygen is decreased because of pleural involvement


2. The inflammatory process causes the body to demand more oxygen to meet its needs.
3. The heart has to pump faster to meet the demand for oxygen when there is lowered arterial oxygen
tension.
4. Respirations are labored.
45. A client enters the ER complaining of chest pressure and severe epigastric distress. His
VS are 158/90, 94, 24, and 99*F. The doctor orders cardiac enzymes. If the client were
diagnosed with an MI, the nurse would expect which cardiac enzyme to rise within the next
3 to 8 hours?
1. Creatine kinase (CK or CPK)
2. Lactic dehydrogenase (LDH)
3. LDH-1
4. LDH-2
46. A 45-year-old male client with leg ulcers and arterial insufficiency is admitted to the
hospital. The nurse understands that leg ulcers of this nature are usually caused by:
1. Decreased arterial blood flow secondary to vasoconstriction
2. Decreased arterial blood flow leading to hyperemia
3. Atherosclerotic obstruction of the arteries
4. Trauma to the lower extremities
47. Which of the following instructions should be included in the discharge teaching for a
patient discharged with a transdermal nitroglycerin patch?
1. Apply the patch to a non hairy, nonfatty area of the upper torso or arms.
2. Apply the patch to the same site each day to maintain consistent drug absorption.
3. If you get a headache, remove the patch for 4 hours and then reapply.
4. If you get chest pain, apply a second patch right next to the first patch.
48. In order to prevent the development of tolerance, the nurse instructs the patient to:
1. Apply the nitroglycerin patch every other day
2. Switch to sublingual nitroglycerin when the patients systolic blood pressure elevates to >140 mm
Hg
3. Apply the nitroglycerin patch for 14 hours each and remove for 10 hours at night
4. Use the nitroglycerin patch for acute episodes of angina only
49. Direct-acting vasodilators have which of the following effects on the heart rate?
1. Heart rate decreases
2. Heart rate remains significantly unchanged
3. Heart rate increases
4. Heart rate becomes irregular
50. When teaching a patient why spironolactone (Aldactone) and furosemide (Lasix) are
prescribed together, the nurse bases teaching on the knowledge that:
1. Moderate doses of two different types of diuretics are more effective than a large dose of one type
2. This combination promotes diuresis but decreases the risk of hypokalemia
3. This combination prevents dehydration and hypovolemia
4. Using two drugs increases osmolality of plasma and the glomerular filtration rate

Answers and Rationale


Gauge your performance by counter checking your answers to the answers below. Learn more about
the question by reading the rationale. If you have any disputes or questions, please direct them to the
comments section.

12

1. Answer: 4. This procedure requires an informed consent because it involves injection of a


radiopaque dye into the blood vessel. The risk of allergic reaction and possible anaphylaxis is serious
and must be assessed before the procedure.
2. Answer: 3. Chest pain is assessed by using the standard pain assessment parameters. Options 1, 2,
and 4 may or may not help discriminate the origin of pain. Pain of pleuropulmonary origin usually
worsens on inspiration.
3. Answer: 2. On transfer from the CCU, the client is allowed self-care activities and bathroom
privileges. Supervised ambulation for brief distances are encouraged, with distances gradually
increased (50, 100, 200 feet).
4. Answer: 1. Edema, the accumulation of excess fluid in the interstitial spaces, can be measured by
intake greater than output and by a sudden increase in weight. Diuretics should be given in the
morning whenever possible to avoid nocturia. Strict sodium restrictions are reserved for clients with
severe symptoms.
5. Answer: 1. Sudden loss of electrocardiogram complexes indicates ventricular asystole or possible
electrode displacement. Accurate assessment of the client and equipment is necessary to determine
the cause and identify the appropriate intervention.
6. Answer: 4. BP should be taken with the client seated with the arm bared, positioned with support
and at heart level. The client should sit with the legs on the floor, feet uncrossed, and not speak during
the recording. The client should not have smoked tobacco or taken in caffeine in the 30 minutes
preceding the measurement. The client should rest quietly for 5 minutes before the reading is taken.
The cuff bladder should encircle at least 80% of the limb being measured. Gauges other than a
mercury sphygmomanometer should be calibrated every 6 months to ensure accuracy.
7. Answer: 4. The antidote to heparin is protamine sulfate and should be readily available for use if
excessive bleeding or hemorrhage should occur. Vitamin K is an antidote for warfarin.
8.
. Detection of myoglobin is one diagnostic tool to determine whether myocardial damage has occurred.
Myoglobin is generally Answer: 3. The therapeutic range for prothrombin time is 1.5 to 2 times the
control for clients at risk for thrombus. Based on the clients control value, the therapeutic range for
this individual would be 16.5 to 22 seconds. Therefore the result is within therapeutic range.
9. Answer: 2. Warfarin works in the liver and inhibits synthesis of four vitamin K-dependent clotting
factors (X, IX, VII, and II), but it takes 3 to 4 days before the therapeutic effect of warfarin is exhibited.
10. Answer: 1. Although obtaining the ECG, chest x-ray, and blood work are all important, the nurses
priority action would be to relieve the crushing chest pain.
11. Answer: 2. Thrombolytic drugs are administered within the first 6 hours after onset of a MI to lyse
clots and reduce the extent of myocardial damage.
12. Answer: 1, 3, 5. In a client who has had an ECG, the P wave represents the activation of the
electrical impulse in the SA node, which is then transmitted to the AV node. In addition, the P wave
represents atrial muscle depolarization, not ventricular depolarization. The normal duration of the P
wave is 0.11 seconds or less in duration and 2.5 mm or more in height.
13. Answer: 2. Advanced cardiac life support recommends that at least one or two intravenous lines be
inserted in one or both of the antecubital spaces. Calling the physician, obtaining a portable chest
radiograph, and drawing blood are important but secondary to starting the intravenous line.
14. Answer: 4detected about one hour after a heart attack is experienced and peaks within 4 to 6
hours after infarction (Remember, less than 90 mg/L is normal).
15. Answer: 1. Propranolol hydrochloride is a beta-adrenergic blocking agent. Actions of propranolol
hydrochloride include reducing heart rate, decreasing myocardial contractility, and slowing conduction.

13

16. Answer: 3. Compliance is the most critical element of hypertensive therapy. In most cases,
hypertensive clients require lifelong treatment and their hypertension cannot be managed successfully
without drug therapy. Stress management and weight management are important components of
hypertension therapy, but the priority goal is related to compliance.
17. Answer: 1. Hypertension is referred to as the silent killer for adults, because until the adult has
significant damage to other systems, the hypertension may go undetected. CVAs can be related to
long-term hypertension. Liver or pulmonary disease is generally not associated with hypertension.
Myocardial infarction is generally related to coronary artery disease.
18. Answer: 3. Nitroglycerin may be used prophylactically before stressful physical activities such as
stair climbing to help the client remain pain free. Visiting her friend early in the day would have no
impact on decreasing pain episodes. Resting before or after an activity is not as likely to help prevent
an activity-related pain episode.
19. Answer: 1. The client should report a change in the pattern of chest pain. It may indicate increasing
severity of CAD.
20. Answer: 2. Cardiac catheterization is done in clients with angina primarily to assess the extent and
severity of the coronary artery blockage, A decision about medical management, angioplasty, or
coronary artery bypass surgery will be based on the catheterization results.
21. Answer: 3. Nitroglycerin produces peripheral vasodilation, which reduces myocardial oxygen
consumption and demand. Vasodilation in coronary arteries and collateral vessels may also increase
blood flow to the ischemic areas of the heart. Nitroglycerin decreases myocardial oxygen demand.
Nitroglycerin does not have an effect on pericardial spasticity or conductivity in the myocardium.
22. Answer: 1. Because of the widespread vasodilating effects, nitroglycerin often produces such side
effects as headache, hypotension, and dizziness. The client should lie or shit down to avoid fainting.
Nitro does not cause shortness of breath or stomach cramps.
23. Answer: 3. The correct protocol for nitroglycerin used involves immediate administration, with
subsequent doses taken at 5-minute intervals as needed, for a total dose of 3 tablets. Sublingual
nitroglycerin appears in the bloodstream within 2 to 3 minutes and is metabolized within about 10
minutes.
24. Answer: 3. The left anterior descending artery is the primary source of blood flow for the anterior
wall of the heart. The circumflex artery supplies the lateral wall, the internal mammary supplies the
mammary, and the right coronary artery supplies the inferior wall of the heart.
25. Answer: 2. Although the coronary arteries may receive a minute portion of blood during systole,
most of the blood flow to coronary arteries is supplied during diastole. Breathing patterns are irrelevant
to blood flow.
26. Answer: 3. The right coronary artery supplies the right ventricle, or the inferior portion of the heart.
Therefore, prolonged occlusion could produce an infarction in that area. The right coronary artery
doesnt supply the anterior portion (left ventricle), lateral portion (some of the left ventricle and the left
atrium), or the apical portion (left ventricle) of the heart.
27. Answer: 3. Abnormalities of the pulmonic valve are auscultated at the second left intercostal space
along the left sternal border. Aortic valve abnormalities are heard at the second intercostal space, to
the right of the sternum. Mitral valve abnormalities are heard at the fifth intercostal space in the
midclavicular line. Tricupsid valve abnormalities are heard at the 3rd and 4th intercostal spaces along
the sternal border.
28. Answer: 3. Troponin I levels rise rapidly and are detectable within 1 hour of myocardial injury.
Troponin levels arent detectable in people without cardiac injury.
29. Answer: 4. The ECG is the quickest, most accurate, and most widely used tool to determine the
location of myocardial infarction. Cardiac enzymes are used to diagnose MI but cant determine the

14

location. An echocardiogram is used most widely to view myocardial wall function after an MI has been
diagnosed. Cardiac catheterization is an invasive study for determining coronary artery disease and
may also indicate the location of myocardial damage, but the study may not be performed
immediately.
30. Answer: 4. The pain of angina usually ranges from a vague feeling of tightness to heavy, intense
pain. Pain impulses originate in the most visceral muscles and may move to such areas as the chest,
neck, and arms.
31. Answer: 4. Vascular resistance is the impedance of blood flow by the arterioles that most
predominantly affects the diastolic pressure. Cardiac output determines systolic blood pressure.
32. Answer: 3. The kidneys respond to a rise in blood pressure by excreting sodium and excess water.
This response ultimately affects systolic pressure by regulating blood volume.
33. Answer: 1. Baroreceptors located in the carotid arteries and aorta sense pulsatile pressure.
Decreases in pulsatile pressure cause a reflex increase in heart rate. Chemoreceptors in the medulla
are primarily stimulated by carbon dioxide. Peripheral chemoreceptors in the aorta and carotid arteries
are primarily stimulated by oxygen.
34. Answer: 1. Afterload refers to the resistance normally maintained by the aortic and pulmonic
valves, the condition and tone of the aorta, and the resistance offered by the systemic and pulmonary
arterioles. Cardiac output is the amount of blood expelled from the heart per minute. Overload refers
to an abundance of circulating volume. Preload is the volume of blood in the ventricle at the end of
diastole.
35. Answer: 4. Preload is the amount of stretch of the cardiac muscle fibers at the end of diastole. The
volume of blood in the ventricle at the end of diastole determines the preload. Afterload is the force
against which the ventricle must expel blood. Cardiac index is the individualized measurement of
cardiac output, based on the clients body surface area. Cardiac output is the amount of blood the
heart is expelling per minute.
36. Answer: 3. Propranolol and other beta-adrenergic blockers are contraindicated in a client with
asthma, so the nurse should question the physician before giving the dose. The other responses are
appropriate actions for a client receiving propranolol, but questioning the physician takes priority. The
clients apical pulse should always be checked before giving propranolol; if the pulse rate is extremely
low, the nurse should withhold the drug and notify the physician.
37. Answer: 3. Furosemide is a potassium-depleting diuretic than can cause hypokalemia. In turn,
hypokalemia increases myocardial excitability, leading to ventricular tachycardia.
38. Answer: 4. Because spironolactone is a potassium-sparing diuretic, the client should avoid salt
substitutes because of their high potassium content. The client should also avoid potassium-rich foods
and potassium supplements. To reduce fluid-volume overload, sodium restrictions should continue.
39. Answer: 4. The P-R interval is measured on the ECG strip from the beginning of the P wave to the
beginning of the QRS complex. It is the time it takes for the impulse to travel to the ventricle.
40. Answer: 1. Such a response does not have false hope to the client but is positive and realistic. The
answer tells the client what cardiac rehabilitation is and does not dwell upon his negativity about it.
41. Answer: 3. Palpating pulses distal to the insertion site is important to evaluate for thrombophlebitis
and vessel occlusion. They should be bilateral and strong.
42. Answer: 4. This scan detects myocardial damage and perfusion, an acute or chronic MI. It is a more
specific answer than (1) or (2). Specific ventricular function is tested by a gated cardiac blood pool
scan.
43. Answer: 3. Even though initial tests seem to be within normal range, it takes at least 3 hours for
the cardiac enzyme studies to register. In the meantime, the client needs to be watched for

15

bradycardia, heart block, ventricular irritability, and other arrhythmias. Other activities can be
accomplished around the MI monitoring.
44. Answer: 3. The arterial oxygen supply is lowered and the demand for oxygen is increased, which
results in the hearts having to beat faster to meet the bodys needs for oxygen.
45. Answer: 1. Creatine kinase (CK, formally known as CPK) rises in 3-8 hours if an MI is present. When
the myocardium is damaged, CPK leaks out of the cell membranes and into the bloodstream. Lactic
dehydrogenase rises in 24-48 hours, and LDH-1 and LDH-2 rises in 8-24 hours.
46. Answer: 1. Decreased arterial flow is a result of vasospasm. The etiology is unknown. It is more
problematic in colder climates or when the person is under stress. Hyperemia occurs when the
vasospasm is relieved.
47. Answer: 1. A nitroglycerin patch should be applied to a non hairy, nonfatty area for the best and
most consistent absorption rates. Sites should be rotated to prevent skin irritation, and the drug should
be continued if headache occurs because tolerance will develop. Sublingual nitroglycerin should be
used to treat chest pain.
48. Answer: 3. Tolerance can be prevented by maintaining an 8- to 12-hour nitrate-free period each
day.
49. Answer: 3. Heart rate increases in response to decreased blood pressure caused by vasodilation.
50. Answer: 2. Spironolactone is a potassium-sparing diuretic; furosemide is a potassium-losing
diuretic. Giving these together minimizes electrolyte imbalance.

Introduction
This is a 40-item examination about Hematologic Disorders like Hemophilia, Sickle Cell Disease,
Anemia and Polycythemia Vera. This is an NCLEX style examination.

Topics

Hemophilia

Sickle Cell Disease

Anemia

Polycythemia Vera

Guidelines

Read each question carefully and choose the best answer.

You are given one minute per question. Spend your time wisely!

Answers and rationales are given below. Be sure to read them.

If you need more clarifications, please direct them to the comments section.

Questions
1. The nurse is preparing to teach a client with microcytic hypochromic anemia about the
diet to follow after discharge. Which of the following foods should be included in the diet?
1. Eggs
2. Lettuce
3. Citrus fruits
4. Cheese
2. The nurse would instruct the client to eat which of the following foods to obtain the best
supply of vitamin B12?

16

1. Whole grains
2. Green leafy vegetables
3. Meats and dairy products
4. Broccoli and Brussels sprouts
3. The nurse has just admitted a 35-year-old female client who has a serum B12
concentration of 800 pg/ml. Which of the following laboratory findings would cue the nurse
to focus the client history on specific drug or alcohol abuse?
1. Total bilirubin, 0.3 mg/dL
2. Serum creatinine, 0.5 mg/dL
3. Hemoglobin, 16 g/dL
4. Folate, 1.5 ng/mL
4. The nurse understands that the client with pernicious anemia will have which
distinguishing laboratory findings?
1. Schillings test, elevated
2. Intrinsic factor, absent.
3. Sedimentation rate, 16 mm/hour
4. RBCs 5.0 million
5. The nurse devises a teaching plan for the patient with aplastic anemia. Which of the
following is the most important concept to teach for health maintenance?
1. Eat animal protein and dark leafy vegetables each day
2. Avoid exposure to others with acute infection
3. Practice yoga and meditation to decrease stress and anxiety
4. Get 8 hours of sleep at night and take naps during the day
6. A client comes into the health clinic 3 years after undergoing a resection of the terminal
ileum complaining of weakness, shortness of breath, and a sore tongue. Which client
statement indicates a need for intervention and client teaching?
1. I have been drinking plenty of fluids.
2. I have been gargling with warm salt water for my sore tongue.
3. I have 3 to 4 loose stools per day.
4. I take a vitamin B12 tablet every day.
7. A vegetarian client was referred to a dietitian for nutritional counseling for anemia.
Which client outcome indicates that the client does not understand nutritional counseling?
The client:
1. Adds dried fruit to cereal and baked goods
2. Cooks tomato-based foods in iron pots
3. Drinks coffee or tea with meals
4. Adds vitamin C to all meals
8. A client was admitted with iron deficiency anemia and blood-streaked emesis. Which
question is most appropriate for the nurse to ask in determining the extent of the clients
activity intolerance?
1. What activities were you able to do 6 months ago compared with the present?
2. How long have you had this problem?
3. Have you been able to keep up with all your usual activities?
4. Are you more tired now than you used to be?
9. The primary purpose of the Schilling test is to measure the clients ability to:
1. Store vitamin B12
2. Digest vitamin B12

17

3. Absorb vitamin B12


4. Produce vitamin B12
10. The nurse implements which of the following for the client who is starting a Schilling
test?
1. Administering methylcellulose (Citrucel)
2. Starting a 24- to 48 hour urine specimen collection
3. Maintaining NPO status
4. Starting a 72 hour stool specimen collection
11. A client with pernicious anemia asks why she must take vitamin B12 injections for the
rest of her life. What is the nurses best response?
1. The reason for your vitamin deficiency is an inability to absorb the vitamin because the stomach is
not producing sufficient acid.
2. The reason for your vitamin deficiency is an inability to absorb the vitamin because the stomach is
not producing sufficient intrinsic factor.
3. The reason for your vitamin deficiency is an excessive excretion of the vitamin because of kidney
dysfunction.
4. The reason for your vitamin deficiency is an increased requirement for the vitamin because of rapid
red blood cell production.
12. The nurse is assessing a clients activity intolerance by having the client walk on a
treadmill for 5 minutes. Which of the following indicates an abnormal response?
1. Pulse rate increased by 20 bpm immediately after the activity
2. Respiratory rate decreased by 5 breaths/minute
3. Diastolic blood pressure increased by 7 mm Hg
4. Pulse rate within 6 bpm of resting phase after 3 minutes of rest.
13. When comparing the hematocrit levels of a post-op client, the nurse notes that the
hematocrit decreased from 36% to 34% on the third day even though the RBC and
hemoglobin values remained stable at 4.5 million and 11.9 g/dL, respectively. Which
nursing intervention is most appropriate?
1. Check the dressing and drains for frank bleeding
2. Call the physician
3. Continue to monitor vital signs
4. Start oxygen at 2L/min per NC
14. A client is to receive epoetin (Epogen) injections. What laboratory value should the
nurse assess before giving the injection?
1. Hematocrit
2. Partial thromboplastin time
3. Hemoglobin concentration
4. Prothrombin time
15. A client states that she is afraid of receiving vitamin B12 injections because of the
potential toxic reactions. What is the nurses best response to relieve these fears?
1. Vitamin B12 will cause ringing in the eats before a toxic level is reached.
2. Vitamin B12 may cause a very mild skin rash initially.
3. Vitamin B12 may cause mild nausea but nothing toxic.
4. Vitamin B12 is generally free of toxicity because it is water soluble.
16. A client with microcytic anemia is having trouble selecting food items from the
hospital menu. Which food is best for the nurse to suggest for satisfying the clients
nutritional needs and personal preferences?

18

1. Egg yolks
2. Brown rice
3. Vegetables
4. Tea
17. A client with macrocytic anemia has a burn on her foot and states that she had been
watching television while lying on a heating pad. What is the nurses first response?
1. Assess for potential abuse
2. Check for diminished sensations
3. Document the findings
4. Clean and dress the area
18. Which of the following nursing assessments is a late symptom of polycythemia vera?
1. Headache
2. Dizziness
3. Pruritus
4. Shortness of breath
19. The nurse is teaching a client with polycythemia vera about potential complications
from this disease. Which manifestations would the nurse include in the clients teaching
plan? Select all that apply.
1. Hearing loss
2. Visual disturbance
3. Headache
4. Orthopnea
5. Gout
6. Weight loss
20. When a client is diagnosed with aplastic anemia, the nurse monitors for changes in
which of the following physiological functions?
1. Bleeding tendencies
2. Intake and output
3. Peripheral sensation
4. Bowel function
21. Which of the following blood components is decreased in anemia?
1. Erythrocytes
2. Granulocytes
3. Leukocytes
4. Platelets
22. A client with anemia may be tired due to a tissue deficiency of which of the following
substances?
1. Carbon dioxide
2. Factor VIII
3. Oxygen
4. T-cell antibodies
23. Which of the following cells is the precursor to the red blood cell (RBC)?
1. B cell
2. Macrophage
3. Stem cell
4. T cell
24. Which of the following symptoms is expected with hemoglobin of 10 g/dl?

19

1. None
2. Pallor
3. Palpitations
4. Shortness of breath
25. Which of the following diagnostic findings are most likely for a client with aplastic
anemia?
1. Decreased production of T-helper cells
2. Decreased levels of white blood cells, red blood cells, and platelets
3. Increased levels of WBCs, RBCs, and platelets
4. Reed-Sternberg cells and lymph node enlargement
26. A client with iron deficiency anemia is scheduled for discharge. Which instruction
about prescribed ferrous gluconate therapy should the nurse include in the teaching plan?
1. Take the medication with an antacid.
2. Take the medication with a glass of milk.
3. Take the medication with cereal.
4. Take the medication on an empty stomach.
27. Which of the following disorders results from a deficiency of factor VIII?
1. Sickle cell disease
2. Christmas disease
3. Hemophilia A
4. Hemophilia B
28. The nurse explains to the parents of a 1-year-old child admitted to the hospital in a
sickle cell crisis that the local tissue damage the child has on admission is caused by which
of the following?
1. Autoimmune reaction complicated by hypoxia
2. Lack of oxygen in the red blood cells
3. Obstruction to circulation
4. Elevated serum bilirubin concentration.
29. The mothers asks the nurse why her childs hemoglobin was normal at birth but now
the child has S hemoglobin. Which of the following responses by the nurse is most
appropriate?
1. The placenta bars passage of the hemoglobin S from the mother to the fetus.
2. The red bone marrow does not begin to produce hemoglobin S until several months after birth.
3. Antibodies transmitted from you to the fetus provide the newborn with temporary immunity.
4. The newborn has a high concentration of fetal hemoglobin in the blood for some time after birth.
30. Which of the following would the nurse identify as the priority nursing diagnosis
during a toddlers vaso-occlusive sickle cell crisis?
1. Ineffective coping related to the presence of a life-threatening disease
2. Decreased cardiac output related to abnormal hemoglobin formation
3. Pain related to tissue anoxia
4. Excess fluid volume related to infection
31. A mother asks the nurse if her childs iron deficiency anemia is related to the childs
frequent infections. The nurse responds based on the understanding of which of the
following?
1. Little is known about iron-deficiency anemia and its relationship to infection in children.
2. Children with iron deficiency anemia are more susceptible to infection than are other children.

20

3. Children with iron-deficiency anemia are less susceptible to infection than are other children.
4. Children with iron-deficient anemia are equally as susceptible to infection as are other children.
32. Which statements by the mother of a toddler would lead the nurse to suspect that the
child has iron-deficiency anemia? Select all that apply.
1. He drinks over 3 cups of milk per day.
2. I cant keep enough apple juice in the house; he must drink over 10 ounces per day.
3. He refuses to eat more than 2 different kinds of vegetables.
4. He doesnt like meat, but he will eat small amounts of it.
5. He sleeps 12 hours every night and take a 2-hour nap.
33. Which of the following foods would the nurse encourage the mother to offer to her
child with iron deficiency anemia?
1. Rice cereal, whole milk, and yellow vegetables
2. Potato, peas, and chicken
3. Macaroni, cheese, and ham
4. Pudding, green vegetables, and rice
34. The physician has ordered several laboratory tests to help diagnose an infants
bleeding disorder. Which of the following tests, if abnormal, would the nurse interpret as
most likely to indicate hemophilia?
1. Bleeding time
2. Tourniquet test
3. Clot retraction test
4. Partial thromboplastin time (PTT)
35. Which of the following assessments in a child with hemophilia would lead the nurse to
suspect early hemarthrosis?
1. Childs reluctance to move a body part
2. Cool, pale, clammy extremity
3. Eccymosis formation around a joint
4. Instability of a long bone in passive movement
36. Because of the risks associated with administration of factor VIII concentrate, the
nurse would teach the clients family to recognize and report which of the following?
1. Yellowing of the skin
2. Constipation
3. Abdominal distention
4. Puffiness around the eyes
37. A child suspected of having sickle cell disease is seen in a clinic, and laboratory
studies are performed. A nurse checks the lab results, knowing that which of the following
would be increased in this disease?
1. Platelet count
2. Hematocrit level
3. Reticulocyte count
4. Hemoglobin level
38. A clinic nurse instructs the mother of a child with sickle cell disease about the
precipitating factors related to pain crisis. Which of the following, if identified by the
mother as a precipitating factor, indicates the need for further instructions?
1. Infection
2. Trauma

21

3. Fluid overload
4. Stress
39. Laboratory studies are performed for a child suspected of having iron deficiency
anemia. The nurse reviews the laboratory results, knowing that which of the following
results would indicate this type of anemia?
1. An elevated hemoglobin level
2. A decreased reticulocyte count
3. An elevated RBC count
4. Red blood cells that are microcytic and hypochromic
40. A pediatric nurse health educator provides a teaching session to the nursing staff
regarding hemophilia. Which of the following information regarding this disorder would the
nurse plan to include in the discussion?
1. Hemophilia is a Y linked hereditary disorder
2. Males inherit hemophilia from their fathers
3. Females inherit hemophilia from their mothers
4. Hemophilia A results from a deficiency of factor VIII

Answers and Rationale


1. Answer: 1. One of the microcytic, hypochromic anemias is iron-deficiency anemia. A rich source of
iron is needed in the diet, and eggs are high in iron. Other foods high in iron include organ and muscle
(dark) meats; shellfish, shrimp, and tuna; enriched, whole-grain, and fortified cereals and breads;
legumes, nuts, dried fruits, and beans; oatmeal; and sweet potatoes. Dark green leafy vegetables and
citrus fruits are good sources of vitamin C. Cheese is a good source of calcium.
2. Answer: 3. Good sources of vitamin B12 include meats and dairy products. Whole grains are a good
source of thiamine. Green leafy vegetables are good sources of niacin, folate, and carotenoids
(precursors of vitamin A). Broccoli and Brussels sprouts are good sources of ascorbic acid (vitamin C).
3. Answer: 4. The normal range of folic acid is 1.8 to 9 ng/mL, and the normal range of vitamin B12 is
200 to 900 pg/mL. A low folic acid level in the presence of a normal vitamin B12 level is indicative of a
primary folic acid-deficiency anemia. Factors that affect the absorption of folic acid are drugs such as
methotrexate, oral contraceptives, antiseizure drugs, and alcohol. The total bilirubin, serum creatinine,
and hemoglobin values are within normal limits.
4. Answer: 2. The defining characteristic of pernicious anemia, a megaloblastic anemia, is lack of the
intrinsic factor, which results from atrophy of the stomach wall. Without the intrinsic factor, vitamin
B12 cannot be absorbed in the small intestines, and folic acid needs vitamin B12 for DNA synthesis of
RBCs. The gastric analysis was done to determine the primary cause of the anemia. An elevated
excretion of the injected radioactive vitamin B12, which is protocol for the first and second stage of the
Schilling test, indicates that the client has the intrinsic factor and can absorb vitamin B12 into the
intestinal tract. A sedimentation rate of 16 mm/hour is normal for both men and women and is a
nonspecific test to detect the presence of inflammation. It is not specific to anemias. An RBC value of
5.0 million is a normal value for both men and women and does not indicate anemia.
5. Answer: 2. Clients with aplastic anemia are severely immunocompromised and at risk for infection
and possible death related to bone marrow suppression and pancytopenia. Strict aseptic technique and
reverse isolation are important measures to prevent infection. Although diet, reduced stress, and rest
are valued in supporting health, the potentially fatal consequence of an acute infection places it as a
priority for teaching the client about health maintenance. Animal meat and dark green leafy
vegetables, good sources of vitamin B12 and folic acid, should be included in the daily diet. Yoga and

22

meditation are good complimentary therapies to reduce stress. Eight hours of rest and naps are good
for spacing and pacing activity and rest.
6. Answer: 4. Vitamin B12 combines with intrinsic factor in the stomach and is then carried to the
ileum, where it is absorbed in the bloodstream. In this situation, vitamin B12 cannot be absorbed
regardless of the amount of oral intake of sources of vitamin B12 such as animal protein or vitamin B12
tablets. Vitamin B12 needs to be injected every month, because the ileum has been surgically
removed. Replacement of fluids and electrolytes is important when the client has continuous multiple
loose stools on a daily basis. Warm salt water is used to soothe sore mucous membranes. Crohns
disease and small bowel resection may cause several loose stools a day.
7. Answer: 3. Coffee and tea increase gastrointestinal motility and inhibit the absorption of nonheme
iron. Clients are instructed to add dried fruits to dishes at every meal because dried fruits are a
nonheme or nonanimal iron source. Cooking in iron cookware, especially acid-based foods such as
tomatoes, adds iron to the diet. Clients are instructed to add a rich supply of vitamin C to every meal
because the absorption of iron is increased when food with vitamin C or ascorbic acid is consumed.
8. Answer: 1. It is difficult to determine activity intolerance without objectively comparing activities
from one time frame to another. Because iron deficiency anemia can occur gradually and individual
endurance varies, the nurse can best assess the clients activity tolerance by asking the client to
compare activities 6 months ago and at the present. Asking a client how long a problem has existed is
a very open-ended question that allows for too much subjectivity for any definition of the clients
activity tolerance. Also, the client may not even identify that a problem exists. Asking the client
whether he is staying abreast of usual activities addresses whether the tasks were completed, not the
tolerance of the client while the tasks were being completed or the resulting condition of the client
after the tasks were completed. Asking the client if he is more tired now than usual does not address
his activity tolerance. Tiredness is a subjective evaluation and again can be distorted by factors such
as the gradual onset of the anemia or the endurance of the individual.
9. Answer: 3. Pernicious anemia is caused by the bodys inability to absorb vitamin B12. This results in
a lack of intrinsic factor in the gastric juices. Schillings test helps diagnose pernicious anemia by
determining the clients ability to absorb vitamin B12.
10. Answer: 2. Urinary vitamin B12 levels are measured after the ingestion of radioactive vitamin B12.
A 24-to 48- hour urine specimen is collected after administration of an oral dose of radioactively
tagged vitamin B12 and an injection of non-radioactive vitamin B12. In a healthy state of absorption,
excess vitamin B12 is excreted in the urine; in a malabsorption state or when the intrinsic factor is
missing, vitamin B12 is excreted in the feces. Citrucel is a bulk-forming agent. Laxatives interfere with
the absorption of vitamin B12. The client is NPO 8 to 12 hours before the test but is not NPO during the
test. A stool collection is not part of the Schilling test. If stool contaminates the urine collection, the
results will be altered.
11. Answer: 2. Most clients with pernicious anemia have deficient production of intrinsic factor in the
stomach. Intrinsic factor attaches to the vitamin in the stomach and forms a complex that allows the
vitamin to be absorbed in the small intestine. The stomach is producing enough acid, there is not an
excessive excretion of the vitamin, and there is not a rapid production of RBCs in this condition.
12. Answer: 2. The normal physiologic response to activity is an increased metabolic rate over the
resting basal rate. The decrease in respiratory rate indicates that the client is not strong enough to
complete the mechanical cycle of respiration needed for gas exchange. The post activity pulse is
expected to increase immediately after activity but by no more than 50 bpm if it is strenuous activity.
The diastolic blood pressure is expected to rise but by no more than 15 mm Hg. The pulse returns to
within 6 bpm of the resting pulse after 3 minutes of rest.

23

13. Answer: 3. The nurse should continue to monitor the client, because this value reflects a normal
physiologic response. The physician does not need to be called, and oxygen does not need to be
started based on these laboratory findings. Immediately after surgery, the clients hematocrit reflects a
falsely high value related to the bodys compensatory response to the stress of sudden loss of fluids
and blood. Activation of the intrinsic pathway and the renin-angiotensin cycle via antidiuretic hormone
produces vasoconstriction and retention of fluid for the first 1 to 2 day post-op. By the second to third
day, this response decreases and the clients hematocrit level is more reflective of the amount of RBCs
in the plasma. Fresh bleeding is a less likely occurrence on the third post-op day but is not impossible;
however, the nurse would have expected to see a decrease in the RBC and hemoglobin values
accompanying the hematocrit.
14. Answer: 1. Epogen is a recombinant DNA form of erythropoietin, which stimulates the production of
RBCs and therefore causes the hematocrit to rise. The elevation in hematocrit causes an elevation in
blood pressure; therefore, the blood pressure is a vital sign that should be checked. The PTT,
hemoglobin level, and PT are not monitored for this drug.
15. Answer: 4. Vitamin B12 is a water-soluble vitamin. When water-soluble vitamins are taken in excess
of the bodys needs, they are filtered through the kidneys and excreted. Vitamin B12 is considered to
be nontoxic. Adverse reactions that have occurred are believed to be related to impurities or to the
preservative in B12 preparations. Ringing in the ears, skin rash, and nausea are not considered to be
related to vitamin B12 administration.
16. Answer: 2. Brown rice is a source of iron from plant sources (nonheme iron). Other sources of non
heme iron are whole-grain cereals and breads, dark green vegetables, legumes, nuts, dried fruits
(apricots, raisins, dates), oatmeal, and sweet potatoes. Egg yolks have iron but it is not as well
absorbed as iron from other sources. Vegetables are a good source of vitamins that may facilitate iron
absorption. Tea contains tannin, which combines with nonheme iron, preventing its absorption.
17. Answer: 2. Macrocytic anemias can result from deficiencies in vitamin B12 or ascorbic acid. Only
vitamin B12 deficiency causes diminished sensations of peripheral nerve endings. The nurse should
assess for peripheral neuropathy and instruct the client in self-care activities for her diminished
sensation to heat and pain. The burn could be related to abuse, but this conclusion would require more
supporting data. The findings should be documented, but the nurse would want to address the clients
sensations first. The decision of how to treat the burn should be determined by the physician.
18. Answer: 3. Pruritus is a late symptom that results from abnormal histamine metabolism. Headache
and dizziness are early symptoms from engorged veins. Shortness of breath is an early symptom from
congested mucous membrane and ineffective gas exchange.
19. Answer: 2, 3, 4, 5. Polycythemia vera, a condition in which too many RBCs are produced in the
blood serum, can lead to an increase in the hematocrit and hypervolemia, hyperviscosity, and
hypertension. Subsequently, the client can experience dizziness, tinnitus, visual disturbances,
headaches, or a feeling of fullness in the head. The client may also experience cardiovascular
symptoms such as heart failure (shortness of breath and orthopnea) and increased clotting time or
symptoms of an increased uric acid level such as painful swollen joints (usually the big toe). Hearing
loss and weight loss are not manifestations associated with polycythemia vera.
20. Answer: 1. Aplastic anemia decreases the bone marrow production of RBCs, WBCs, and platelets.
The client is at risk for bruising and bleeding tendencies. A change in the intake and output is
important, but assessment for the potential for bleeding takes priority. Change in the peripheral
nervous system is a priority problem specific to clients with vitamin B12 deficiency. Change in bowel
function is not associated with aplastic anemia.

24

21. Answer: 1. Anemia is defined as a decreased number of erythrocytes (red blood cells). Leukopenia
is a decreased number of leukocytes (white blood cells). Thrombocytopenia is a decreased number of
platelets. Lastly, granulocytopenia is a decreased number of granulocytes (a type of white blood cells).
22. Answer: 3. Anemia stems from a decreased number of red blood cells and the resulting deficiency
in oxygen and body tissues. Clotting factors, such as factor VIII, relate to the bodys ability to form
blood clots and arent related to anemia, not is carbon dioxide of T antibodies.
23. Answer: 3. The precursor to the RBC is the stem cell. B cells, macrophages, and T cells and
lymphocytes, not RBC precursors.
24. Answer: 1. Mild anemia usually has no clinical signs. Palpitations, SOB, and pallor are all associated
with severe anemia.
25. Answer: 2. In aplastic anemia, the most likely diagnostic findings are decreased levels of all the
cellular elements of the blood (pancytopenia). T-helper cell production doesnt decrease in aplastic
anemia. Reed-Sternberg cells and lymph node enlargement occur with Hodgkins disease.
26. Answer: 4. Preferably, ferrous gluconate should be taken on an empty stomach. Ferrous gluconate
should not be taken with antacids, milk, or whole-grain cereals because these foods reduce iron
absorption.
27. Answer: 3. Hemophilia A results from a deficiency of factor VIII. Sickle cell disease is caused by a
defective hemoglobin molecule. Christmas disease, also called hemophilia B, results in a factor IX
deficiency.
28. Answer: 3. Characteristic sickle cells tend to cause log jams in capillaries. This results in poor
circulation to local tissues, leading to ischemia and necrosis. The basic defect in sickle cell disease is
an abnormality in the structure of RBCs. The erythrocytes are sickle-shaped, rough in texture, and
rigid. Sickle cell disease is an inherited disease, not an autoimmune reaction. Elevated serum bilirubin
concentrations are associated with jaundice, not sickle cell disease.
29. Answer: 4. Sickle cell disease is an inherited disease that is present at birth. However, 60% to 80%
of a newborns hemoglobin is fetal hemoglobin, which has a structure different from that of hemoglobin
S or hemoglobin A. Sickle cell symptoms usually occur about 4 months after birth, when hemoglobin S
begins to replace the fetal hemoglobin. The gene for sickle cell disease is transmitted at the time of
conception, not passed through the placenta. Some hemoglobin S is produced by the fetus near term.
The fetus produces all its own hemoglobin from the earliest production in the first trimester. Passive
immunity conferred by maternal antibodies is not related to sickle cell disease, but this transmission of
antibodies is important to protect the infant from various infections during early infancy.
30. Answer: 3. For the child in a sickle cell crisis, pain is the priority nursing diagnosis because the
sickled cells clump and obstruct the blood vessels, leading to occlusion and subsequent tissue
ischemia. Although ineffective coping may be important, it is not the priority. Decreased cardiac output
is not a problem with this type of vaso occlusive crisis. Typically, a sickle cell crisis can be precipitated
by a fluid volume deficit or dehydration.
31. Answer: 2. Children with iron-deficiency anemia are more susceptible to infection because of
marked decreases in bone marrow functioning with microcytosis.
32. Answer: 1, 2. Toddlers should have between 2 and 3 cups of milk per day and 8 ounces of juice per
day. If they have more than that, then they are probably not eating enough other foods, including ironrich foods that have the needed nutrients.
33. Answer: 2. Potato, peas, chicken, green vegetables, and rice cereal contain significant amounts of
iron and therefore would be recommended. Milk and yellow vegetables are not good iron sources. Rice
by itself also is not a good source of iron.
34. Answer: 4. PTT measures the activity of thromboplastin, which is dependent on intrinsic clotting
factors. In hemophilia, the intrinsic clotting factor VIII (antihemophilic factor) is deficient, resulting in a

25

prolonged PTT. Bleeding time reflects platelet function; the tourniquet test measures vasoconstriction
and platelet function; and the clot retraction test measures capillary fragility. All of these are
unaffected in people with hemophilia.
35. Answer: 1. Bleeding into the joints in the child with hemophilia leads to pain and tenderness,
resulting in restricted movement. Therefore, an early sign of hemarthrosis would be the childs
reluctance to move a body part. If the bleeding into the joint continues, the area becomes hot, swollen,
and immobilenot cool, pale, and clammy. Ecchymosis formation around a joint would be difficult to
assess. Instability of a long bone on passive movement is not associated with joint hemarthrosis.
36. Answer: 1. Because factor VIII concentrate is derived from large pools of human plasma, the risk of
hepatitis is always present. Clinical manifestations of hepatitis include yellowing of the skin, mucous
membranes, and sclera. Use of factor VIII concentrate is not associated with constipation, abdominal
distention, or puffiness around the eyes.
37. Answer: 3. A diagnosis is established based on a complete blood count, examination for sickled red
blood cells in the peripheral smear, and hemoglobin electrophoresis. Laboratory studies will show
decreased hemoglobin and hematocrit levels and a decreased platelet count, and increased
reticulocyte count, and the presence of nucleated red blood cells. Increased reticulocyte counts occur
in children with sickle cell disease because the life span of their sickled red blood cells is shortened.
38. Answer: 3. Pain crisis may be precipitated by infection, dehydration, hypoxia, trauma, or physical
or emotional stress. The mother of a child with sickle cell disease should encourage fluid intake of 1
to 2 times the daily requirement to prevent dehydration.
39. Answer: 4. The results of a CBC in children with iron deficiency anemia will show decreased
hemoglobin levels and microcytic and hypochromic red blood cells. The red blood cell count is
decreased. The reticulocyte count is usually normal or slightly elevated.
40. Answer: 4. Males inherit hemophilia from their mothers, and females inherit the carrier status from
their fathers. Hemophilia is inherited in a recessive manner via a genetic defect on the X-chromosome.
Hemophilia A results from a deficiency of factor VIII. Hemophilia B (Christmas disease) is a deficiency
of factor IX.

Introduction
In this 20-item NCLEX style exam, you knowledge about the different Peripheral Vascular Diseases will
be challenged.

Topics

Peripheral Vascular Diseases

Heparin and Warfarin

Deep Vein Thrombosis

Guidelines

Read each question carefully and choose the best answer.

You are given one minute per question. Spend your time wisely!

Answers and rationales are given below. Be sure to read them.

If you need more clarifications, please direct them to the comments section.

26

Questions
1. The most important factor in regulating the caliber of blood vessels, which determines
resistance to flow, is:
1. Hormonal secretion
2. Independent arterial wall activity.
3. The influence of circulating chemicals
4. The sympathetic nervous system
2. With peripheral arterial insufficiency, leg pain during rest can be reduced by:
1. Elevating the limb above heart level
2. Lowering the limb so it is dependent
3. Massaging the limb after application of cold compresses
4. Placing the limb in a plane horizontal to the body
3. Buergers disease is characterized by all of the following except:
1. Arterial thrombosis formation and occlusion
2. Lipid deposits in the arteries
3. Redness or cyanosis in the limb when it is dependent
4. Venous inflammation and occlusion
4. A significant cause of venous thrombosis is:
1. Altered blood coagulation
2. Stasis of blood
3. Vessel wall injury
4. All of the above
5. When caring for a patient who has started anticoagulant therapy with warfarin
(Coumadin), the nurse knows not to expect therapeutic benefits for:
1. At least 12 hours
2. The first 24 hours
3. 2-3 days
4. 1 week
6. Mike, a 43-year old construction worker, has a history of hypertension. He smokes two
packs of cigarettes a day, is nervous about the possibility of being unemployed, and has
difficulty coping with stress. His current concern is calf pain during minimal exercise that
decreased with rest. The nurse assesses Mikes symptoms as being associated with
peripheral arterial occlusive disease. The nursing diagnosis is probably:
1. Alteration in tissue perfusion related to compromised circulation
2. Dysfunctional use of extremities related to muscle spasms
3. Impaired mobility related to stress associated with pain
4. Impairment in muscle use associated with pain on exertion.
7. A 24-year old man seeks medical attention for complaints of claudication in the arch of
the foot. A nurse also notes superficial thrombophlebitis of the lower leg. The nurse would
next assess the client for:
1. Familial tendency toward peripheral vascular disease
2. Smoking history
3. Recent exposures to allergens
4. History of insect bites
8. Intravenous heparin therapy is ordered for a client. While implementing this order, a
nurse ensures that which of the following medications is available on the nursing unit?

27

1. Vitamin K
2. Aminocaproic acid
3. Potassium chloride
4. Protamine sulfate
9. A client who has been receiving heparin therapy also is started on warfarin sodium
(coumadin). The client asks the nurse why both medications are being administered. In
formulating a response, the nurse incorporates the understanding that warfarin sodium:
1. Stimulates the breakdown of specific clotting factors by the liver, and it takes 2-3 days for this is
exhibit an anticoagulant effect.
2. Inhibits synthesis of specific clotting factors in the liver, and it takes 3 to 4 days for this medication
to exert an anticoagulation effect.
3. Stimulates production of the bodys own thrombolytic substances, but it takes 2-4 days for it to
begin.
4. Has the same mechanism action of heparin, and the crossover time is needed for the serum level of
warfarin sodium to be therapeutic.
10. A nurse has an order to begin administering warfarin sodium (coumadin) to a client.
While implementing this order, the nurse ensures that which of the following medications
is available on the nursing unit as the antidote for Coumadin?
1. Vitamin K
2. Aminocaproic acid
3. Potassium chloride
4. Protamine sulfate
11. A nurse is assessing the neurovascular status of a client who returned to the surgical
nursing unit 4 hours ago after undergoing aortoiliac bypass graft. The affected leg is warm,
and the nurse notes redness and edema. The pedal pulse is palpable and unchanged from
admission. The nurse interprets that the neurovascular status is:
1. Normal because of the increased blood flow through the leg
2. Slightly deteriorating and should be monitored for another hour
3. Moderately impaired, and the surgeon should be called.
4. Adequate from the arterial approach, but venous complications are arising.
12. A client is admitted with a venous stasis leg ulcer. A nurse assesses the ulcer,
expecting to note that the ulcer:
1. Has a pale colored base
2. Is deep, with even edges
3. Has little granulation tissue
4. Has brown pigmentation around it.
13. In preparation for discharge of a client with arterial insufficiency and Raynauds
disease, client teaching instructions should include:
1. Walking several times each day as an exercise program.
2. Keeping the heat up so that the environment is warm
3. Wearing TED hose during the day
4. Using hydrotherapy for increasing oxygenation
14. A client comes to the outpatient clinic and tells the nurse that he has had legs pains
that began when he walks but cease when he stops walking. Which of the following
conditions would the nurse assess for?
1. An acute obstruction in the vessels of the legs
2. Peripheral vascular problems in both legs

28

3. Diabetes
4. Calcium deficiency
15. Which of the following characteristics is typical of the pain associated with DVT?
1. Dull ache
2. No pain
3. Sudden onset
4. Tingling
16. Cancer can cause changes in what component of Virchows triad?
1. Blood coagulability
2. Vessel walls
3. Blood flow
4. Blood viscosity
17. Varicose veins can cause changes in what component of Virchows triad?
1. Blood coagulability
2. Vessel walls
3. Blood flow
4. Blood viscosity
18. Which technique is considered the gold standard for diagnosing DVT?
1. Ultrasound imaging
2. Venography
3. MRI
4. Doppler flow study
19. A nurse is assessing a client with an abdominal aortic aneurysm. Which of the following
assessment findings by the nurse is probably unrelated to the aneurysm?
1. Pulsatile abdominal mass
2. Hyperactive bowel sounds in that area
3. Systolic bruit over the area of the mass
4. Subjective sensation of heart beating in the abdomen.
20. A nurse is caring for a client who had a percutaneous insertion of an inferior vena cava
filter and was on heparin therapy before surgery. The nurse would inspect the surgical site
most closely for signs of:
1. Thrombosis and infection
2. Bleeding and infection
3. Bleeding and wound dehiscence.
4. Wound dehiscence and evisceration.

Answers and Rationale


1. Answer: 4
2. Answer: 2
3. Answer: 2
4. Answer: 4
5. Answer: 3
6. Answer: 1
7. Answer: 2. The mixture of arterial and venous manifestations (claudication and phlebitis,
respectively) in the young male client suggests Buergers disease. This is an uncommon disorder
characterized by inflammation and thrombosis of smaller arteries and veins. This disorder typically is

29

found in young adult males who smoke. The cause is not known precisely but is suspected to have an
autoimmune component.
8. Answer: 4. The antidote to heparin is protamine sulfate and should be readily available for use if
excessive bleeding or hemorrhage should occur
9. Answer: 2. Warfarin sodium works in the liver and inhibits synthesis of four vitamin K-dependent
clotting factors (X, IX, VII, and II), but it takes 3 to 4 days before the therapeutic effect of warfarin is
exhibited.
10. Answer: 1. The antidote to warfarin (Coumadin) is Vitamin K and should be readily available for use
if excessive bleeding or hemorrhage should occur.
11. Answer: 1. An expected outcome of surgery is warmth, redness, and edema in the surgical
extremity because of increased blood flow. Options 2, 3, and 4 are incorrect interpretations.
12. Answer: 4. Venous leg ulcers, also called stasis ulcers, tend to be more superficial than arterial
ulcers, and the ulcer bed is pink. The edges of the ulcer are uneven, and granulation tissue is evident.
The skin has a brown pigmentation from accumulation of metabolic waste products resulting from
venous stasis. The client also exhibits peripheral edema. (options 1, 2, and 3 is due to tissue
malnutrition; and thus us an arterial problem)
13. Answer: 2. The clients instructions should include keeping the environment warm to prevent
vasoconstriction. Wearing gloves, warm clothes, and socks will also be useful when preventing
vasoconstriction, but TED hose would not be therapeutic. Walking would most likely increase pain.
14. Answer: 2. Intermittent claudication is a condition that indicates vascular deficiencies in the
peripheral vascular system. If an obstruction were present, the leg pain would persist when the client
stops walking. Low calcium levels may cause leg cramps but would not necessarily be related to
walking.
15. Answer: 3. DVT is associated with deep leg pain of sudden onset, which occurs secondary to the
occlusion. A dull ache is more commonly associated with varicose veins. A tingling sensation is
associated with an alteration in arterial blood flow. If the thrombus is large enough, it will cause pain.
16. Answer: 1
17. Answer: 3
18. Answer: 2
19. Answer: 2. Not all clients with abdominal aortic aneurysms exhibit symptoms. Those who do
describe a feeling of the heart beating in the abdomen when supine or be able to feel the mass
throbbing. A pulsatile mass may be palpated in the middle and upper abdomen. A systolic bruit may be
auscultated over the mass. Hyperactive bowel sounds are not related specifically to an abdominal
aortic aneurysm.
20. Answer: 2. After inferior vena cava insertion, the nurse inspects the surgical site for bleeding and
signs and symptoms of infection. Otherwise, care is the same as for any post-op client.

Introduction
A simple 10 item NCLEX style examination about Valvular Diseases.

Topics

Valvular Diseases

Guidelines

Read each question carefully and choose the best answer.

You are given one minute per question. Spend your time wisely!

30

Answers and rationales are given below. Be sure to read them.

If you need more clarifications, please direct them to the comments section.

Questions
1. A 68-year-old woman is scheduled to undergo mitral valve replacement for severe mitral
stenosis and mitral regurgitation. Although the diagnosis was made during childhood, she
did not have any symptoms until 4 years ago. Recently, she noticed increased symptoms,
despite daily doses of digoxin and furosemide. During the initial interview with the nice
lady, the nurse would most likely learn that the clients childhood health history included:
1. Chicken pox
2. poliomyelitis
3. Rheumatic fever
4. meningitis
2. Which of the following signs and symptoms would most likely be found in a client with
mitral regurgitation?
1. Exertional dyspnea
2. Confusion
3. Elevated creatine phosphokinase concentration
4. Chest pain
3. The nurse expects that a client with mitral stenosis would demonstrate symptoms
associated with congestion in the:
1. Aorta
2. Right atrium
3. Superior vena cava
4. Pulmonary circulation
4. Because a client has mitral stenosis and is a prospective valve recipient, the nurse
preoperatively assesses the clients past compliance with medical regimens. Lack of
compliance with which of the following regimens would pose the greatest health hazard to
this client?
1. Medication therapy
2. Diet modification
3. Activity restrictions
4. Dental care
5. Good dental care is an important measure in reducing the risk of endocarditis. A
teaching plan to promote good dental care in a client with mitral stenosis should include
demonstration of the proper use of:
1. A manual toothbrush
2. An electric toothbrush
3. An irrigation device
4. Dental floss
6. A client has been admitted to the hospital with a diagnosis of suspected bacterial
endocarditis. The complication the nurse will constantly observe for is:
1. Presence of heart murmur
2. Systemic emboli
3. Fever
4. Congestive heart failure
7. Cholesterol, frequently discussed in relation to atherosclerosis, is a substance that:

31

1. May be controlled by eliminating food sources


2. Is found in many foods, both plant and animal sources
3. All persons would be better off without because it causes the disease process
4. Circulates in the blood, the level of which usually decreases when unsaturated fats are substituted
for saturated fats.
8. When teaching a client with a cardiac problem, who is on a high-unsaturated fatty-acid
diet, the nurse should stress the importance of increasing the intake of:
1. Enriched whole milk
2. Red meats, such as beef
3. Vegetables and whole grains
4. Liver and other glandular organ meats
9. A 2-gram sodium diet is prescribed for a client with severe hypertension. The client does
not like the diet, and the nurse hears the client request that the spouse Bring in some
good home-cooked food. It would be most effective for the nurse to plan to:
1. Call in the dietician for client teaching
2. Wait for the clients family and discuss the diet with the client and family
3. Tell the client that the use of salt is forbidden, because it will raise BP
4. Catch the family members before they go into the clients room and tell them about the diet.
10. What criteria should the nurse use to determine normal sinus rhythm for a client on a
cardiac monitor? Check all that apply.
1. The RR intervals are relatively consistent
2. One P wave precedes each QRS complex
3. Four to eight complexes occur in a 6-second strip
4. The ST segment is higher than the PR interval
5. The QRS complex ranges from 0.12 to 0.2 seconds

Answers and Rationale


Gauge your performance by counter checking your answers to the answers below. Learn more about
the question by reading the rationale. If you have any disputes or questions, please direct them to the
comments section.
1. Answer: 3. Most clients with mitral stenosis have a history of rheumatic fever or bacterial
endocarditis.
2. Answer: 1. Weight gain, due to fluid retention and worsening heart failure, causes exertional
dyspnea in clients with mitral regurgitation. The rise in left atrial pressure that accompanies mitral
valve disease is transmitted backward into pulmonary veins, capillaries, and arterioles and eventually
to the right ventricle. Signs and symptoms of pulmonary and systemic venous congestion follow.
3. Answer: 4. When mitral stenosis is present, the left atrium has difficulty emptying its contents into
the left ventricle. Hence, because there is no valve to prevent backward flow into the pulmonary vein,
the pulmonary circulation is under pressure.
4. Answer: 1. Preoperatively, anticoagulants may be prescribed for the client with advanced valvular
heart disease to prevent emboli. Post-op, all clients with mechanical valves and some with
bioprostheses are maintained indefinitely on anticoagulation therapy. Adhering strictly to a dosage
schedule and observing specific precautions are necessary to prevent hemorrhage or
thromboembolism. Some clients are maintained on lifelong antibiotic prophylaxis to prevent recurrence
from rheumatic fever. Episodic prophylaxis is required to prevent infective endocarditis after dental
procedures or upper respiratory, GI, or GU surgery.

32

5. Answer: 1. Daily dental care and frequent checkups by a dentist who is informed about the clients
condition are required to maintain good oral health. Use of an electric toothbrush, an irrigation device,
or dental floss may cause gums to bleed and allow bacteria to enter mucous membranes and the
bloodstream, increasing the risk of endocarditis.
6. Answer: 2. Emboli are the major problem; those arising in the right heart chambers will terminate in
the lungs and left chamber emboli may travel anywhere in the arteries. Heart murmurs, fever, and
night sweats may be present, but do not indicate a problem with emboli. CHF may be a result, but this
is not as dangerous an outcome as emboli.
7. Answer: 4. Cholesterol is a sterol found in tissue; it is attributed in part to diets high in saturated
fats.
8. Answer: 3. Vegetables and whole grains are low in fat and may reduce the risk for heart disease.
9. Answer: 2. Clients families should be included in dietary teaching; families provide support that
promotes adherence.
10. Answers: 1 and 2. (1) The consistency of the RR interval indicates a regular rhythm. (2) A normal P
wave before each complex indicates the impulse originated in the SA node. (3) The number of
complexes in a 6-second strip is multiplied by 10 to approximate the heart rate; normal sinus rhythm is
60 to 100. (4) Elevation of the ST segment is a sign of cardiac ischemia and is unrelated to the rhythm.
(5) The QRS duration should be less than 0.12 seconds; the PR interval should be 0.12 to 0.2 seconds.

This NCLEX exam has 80 questions that covers the diseases of the Gastrointestinal and Digestive
System.

Topics

Liver Failure

Liver Disorders

Ostomy Care

Guidelines

Read each question carefully and choose the best answer.

You are given one minute per question. Spend your time wisely!

Answers and rationales are given below. Be sure to read them.

If you need more clarifications, please direct them to the comments section.

Questions
1. A patient with chronic alcohol abuse is admitted with liver failure. You closely monitor
the patients blood pressure because of which change that is associated with the liver
failure?
1. Hypoalbuminemia
2. Increased capillary permeability
3. Abnormal peripheral vasodilation
4. Excess rennin release from the kidneys

33

2. Youre assessing the stoma of a patient with a healthy, well-healed colostomy. You
expect the stoma to appear:
1. Pale, pink and moist
2. Red and moist
3. Dark or purple colored
4. Dry and black
3. Youre caring for a patient with a sigmoid colostomy. The stool from this colostomy is:
1. Formed
2. Semisolid
3. Semiliquid
4. Watery
4. Youre advising a 21 y.o. with a colostomy who reports problems with flatus. What food
should you recommend?
1. Peas
2. Cabbage
3. Broccoli
4. Yogurt
5. You have to teach ostomy self care to a patient with a colostomy. You tell the patient to
measure and cut the wafer:
1. To the exact size of the stoma.
2. About 1/16 larger than the stoma.
3. About 1/8 larger than the stoma.
4. About 1/4 larger than the stoma.
6. Youre performing an abdominal assessment on Brent who is 52 y.o. In which order do
you proceed?
1. Observation, percussion, palpation, auscultation
2. Observation, auscultation, percussion, palpation
3. Percussion, palpation, auscultation, observation
4. Palpation, percussion, observation, auscultation
7. Youre doing preoperative teaching with Gertrude who has ulcerative colitis who needs
surgery to create an ileoanal reservoir. Which information do you include?
1. A reservoir is created that exits through the abdominal wall.
2. A second surgery is required 12 months after the first surgery.
3. A permanent ileostomy is created.
4. The surgery occurs in two stages.
8. Youre caring for Carin who has just had ileostomy surgery. During the first 24 hours
post-op, how much drainage can you expect from the ileostomy?
1. 100 ml
2. 500 ml
3. 1500 ml
4. 5000 ml
9. Youre preparing a teaching plan for a 27 y.o. named Jeff who underwent surgery to close
a temporary ileostomy. Which nutritional guideline do you include in this plan?
1. There is no need to change eating habits.
2. Eat six small meals a day.
3. Eat the largest meal in the evening.
4. Restrict fluid intake.

34

10. Arthur has a family history of colon cancer and is scheduled to have a sigmoidoscopy.
He is crying as he tells you, I know that I have colon cancer, too. Which response is most
therapeutic?
1. I know just how you feel.
2. You seem upset.
3. Oh, dont worry about it, everything will be just fine.
4. Why do you think you have cancer?
11. Youre caring for Beth who underwent a Billroth II procedure (surgical removal of the
pylorus and duodenum) for treatment of a peptic ulcer. Which findings suggest that the
patient is developing dumping syndrome, a complication associated with this procedure?
1. Flushed, dry skin.
2. Headache and bradycardia.
3. Dizziness and sweating.
4. Dyspnea and chest pain.
12. Youre developing the plan of care for a patient experiencing dumping syndrome after a
Billroth II procedure. Which dietary instructions do you include?
1. Omit fluids with meals.
2. Increase carbohydrate intake.
3. Decrease protein intake.
4. Decrease fat intake.
13. Youre caring for Lewis, a 67 y.o. patient with liver cirrhosis who developed ascites and
requires paracentesis. Relief of which symptom indicated that the paracentesis was
effective?
1. Pruritus
2. Dyspnea
3. Jaundice
4. Peripheral Neuropathy
14. Youre caring for Jane, a 57 y.o. patient with liver cirrhosis who developed ascites and
requires paracentesis. Before her paracentesis, you instruct her to:
1. Empty her bladder.
2. Lie supine in bed.
3. Remain NPO for 4 hours.
4. Clean her bowels with an enema.
15. After abdominal surgery, your patient has a severe coughing episode that causes
wound evisceration. In addition to calling the doctor, which intervention is most
appropriate?
1. Irrigate the wound & organs with Betadine.
2. Cover the wound with a saline soaked sterile dressing.
3. Apply a dry sterile dressing & binder.
4. Push the organs back & cover with moist sterile dressings.
16. Youre caring for Betty with liver cirrhosis. Which of the following assessment findings
leads you to suspect hepatic encephalopathy in her?
1. Asterixis
2. Chvosteks sign
3. Trousseaus sign
4. Hepatojugular reflex

35

17. You are developing a care plan on Sally, a 67 y.o. patient with hepatic encephalopathy.
Which of the following do you include?
1. Administering a lactulose enema as ordered.
2. Encouraging a protein-rich diet.
3. Administering sedatives, as necessary.
4. Encouraging ambulation at least four times a day.
18. You have a patient with achalasia (incomplete muscle relaxtion of the GI tract,
especially sphincter muscles). Which medications do you anticipate to administer?
1. Isosorbide dinitrate (Isordil)
2. Digoxin (Lanoxin)
3. Captopril (Capoten)
4. Propranolol (Inderal)
19. The student nurse is preparing a teaching care plan to help improve nutrition in a
patient with achalasia. You include which of the following:
1. Swallow foods while leaning forward.
2. Omit fluids at mealtimes.
3. Eat meals sitting upright.
4. Avoid soft and semi soft foods.
20. Britney, a 20 y.o. student is admitted with acute pancreatitis. Which laboratory findings
do you expect to be abnormal for this patient?
1. Serum creatinine and BUN
2. Alanine aminotransferase (ALT) and aspartate aminotransferase (AST)
3. Serum amylase and lipase
4. Cardiac enzymes
21. A patient with Crohns disease is admitted after 4 days of diarrhea. Which of the
following urine specific gravity values do you expect to find in this patient?
1. 1.005
2. 1.011
3. 1.020
4. 1.030
22. Your goal is to minimize Davids risk of complications after a herniorrhaphy. You
instruct the patient to:
1. Avoid the use of pain medication.
2. Cough and deep breathe Q2H.
3. Splint the incision if he cant avoid sneezing or coughing.
4. Apply heat to scrotal swelling.
23. Janice is waiting for discharge instructions after her herniorrhaphy. Which of the
following instructions do you include?
1. Eat a low-fiber diet.
2. Resume heavy lifting in 2 weeks.
3. Lose weight, if obese.
4. Resume sexual activity once discomfort is gone.
24. Develop a teaching care plan for Angie who is about to undergo a liver biopsy. Which of
the following points do you include?
1. Youll need to lie on your stomach during the test.
2. Youll need to lie on your right side after the test.

36

3. During the biopsy youll be asked to exhale deeply and hold it.
4. The biopsy is performed under general anesthesia.
25. Stephen is a 62 y.o. patient that has had a liver biopsy. Which of the following groups
of signs alert you to a possible pneumothorax?
1. Dyspnea and reduced or absent breath sounds over the right lung
2. Tachycardia, hypotension, and cool, clammy skin
3. Fever, rebound tenderness, and abdominal rigidity
4. Redness, warmth, and drainage at the biopsy site
26. Michael, a 42 y.o. man is admitted to the med-surg floor with a diagnosis of acute
pancreatitis. His BP is 136/76, pulse 96, Resps 22 and temp 101. His past history includes
hyperlipidemia and alcohol abuse. The doctor prescribes an NG tube. Before inserting the
tube, you explain the purpose to patient. Which of the following is a most accurate
explanation?
1. It empties the stomach of fluids and gas.
2. It prevents spasms at the sphincter of Oddi.
3. It prevents air from forming in the small intestine and large intestine.
4. It removes bile from the gallbladder.
27. Jason, a 22 y.o. accident victim, requires an NG tube for feeding. What should you
immediately do after inserting an NG tube for liquid enteral feedings?
1. Aspirate for gastric secretions with a syringe.
2. Begin feeding slowly to prevent cramping.
3. Get an X-ray of the tip of the tube within 24 hours.
4. Clamp off the tube until the feedings begin.
28. Stephanie, a 28 y.o. accident victim, requires TPN. The rationale for TPN is to provide:
1. Necessary fluids and electrolytes to the body.
2. Complete nutrition by the I.V. route.
3. Tube feedings for nutritional supplementation.
4. Dietary supplementation with liquid protein given between meals.
29. Type A chronic gastritis can be distinguished from type B by its ability to:
1. Cause atrophy of the parietal cells.
2. Affect only the antrum of the stomach.
3. Thin the lining of the stomach walls.
4. Decrease gastric secretions.
30. Matt is a 49 y.o. with a hiatal hernia that you are about to counsel. Health care
counseling for Matt should include which of the following instructions?
1. Restrict intake of high-carbohydrate foods.
2. Increase fluid intake with meals.
3. Increase fat intake.
4. Eat three regular meals a day.
31. Jerod is experiencing an acute episode of ulcerative colitis. Which is priority for this
patient?
1. Replace lost fluid and sodium.
2. Monitor for increased serum glucose level from steroid therapy.
3. Restrict the dietary intake of foods high in potassium.
4. Note any change in the color and consistency of stools.
32. A 29 y.o. patient has an acute episode of ulcerative colitis. What diagnostic test
confirms this diagnosis?

37

1. Barium Swallow.
2. Stool examination.
3. Gastric analysis.
4. Sigmoidoscopy.
33. Eleanor, a 62 y.o. woman with diverticulosis is your patient. Which interventions would
you expect to include in her care?
1. Low-fiber diet and fluid restrictions.
2. Total parenteral nutrition and bed rest.
3. High-fiber diet and administration of psyllium.
4. Administration of analgesics and antacids.
34. Regina is a 46 y.o. woman with ulcerative colitis. You expect her stools to look like:
1. Watery and frothy.
2. Bloody and mucous.
3. Firm and well-formed.
4. Alternating constipation and diarrhea.
35. Donald is a 61 y.o. man with diverticulitis. Diverticulitis is characterized by:
1. Periodic rectal hemorrhage.
2. Hypertension and tachycardia.
3. Vomiting and elevated temperature.
4. Crampy and lower left quadrant pain and low-grade fever.
36. Brenda, a 36 y.o. patient is on your floor with acute pancreatitis. Treatment for her
includes:
1. Continuous peritoneal lavage.
2. Regular diet with increased fat.
3. Nutritional support with TPN.
4. Insertion of a T tube to drain the pancreas.
37. Glenda has cholelithiasis (gallstones). You expect her to complain of:
1. Pain in the right upper quadrant, radiating to the shoulder.
2. Pain in the right lower quadrant, with rebound tenderness.
3. Pain in the left upper quadrant, with shortness of breath.
4. Pain in the left lower quadrant, with mild cramping.
38. After an abdominal resection for colon cancer, Madeline returns to her room with a
Jackson-Pratt drain in place. The purpose of the drain is to:
1. Irrigate the incision with a saline solution.
2. Prevent bacterial infection of the incision.
3. Measure the amount of fluid lost after surgery.
4. Prevent accumulation of drainage in the wound.
39. Anthony, a 60 y.o. patient, has just undergone a bowel resection with a colostomy.
During the first 24 hours, which of the following observations about the stoma should you
report to the doctor?
1. Pink color.
2. Light edema.
3. Small amount of oozing.
4. Trickles of bright red blood.
40. Your teaching Anthony how to use his new colostomy. How much skin should remain
exposed between the stoma and the ring of the appliance?

38

1. 1/16
2. 1/4
3. 1/2
4. 1
41. Claire, a 33 y.o. is on your floor with a possible bowel obstruction. Which intervention is
priority for her?
1. Obtain daily weights.
2. Measure abdominal girth.
3. Keep strict intake and output.
4. Encourage her to increase fluids.
42. Your patient has a GI tract that is functioning, but has the inability to swallow foods.
Which is the preferred method of feeding for your patient?
1. TPN
2. PPN
3. NG feeding
4. Oral liquid supplements
43. Youre patient is complaining of abdominal pain during assessment. What is your
priority?
1. Auscultate to determine changes in bowel sounds.
2. Observe the contour of the abdomen.
3. Palpate the abdomen for a mass.
4. Percuss the abdomen to determine if fluid is present.
44. Before bowel surgery, Lee is to administer enemas until clear. During administration,
he complains of intestinal cramps. What do you do next?
1. Discontinue the procedure.
2. Lower the height of the enema container.
3. Complete the procedure as quickly as possible.
4. Continue administration of the enema as ordered without making any adjustments.
45. Leigh Ann is receiving pancrelipase (Viokase) for chronic pancreatitis. Which
observation best indicates the treatment is effective?
1. There is no skin breakdown.
2. Her appetite improves.
3. She loses more than 10 lbs.
4. Stools are less fatty and decreased in frequency.
46. Ralph has a history of alcohol abuse and has acute pancreatitis. Which lab value is
most likely to be elevated?
1. Calcium
2. Glucose
3. Magnesium
4. Potassium
47. Anna is 45 y.o. and has a bleeding ulcer. Despite multiple blood transfusions, her HGB
is 7.5g/dl and HCT is 27%. Her doctor determines that surgical intervention is necessary
and she undergoes partial gastrectomy. Postoperative nursing care includes:
1. Giving pain medication Q6H.
2. Flushing the NG tube with sterile water.
3. Positioning her in high Fowlers position.
4. Keeping her NPO until the return of peristalsis.

39

48. Sitty, a 66 y.o. patient underwent a colostomy for ruptured diverticulum. She did well
during the surgery and returned to your med-surg floor in stable condition. You assess her
colostomy 2 days after surgery. Which finding do you report to the doctor?
1. Blanched stoma
2. Edematous stoma
3. Reddish-pink stoma
4. Brownish-black stoma
49. Sharon has cirrhosis of the liver and develops ascites. What intervention is necessary
to decrease the excessive accumulation of serous fluid in her peritoneal cavity?
1. Restrict fluids
2. Encourage ambulation
3. Increase sodium in the diet
4. Give antacids as prescribed
50. Katrina is diagnosed with lactose intolerance. To avoid complications with lack of
calcium in the diet, which food should be included in the diet?
1. Fruit
2. Whole grains
3. Milk and cheese products
4. Dark green, leafy vegetables
51. Nathaniel has severe pruritus due to having hepatitis B. What is the best intervention
for his comfort?
1. Give tepid baths.
2. Avoid lotions and creams.
3. Use hot water to increase vasodilation.
4. Use cold water to decrease the itching.
52. Rob is a 46 y.o. admitted to the hospital with a suspected diagnosis of Hepatitis B. Hes
jaundiced and reports weakness. Which intervention will you include in his care?
1. Regular exercise.
2. A low-protein diet.
3. Allow patient to select his meals.
4. Rest period after small, frequent meals.
53. Youre discharging Nathaniel with hepatitis B. Which statement suggests
understanding by the patient?
1. Now I can never get hepatitis again.
2. I can safely give blood after 3 months.
3. Ill never have a problem with my liver again, even if I drink alcohol.
4. My family knows that if I get tired and start vomiting, I may be getting sick again.
54. Gail is scheduled for a cholecystectomy. After completion of preoperative teaching, Gail
states,If I lie still and avoid turning after the operation, Ill avoid pain. Do you think this is
a good idea? What is the best response?
1. Youll need to turn from side to side every 2 hours.
2. Its always a good idea to rest quietly after surgery.
3. The doctor will probably order you to lie flat for 24 hours.
4. Why dont you decide about activity after you return from the recovery room?
55. Youre caring for a 28 y.o. woman with hepatitis B. Shes concerned about the duration
of her recovery. Which response isnt appropriate?

40

1. Encourage her to not worry about the future.


2. Encourage her to express her feelings about the illness.
3. Discuss the effects of hepatitis B on future health problems.
4. Provide avenues for financial counseling if she expresses the need.
56. Elmer is scheduled for a proctoscopy and has an I.V. The doctor wrote an order for 5mg
of I.V. diazepam(Valium). Which order is correct regarding diazepam?
1. Give diazepam in the I.V. port closest to the vein.
2. Mix diazepam with 50 ml of dextrose 5% in water and give over 15 minutes.
3. Give diazepam rapidly I.V. to prevent the bloodstream from diluting the drug mixture.
4. Question the order because I.V. administration of diazepam is contraindicated.
57. Annabelle is being discharged with a colostomy, and youre teaching her about
colostomy care. Which statement correctly describes a healthy stoma?
1. At first, the stoma may bleed slightly when touched.
2. The stoma should appear dark and have a bluish hue.
3. A burning sensation under the stoma faceplate is normal.
4. The stoma should remain swollen away from the abdomen.
58. A patient who underwent abdominal surgery now has a gaping incision due to delayed
wound healing. Which method is correct when you irrigate a gaping abdominal incision
with sterile normal saline solution, using a piston syringe?
1. Rapidly instill a stream of irrigating solution into the wound.
2. Apply a wet-to-dry dressing to the wound after the irrigation.
3. Moisten the area around the wound with normal saline solution after the irrigation.
4. Irrigate continuously until the solution becomes clear or all of the solution is used.
59. Hepatic encephalopathy develops when the blood level of which substance increases?
1. Ammonia
2. Amylase
3. Calcium
4. Potassium
60. Your patient recently had abdominal surgery and tells you that he feels a popping
sensation in his incision during a coughing spell, followed by severe pain. You anticipate an
evisceration. Which supplies should you take to his room?
1. A suture kit.
2. Sterile water and a suture kit.
3. Sterile water and sterile dressings.
4. Sterile saline solution and sterile dressings.
61. Findings during an endoscopic exam include a cobblestone appearance of the colon in
your patient. The findings are characteristic of which disorder?
1. Ulcer
2. Crohns disease
3. Chronic gastritis
4. Ulcerative colitis
62. What information is correct about stomach cancer?
1. Stomach pain is often a late symptom.
2. Surgery is often a successful treatment.
3. Chemotherapy and radiation are often successful treatments.
4. The patient can survive for an extended time with TPN.
63. Dark, tarry stools indicate bleeding in which location of the GI tract?

41

1. Upper colon.
2. Lower colon.
3. Upper GI tract.
4. Small intestine.
64. A patient has an acute upper GI hemorrhage. Your interventions include:
1. Treating hypovolemia.
2. Treating hypervolemia.
3. Controlling the bleeding source.
4. Treating shock and diagnosing the bleeding source.
65. You promote hemodynamic stability in a patient with upper GI bleeding by:
1. Encouraging oral fluid intake.
2. Monitoring central venous pressure.
3. Monitoring laboratory test results and vital signs.
4. Giving blood, electrolyte and fluid replacement.
66. Youre preparing a patient with a malignant tumor for colorectal surgery and
subsequent colostomy. The patient tells you hes anxious. What should your initial step be
in working with this patient?
1. Determine what the patient already knows about colostomies.
2. Show the patient some pictures of colostomies.
3. Arrange for someone who has a colostomy to visit the patient.
4. Provide the patient with written material about colostomy care.
67. Your patient, Christopher, has a diagnosis of ulcerative colitis and has severe
abdominal pain aggravated by movement, rebound tenderness, fever, nausea, and
decreased urine output. This may indicate which complication?
1. Fistula.
2. Bowel perforation.
3. Bowel obstruction.
4. Abscess.
68. A patient has a severe exacerbation of ulcerative colitis. Long-term medications will
probably include:
1. Antacids.
2. Antibiotics.
3. Corticosteroids.
4. Histamine2-receptor blockers.
69. The student nurse is teaching the family of a patient with liver failure. You instruct
them to limit which foods in the patients diet?
1. Meats and beans.
2. Butter and gravies.
3. Potatoes and pastas.
4. Cakes and pastries.
70. An intubated patient is receiving continuous enteral feedings through a Salem sump
tube at a rate of 60ml/hr. Gastric residuals have been 30-40ml when monitored Q4H. You
check the gastric residual and aspirate 220ml. What is your first response to this finding?
1. Notify the doctor immediately.
2. Stop the feeding, and clamp the NG tube.
3. Discard the 220ml, and clamp the NG tube.
4. Give a prescribed GI stimulant such as metoclopramide (Reglan).

42

71. Your patient with peritonitis is NPO and complaining of thirst. What is your priority?
1. Increase the I.V. infusion rate.
2. Use diversion activities.
3. Provide frequent mouth care.
4. Give ice chips every 15 minutes.
72. Kevin has a history of peptic ulcer disease and vomits coffee-ground emesis. What does
this indicate?
1. He has fresh, active upper GI bleeding.
2. He needs immediate saline gastric lavage.
3. His gastric bleeding occurred 2 hours earlier.
4. He needs a transfusion of packed RBCs.
73. A 53 y.o. patient has undergone a partial gastrectomy for adenocarcinoma of the
stomach. An NG tube is in place and is connected to low continuous suction. During the
immediate postoperative period, you expect the gastric secretions to be which color?
1. Brown.
2. Clear.
3. Red.
4. Yellow.
74. Your patient has a retractable gastric peptic ulcer and has had a gastric vagotomy.
Which factor increases as a result of vagotomy?
1. Peristalsis.
2. Gastric acidity.
3. Gastric motility.
4. Gastric pH.
75. Christina is receiving an enteral feeding that requires a concentration of 80 ml of
supplement mixed with 20 ml of water. How much water do you mix with an 8 oz (240ml)
can of feeding?
1. 60 ml.
2. 70 ml.
3. 80 ml.
4. 90 ml.
76. Which stoma would you expect a malodorous, enzyme-rich, caustic liquid output that is
yellow, green, or brown?
1. Ileostomy.
2. Ascending colostomy.
3. Transverse colostomy.
4. Descending colostomy.
77. George has a T tube in place after gallbladder surgery. Before discharge, what
information or instructions should be given regarding the T tube drainage?
1. If there is any drainage, notify the surgeon immediately.
2. The drainage will decrease daily until the bile duct heals.
3. First, the drainage is dark green; then it becomes dark yellow.
4. If the drainage stops, milk the tube toward the puncture wound.
78. Your patient Maria takes NSAIDS for her degenerative joint disease, has developed
peptic ulcer disease. Which drug is useful in preventing NSAID-induced peptic ulcer
disease?

43

1. Calcium carbonate (Tums)


2. Famotidine (Pepcid)
3. Misoprostol (Cytotec)
4. Sucralfate (Carafate)
79. The student nurse is participating in colorectal cancer-screening program. Which
patient has the fewest risk factors for colon cancer?
1. Janice, a 45 y.o. with a 25-year history of ulcerative colitis
2. George, a 50 y.o. whose father died of colon cancer
3. Herman, a 60 y.o. who follows a low-fat, high-fiber diet
4. Sissy, a 72 y.o. with a history of breast cancer
80. Youre patient, post-op drainage of a pelvic abscess secondary to diverticulitis, begins
to cough violently after drinking water. His wound has ruptured and a small segment of the
bowel is protruding. Whats your priority?
1. Ask the patient what happened, call the doctor, and cover the area with a water-soaked bedsheet.
2. Obtain vital signs, call the doctor, and obtain emergency orders.
3. Have a CAN hold the wound together while you obtain vital signs, call the doctor and flex the
patients knees.
4. Have the doctor called while you remain with the patient, flex the patients knees, and cover the
wound with sterile towels soaked in sterile saline solution.

Answers and Rationale


1. Answer: 1. Blood pressure decreases as the body is unable to maintain normal oncotic pressure with
liver failure, so patients with liver failure require close blood pressure monitoring. Increased capillary
permeability, abnormal peripheral vasodilation, and excess rennin released from the kidneys arent
direct ramifications of liver failure.
2. Answer: 2. Good circulation causes tissues to be moist and red, so a healthy, well-healed stoma
appears red and moist.
3. Answer: 1. A colostomy in the sigmoid colon produces a solid, formed stool.
4. Answer: 4. High-fiber foods stimulate peristalsis, and a result, flatus. Yogurt reduces gas formation.
5. Answer: 2. A proper fit protects the skin, but doesnt impair circulation. A 1/16 should be cut.
6. Answer: 2. Observation, auscultation, percussion, palpation
7. Answer: 4. An ileoanal reservoir is created in two stages. The two surgeries are about 2 to 3 months
apart. First, diseased intestines are removed and a temporary loop ileostomy is created. Second, the
loop ileostomy is closed and stool goes to the reservoir and out through the anus.
8. Answer: 3. The large intestine absorbs large amounts of water so the initial output from the
ileostomy may be as much as 1500 to 2000 ml/24 hours. Gradually, the small intestine absorbs more
fluid and the output decreases.
9. Answer: 2. To avoid overloading the small intestine, encourage the patient to eat six small, regularly
spaced meals.
10. Answer: 2. Making observations about what you see or hear is a useful therapeutic technique. This
way, you acknowledge that you are interested in what the patient is saying and feeling.
11. C After a Billroth II procedure, a large amount of hypertonic fluid enters the intestine. This causes
extracellular fluid to move rapidly into the bowel, reducing circulating blood volume and producing
vasomotor symptoms. Vasomotor symptoms produced by dumping syndrome include dizziness and
sweating, tachycardia, syncope, pallor, and palpitations.
12. A Gastric emptying time can be delayed by omitting fluids from your patients meal. A diet low in
carbs and high in fat & protein is recommended to treat dumping syndrome.

44

13. B Ascites puts pressure on the diaphragm. Paracentesis is done to remove fluid and reducing
pressure on the diaphragm. The goal is to improve the patients breathing. The others are signs of
cirrhosis that arent relieved by paracentesis.
14. A A full bladder can interfere with paracentesis and be punctured inadvertently.
15. B Cover the organs with a sterile, nonadherent dressing moistened with normal saline. Do this to
prevent infection and to keep the organs from drying out.
16. A Asterixis is an early neurologic sign of hepatic encephalopathy elicited by asking the patient to
hold her arms stretched out. Asterixis is present if the hands rapidly extend and flex.
17. A You may administer the laxative lactulose to reduce ammonia levels in the colon.
18. A Achalasia is characterized by incomplete relaxation of the LES, dilation of the lower esophagus,
and a lack of esophageal peristalsis. Because nitrates relax the lower esophageal sphincter, expect to
give Isordil orally or sublingually.
19. C Eating in the upright position aids in emptying the esophagus. Doing the opposite of the other
three also may be helpful.
20. C Pancreatitis involves activation of pancreatic enzymes, such as amylase and lipase. These levels
are elevated in a patient with acute pancreatitis.
21. D The normal range of specific gravity of urine is 1.010 to 1.025; a value of 1.030 may be seen
with dehydration.
22. C Teach the pt to avoid activities that increase intra-abdominal pressure such as coughing,
sneezing, or straining with a bowel movement.
23. C Because obesity weakens the abdominal muscles, advise weight loss for the patient who has had
a hernia repair.
24. B After a liver biopsy, the patient is placed on the right side to compress the liver and to reduce the
risk of bleeding or bile leakage.
25. A Signs and Symptoms of pneumothorax include dyspnea and decreased or absent breath sounds
over the affected lung (right lung).
26. A An NG tube is inserted into the patients stomach to drain fluid and gas.
27. A Aspirating the stomach contents confirms correct placement. If an X-ray is ordered, it should be
done immediately, not in 24 hours.
28. B TPN is given I.V. to provide all the nutrients your patient needs. TPN isnt a tube feeding nor is it a
liquid dietary supplement.
29. A Type A causes changes in parietal cells.
30. B Increasing fluids helps empty the stomach. A high carb diet isnt restricted and fat intake
shouldnt be increased.
31. A Diarrhea d/t an acute episode of ulcerative colitis leads to fluid & electrolyte losses so fluid
replacement takes priority.
32. D Sigmoidoscopy allows direct observation of the colon mucosa for changes, and if needed, biopsy.
33. C She needs a high-fiber diet and a psyllium (bulk laxative) to promote normal soft stools.
34. B Stools from ulcerative colitis are often bloody and contain mucus.
35. D One sign of acute diverticulitis is crampy lower left quadrant pain. A low-grade fever is another
common sign.
36. C With acute pancreatitis, you need to rest the GI tract by TPN as nutritional support.
37. A The gallbladder is located in the RUQ and a frequent sign of gallstones is pain radiating to the
shoulder.
38. D A Jackson-Pratt drain promotes wound healing by allowing fluid to escape from the wound.

45

39. D After creation of a colostomy, expect to see a stoma that is pink, slightly edematous, with some
oozing. Bright red blood, regardless of amount, indicates bleeding and should be reported to the
doctor.
40. A Only a small amount of skin should be exposed and more than 1/16 of skin allows the
excretement to irritate the skin.
41. B Measuring abdominal girth provides quantitative information about increases or decreases in the
amount of distention.
42. C Because the GI tract is functioning, feeding methods involve the enteral route which bypasses
the mouth but allows for a major portion of the GI tract to be used.
43. B The first step in assessing the abdomen is to observe its shape and contour, then auscultate,
palpate, and then percuss.
44. B Lowering the height decreases the amount of flow, allowing him to tolerate more fluid.
45. D Pancrelipase provides the exocrine pancreatic enzyme necessary for proper protein, fat, and carb
digestion. With increased fat digestion and absorption, stools become less frequent and normal in
appearance.
46. B Glucose level increases and diabetes mellitus may result d/t the pancreatic damage to the islets
of langerhans.
47. D After surgery, she remains NPO until peristaltic activity returns. This decreases the risk for
abdominal distention and obstruction.
48. D A brownish-black color indicates lack of blood flow, and maybe necrosis.
49. A Restricting fluids decrease the amount of body fluid and the accumulation of fluid in the
peritoneal space.
50. D Dark green, leafy vegetables are rich in calcium.
51. A For pruritus, care should include tepid sponge baths and use of emollient creams and lotions.
52. D Rest periods and small frequent meals is indicated during the acute phase of hepatitis B.
53. D Hepatitis B can recur. Patients who have had hepatitis are permanently barred from donating
blood. Alcohol is metabolized by the liver and should be avoided by those who have or had hepatitis B.
54. A To prevent venous stasis and improve muscle tone, circulation, and respiratory function,
encourage her to move after surgery.
55. A Telling her not to worry minimizes her feelings.
56. A Diazepam is absorbed by the plastic I.V. tubing and should be given in the port closest to the
vein.
57. A For the first few days to a week, slight bleeding normally occurs when the stoma is touched
because the surgical site is still new. She should report profuse bleeding immediately.
58. D To wash away tissue debris and drainage effectively, irrigate the wound until the solution
becomes clear or all the solution is used.
59. A Ammonia levels increase d/t improper shunting of blood, causing ammonia to enter systemic
circulation, which carries it to the brain.
60. D Saline solution is isotonic, or close to body fluids in content, and is used along with sterile
dressings to cover an eviscerated wound and keep it moist.
61. B Crohns disease penetrates the mucosa of the colon through all layers and destroys the colon in
patches, which creates a cobblestone appearance.
62. A Stomach pain is often a late sign of stomach cancer; outcomes are particularly poor when the
cancer reaches that point. Surgery, chemotherapy, and radiation have minimal positive effects. TPN
may enhance the growth of the cancer.
63. C Melena is the passage of dark, tarry stools that contain a large amount of digested blood. It
occurs with bleeding from the upper GI tract.

46

64. A A patient with an acute upper GI hemorrhage must be treated for hypovolemia and hemorrhagic
shock. You as a nurse cant diagnose the problem. Controlling the bleeding may require surgery or
intensive medical treatment.
65. D To stabilize a patient with acute bleeding, NS or LR solution is given I.V. until BP rises and urine
output returns to 30ml/hr.
66. A Initially, you should assess the patients knowledge about colostomies and how it will affect his
lifestyle.
67. B An inflammatory condition that affects the surface of the colon, ulcerative colitis causes friability
and erosions with bleeding. Patients with ulcerative colitis are at increased risk for bowel perforation,
toxic megacolon, hemorrhage, cancer, and other anorectal and systemic complications.
68. C Medications to control inflammation such as corticosteroids are used for long-term treatment.
69. A Meats and beans are high-protein foods. In liver failure, the liver is unable to metabolize protein
adequately, causing protein by-products to build up in the body rather than be excreted.
70. B A gastric residual greater than 2 hours worth of feeding or 100-150ml is considered too high. The
feeding should be stopped; NG tube clamped, and then allow time for the stomach to empty before
additional feeding is added.
71. C Frequent mouth care helps relieve dry mouth.
72. C Coffee-ground emesis occurs when there is upper GI bleeding that has undergone gastric
digestion. For blood to appear as coffee-ground emesis, it would have to be digested for approximately
2 hours.
73. C Normally, drainage is bloody for the first 24 hours after a partial gastrectomy; then it changes to
brown-tinged and then to yellow or clear.
74. D If the vagus nerve is cut as it enters the stomach, gastric acid secretion is decreased, but
intestinal motility is also decreased and gastric emptying is delayed. Because gastric acids are
decreased, gastric pH increases.
75. A Dosage problem. Its 80/20 = 240/X. X=60.
76. A The output from an Ileostomy is described.
77. B As healing occurs from the bile duct, bile drains from the tube; the amount of bile should
decrease. Teach the patient to expect dark green drainage and to notify the doctor if drainage stops.
78. C Misoprostol restores prostaglandins that protect the stomach from NSAIDS, which diminish the
prostaglandins.
79. C
80. D

Introduction
Questions about Substance Abuse, Alcoholism, Therapeutic Communication and more are included
in the 25-item NCLEXsample exam.

Topics

Substance Abuse

Therapeutic Communication

Alcohol Abuse and Withdrawal

Guidelines

Read each question carefully and choose the best answer.

47

You are given one minute per question. Spend your time wisely!

Answers and rationales are given below. Be sure to read them.

If you need more clarifications, please direct them to the comments section.

Questions
1. The nurse is planning activities for a client who has bipolar disorder with aggressive
social behavior. Which of the following activities would be most appropriate for this client?
1. Ping pong
2. Writing
3. Chess
4. Basketball
2. A client is admitted to the hospital with a diagnosis of major depression, severe, single
episode. The nurse assesses the client and identifies a nursing diagnosis of imbalanced
nutrition related to poor nutritional intake. The most appropriate nursing intervention
related to this diagnosis is:
1. Explain to the client the importance of a good nutritional intake
2. Weight the client 3 times per week before breakfast
3. Report the nutritional concern to the psychiatrist and obtain a nutritional consultation as soon as
possible.
4. Consult with the nutritionist, offer the client several small meals per day, and schedule brief nursing
interactions with the client during these times.
3. In planning activities for the depressed client, especially during the early stages of
hospitalization, which of the following plans is best?
1. Provide an activity that is quiet and solitary to avoid increased fatigue, such as working on a puzzle
or reading a book.
2. Plan nothing until the client asks to participate in milieu.
3. Offer the client a menu of daily activities and insist the client participate in all of them
4. Provide a structured daily program of activities and encourage the client to participate.
4. The depressed client verbalizes feelings of low self-esteem and self-worth typified by
statements such as Im such a failure I cant do anything right! The best nursing
response would be:
1. To tell the client this is not true; that we all have a purpose in life.
2. To remain with the client and sit in silence; this will encourage the client to verbalize feelings
3. To reassure the client that you know how the client is feeling and that things will get better
4. To identify recent behaviors or accomplishments that demonstrates skill ability.
5. A client with a diagnosis of major depression, recurrent with psychotic features is
admitted to the mental health unit. To create a safe environment for the client, the nurse
most importantly devises a plan of care that deals specifically with the clients:
1. Disturbed thought processes
2. Imbalanced nutrition
3. Self-care deficit
4. Deficient knowledge
6. A depressed client is ready for discharge. The nurse feels comfortable that the client has
a good understanding of the disease process when the client states:
1. Ill never let this happen to me again. I wont let my boss or my job or my family get to me!
2. Its important for me to eat well, exercise, and to take my medication. If I begin to lose my appetite
or not sleep well, Ive got to get in to see my doctor.

48

3. Ive learned that Im a good person and that I am worthy of giving and receiving love. I dont need
anyone; I have myself to rely on!
4. I dont know what happened to me. Ive always been able to make decisions for myself and for my
business. I dont ever want to feel so weak or vulnerable again!
7. The nurse assesses a client with the admitting diagnosis of bipolar affective disorder,
mania. The symptom presented by the client that requires the nurses immediate
intervention is the clients:
1. Outlandish behaviors and inappropriate dress
2. Grandiose delusions of being a royal descendent of King Arthur.
3. Nonstop physical activity and poor nutritional intake
4. Constant, incessant talking that includes sexual innuendoes and teasing the staff
8. The nurse reviews the activity schedule for the day and plans which activity for the
manic client?
1. Brown-bag luncheon and book review
2. Tetherball
3. Paint-by-number activity
4. Deep breathing and progressive relaxation group
9. A hospitalized client is being considered for ECT. The client appears calm, but the family
is anxious. The clients mother begins to cry and states My sons brain will be destroyed.
How can the doctor do this to him? The nurses best response is:
1. It sounds as though you need to speak with the psychiatrist
2. Your son has decided to have this treatment. You should be supportive to him.
3. Perhaps youd like to see the ECT room and speak to the staff.
4. It sounds as though you have some concerns about the ECT procedure. Why dont we sit down
together and discuss any concerns you may have.
10. The manic client announces to everyone in the dayroom that a stripper is coming to
perform this evening. When the nurse firmly states that this will not happen, the manic
client becomes verbally abusive and threatens physical violence to the nurse. Based on the
analysis of this situation, the nurse determines that the most appropriate action would be
to:
1. With assistance, escort the manic client to her room and administer Haldol as prescribed if needed
2. Tell the client that smoking privileges are revoked for 24 hours
3. Orient the client to time, person, and place
4. Tell the client that the behavior is not appropriate.
11. Select all nursing interventions for a hospitalized client with mania who is exhibiting
manipulative behavior.
1. Communicate expected behaviors to the client
2. Enforce rules and inform the client the he or she will not be allowed to attend group therapy
sessions.
3. Ensure that the client knows that he or she is not in charge of the nursing unit
4. Be clear with the client regarding the consequences of exceeding limits set regarding behavior.
5. Assist the client in testing out alternative behaviors for obtaining needs
12. A woman comes into the ER in a severe state of anxiety following a car accident. The
most appropriate nursing intervention is to:
1. Remain with the client
2. Put the client in a quiet room

49

3. Teach the client deep breathing


4. Encourage the client to talk about their feelings and concern.
13. When planning the discharge of a client with chronic anxiety, the nurse directs the
goals at promoting a safe environment at home. The most appropriate maintenance goal
should focus on which of the following?
1. Continued contact with a crisis counselor
2. Identifying anxiety-producing situations
3. Ignoring feelings of anxiety
4. Eliminating all anxiety from daily situations
14. The nurse is monitoring a client who abuses alcohol for signs of alcohol withdrawal.
Which of the following would alert the nurse to the potential for delirium tremors?
1. Hypertension, changes in LOC, hallucinations
2. Hypotension, ataxia, hunger
3. Stupor, agitation, muscular rigidity
4. Hypotension, coarse hand tremors, agitation
15. The spouse of a client admitted to the mental health unit for alcohol withdrawal says to
the nurse I should get out of this bad situation. The most helpful response by the nurse
would be:
1. I agree with you. You should get out of this situation.
2. What do you find difficult about this situation?
3. Why dont you tell your husband about this?
4. This is not the best time to make that decision.
16. The nurse determines that the wife of an alcoholic client is benefiting from attending
Al-Anon group when she hears the wife say:
1. My attendance at the meetings has helped me to see that I provoke my husbands violence.
2. I no longer feel that I deserve the beatings my husband inflicts on me.
3. I can tolerate my husbands destructive behavior now that I know they are common with
alcoholics.
4. I enjoy attending the meetings because they get me out of the house and away from my husband.
17. The client has been hospitalized and is participating in a substance abuse therapy
group sessions. On discharge, the client has consented to participate in AA community
groups. The nurse is monitoring the clients response to the substance abuse sessions.
Which statement by the client best indicates that the client has developed effective coping
response styles and has processed information effectively for self use?
1. I know Im ready to be discharged. I feel I can say no and leave a group of friends if they are
drinking No Problem.
2. This group has really helped a lot. I know it will be different when I go home. But Im sure that my
family and friends will all help me like the people in this group have Theyll all help me I know they
will They wont let me go back to my old ways.
3. Im looking forward to leaving here. I know that I will miss all of you. So, Im happy and Im sad, Im
excited and Im scared. I know that I have to work hard to be strong and that everyone isnt going to
be as helpful as you people.
4. Ill keep all my appointments; go to all my AA groups; Ill do everything Im supposed to Nothing
will go wrong that way.
18. A hospitalized client with a history of alcohol abuse tells the nurse, I am leaving now. I
have to go. I dont want anymore treatment. I have things that I have to do right away.
The client has not been discharged. In fact, the client is scheduled for an important

50

diagnostic test to be performed in 1 hour. After the nurse discusses the clients concerns
with the client, the client dresses and begins to walk out of the hospital room. The most
important nursing action is to:
1. Restrain the client until the physician can be reached
2. Call security to block all areas
3. Tell the client that the client cannot return to this hospital again if the client leaves now.
4. Call the nursing supervisor.
19. Select the appropriate interventions for caring for the client in alcohol withdrawal.
1. Monitor vital signs
2. Provide stimulation in the environment
3. Maintain NPO status
4. Provide reality orientation as appropriate
5. Address hallucinations therapeutically
20. Which of the following nursing actions would be included in a care plan for a client with
PTSD who states the experience was bad luck?
1. Encourage the client to verbalize the experience
2. Assist the client in defining the experience
3. Work with the client to take steps to move on with his life
4. Help the client accept positive and negative feelings
21. Which of the following psychological symptoms would the nurse expect to find in a
hospitalized client who is the only survivor of a train accident?
1. Denial
2. Indifference
3. Perfectionism
4. Trust
22. Which of the following communication guidelines should the nurse use when talking
with a client experiencing mania?
1. Address the client in a light and joking manner
2. Focus and redirect the conversation as necessary
3. Allow the client to talk about several different topic
4. Ask only open ended questions to facilitate conversations
23. What information is important to include in the nutritional counseling of a family with a
member who has bipolar disorder?
1. If sufficient roughage isnt eaten while taking lithium, bowel problems will occur.
2. If the intake of carbohydrates increases, the lithium level increases.
3. If the intake of calories is reduced, the lithium level will increase
4. If the intake of sodium increases, the lithium level will decrease.
24. In conferring with the treatment team, the nurse should make which of the following
recommendations for a client who tells the nurse that everyday thoughts of suicide are
present?
1. A no-suicide contract
2. Weekly outpatient therapy
3. A second psychiatric opinion
4. Intensive inpatient treatment
25. Which of the following short term goals is most appropriate for a client with bipolar
disorder who is having difficulty sleeping?

51

1. Obtain medication for sleep


2. Work on solving a problem
3. Exercise before bedtime
4. Develop a sleep ritual

Answers and Rationale


1. Answer: B. Solitary activities that require a short attention span with mild physical exertion are the
most appropriate activities for a client who is exhibiting aggressive behavior. Writing, walks with staff,
and finger painting are activities that minimize stimuli and provide a constructive release for tension.
Competitive games can stimulate aggression and increase psychomotor activity.
2. Answer: D. Change in appetite is one of the major symptoms of depression. Reporting to the
psychiatrist and nutritionist is to some degree correct but lacks the method as to how one would
increase food intake.
3. Answer: D. A depressed person experiences a depressed mood and is often withdrawn. The person
also experiences difficulty concentrating, loss of interest or pleasure, low energy, fatigue, and feelings
of worthlessness and poor self-esteem. The plan of care needs to provide successful experiences in a
stimulating yet structured environment. Option 3 is a forceful and absolute approach.
4. Answer: D. Feelings of low self-esteem and worthlessness are common symptoms of the depressed
client. An effective plan of care to enhance the clients personal self-esteem is to provide experiences
for the client that are challenging but will not be met with failure. Reminders of the clients past
accomplishments or personal successes are ways to interrupt the clients negative self talk and
distorted cognitive view of self. Silence may be interpreted as agreement. Options 1 and 3 give advice
and devalue the clients feelings.
5. Answer: A. major depression, recurrent, with psychotic features alerts the nurse that in addition to
the criteria that designate the diagnosis of major depression, one also must deal with the clients
psychosis. Psychosis is defined as a state in which a persons mental capacity to recognize reality and
to communicate and relate to others is impaired, thus interfering with the persons capacity to deal
with the demands of life. Altered thought processes generally indicate a state of increased anxiety in
which hallucinations and delusions prevail. Although all of the nursing diagnoses may be appropriate
because the client is experiencing psychosis, option 1 is correct.
6. Answer: B. The exact cause of depression is not known but is believed to be related to biochemical
disruption of neurotransmitters in the brain. Diet, exercise, and medication are recognized treatment
for the disease process.
7. Answer: C. Mania is a mood characterized by excitement, euphoria, hyperactivity, excessive
energy, decreased need for sleep, and impaired ability to concentrate or complete a single train of
thought. Mania is a period when the mood is predominately elevated, expansive, or irritable. All
options reflect a clients possible symptomatology. Option 3, however, clearly presents a problem that
compromises ones physiological integrity and needs to be addressed immediately.
8. Answer: B. A person who is experiencing mania is overactive and full of energy, lacks
concentration, and has poor impulse control. The client needs an activity that will allow use of excess
energy yet not endanger others during the process. Options 1, 3, and 4 are relatively sedate activities
that require concentration, a quality that is lacking in the manic state. Such activities lead to increased
frustration and anxiety for the client. Tetherball is an exercise that uses the large muscle groups of the
body and is a great way to expend the increased energy that the client is experiencing.
9. Answer: D. The nurse encourages the client and the family to verbalize fears and concerns. The
other options avoid dealing with concerns and are blocks to communication.

52

10. Answer: A. The client is at risk for injury to self and others and therefore should be escorted out
of the dayroom. Antipsychotic medications are useful to manage the manic client. Hyperactive and
agitated behavior usually responds to Haldol. Option 2 may increase the agitation that already exists in
this client. Orientation will not halt the behavior. Telling the client that the behavior is not appropriate
already has been attempted by the nurse.
11. Answers: A, D, and E. Interventions for dealing with the client exhibiting manipulative behavior
include setting clear, consistent, and enforceable limits on manipulative behaviors; being clear with
the client regarding the consequences of exceeding limits set; following through with the
consequences in a non-punishment manner; and assisting the client in identifying strengths and in
testing out alternative behaviors for obtaining needs. Enforcing rules and informing the client that he
or she will not be allowed to attend group therapy sessions is a violation of the clients rights. Ensuring
the client knows that he or she is not in charge of the nursing unit is inappropriate, power struggles
need to be avoided.
12. Answer: A. If a client with severe anxiety is left alone; the client may feel abandoned and become
overwhelmed. Placing the client in a quiet room is also important, but the nurse must stay with the
client. Teaching the client deep breathing or relaxation is not possible until the anxiety decreases.
Encouraging the client to discuss concerns and feelings would not take place until the anxiety has
decreased.
13. Answer: B. Recognizing situations that produce anxiety allows the client to prepare to cope with
anxiety or avoid a specific stimulus. Counselors will not be available for all anxiety-producing
situations, and this option does not encourage the development of internal strengths. Ignoring feelings
will not resolve anxiety. Elimination anxiety from life is impossible.
14. Answer: A. Some of the symptoms associated with delirium tremors typically are anxiety,
insomnia, anorexia, hypertension, disorientation, hallucinations, and changes in LOC, agitation, fever,
and delusions.
15. Answer: B. The most helpful response is one that encourages the client to problem solve. Giving
advice implies that the nurse knows what is best and can foster dependency. The nurse should not
agree with the client, nor should the nurse request that the client provide explanations.
16. Answer: B. Al-Anon support groups are protected, supportive opportunity for spouses and
significant others to learn what to expect and to obtain excellent pointers about successful behavior
changes. Option 2 is the most healthy response because is exemplifies and understanding that the
alcoholic partner is responsible for his behavior and cannot be allowed to blame family members for
loss of control.
17. Answer: C. In the defense mechanism of denial the person denies reality. Option 1 identifies
denial. In option 2 the client is relying heavily on others, and the clients focus of control is external. In
option 4 the client is concrete and procedure oriented; again the client identifies that Nothing will go
wrong that way if the client follows all the directions. In option 3 the client is expressing real concern
and ambivalence about discharge from the hospital. The client also demonstrates reality in that
statement.
18. Answer: D. A nurse can be charged with false imprisonment if a client is made to believe
wrongfully that the client cannot leave the hospital. Most health care facilities have documents that
the client is asked to sign that relate to the clients responsibilities when the client leaves against
medical advice. The client should be asked to sign this document before leaving. The nurse should
request that the client wait to speak to the physician before leaving, but if the client refuses to do so,
the nurse cannot hold him against his will. Restraining the client and calling security to block exits
constitutes false imprisonment. Any client has a right to health care and cannot be told otherwise.

53

19. Answers: A, D, and E. When the client is experiencing withdrawal of alcohol, the priority of care
is to prevent the client from harming himself or others. The nurse would provide a low stimulating
environment to maintain the client in as calm a state as possible. The nurse would monitor vital signs
closely and report abnormal findings. The nurse would reorient the client to reality frequently and
would address hallucinations therapeutically. Adequate nutritional and fluid intake needs to be
maintained.
20. Answer: B. The client must define the experience as traumatic to realize the situation wasnt
under his personal control. Encouraging the client to verbalize the experience without first addressing
the denial isnt a useful strategy. The client can move on with life only after acknowledging the trauma
and processing the experience. Acknowledgement of the actual trauma and verbalization of the event
should come before the acceptance of feelings.
21. Answer: A. Denial can act as a protective response. The client tends to be overwhelmed and
disorganized by the trauma, not indifferent to it. Perfectionism is more commonly seen in clients with
eating disorders, not in clients with PTSD. Clients who have had a severe trauma often experience an
inability to trust others.
22. Answer: B. To decrease stimulation, the nurse should attempt to redirect and focus the clients
communication, not allow the client to talk about different topics. By addressing the client in a light
and joking manner, the conversation may contribute to the clients feeling out of control. For a manic
client, its best to ask closed questions because open-minded questions may enable the client to talk
endlessly, again possibly contributing to the clients feeling out of control.
23. Answer: D. Any time the level of sodium increases, such as with a change in the dietary intake,
the levels of lithium will decrease.
24. Answer: D. For a client thinking about suicide on a daily basis, inpatient care would be the best
intervention. Although a no-suicide contract is an important strategy, this client needs additional care.
The client needs a more intensive level of care than weekly outpatient therapy. Immediate intervention
is paramount, not a second psychiatric opinion.
25. Answer: D. A sleep ritual or nighttime routine helps the client to relax and prepare for sleep.
Obtaining sleep medication is a temporary solution. Working on problem solving may excite the client
rather than tire him. Exercise before retiring is inappropriate.

Introduction
This is a 45-item examination that can help you in your NCLEX test. Questions here include topics like
Renal Failure, Dialysis and more.

Topics

Renal Failure

Dialysis

Guidelines

Read each question carefully and choose the best answer.

You are given one minute per question. Spend your time wisely!

Answers and rationales are given below. Be sure to read them.

If you need more clarifications, please direct them to the comments section.

Questions
54

1. Dialysis allows for the exchange of particles across a semipermeable membrane by


which of the following actions?
1. Osmosis and diffusion
2. Passage of fluid toward a solution with a lower solute concentration
3. Allowing the passage of blood cells and protein molecules through it.
4. Passage of solute particles toward a solution with a higher concentration.
2. A client is diagnosed with chronic renal failure and told she must start hemodialysis.
Client teaching would include which of the following instructions?
1. Follow a high potassium diet
2. Strictly follow the hemodialysis schedule
3. There will be a few changes in your lifestyle.
4. Use alcohol on the skin and clean it due to integumentary changes.
3. A client is undergoing peritoneal dialysis. The dialysate dwell time is completed, and the
dwell clamp is opened to allow the dialysate to drain. The nurse notes that the drainage
has stopped and only 500 ml has drained; the amount the dialysate instilled was 1,500 ml.
Which of the following interventions would be done first?
1. Change the clients position.
2. Call the physician.
3. Check the catheter for kinks or obstruction.
4. Clamp the catheter and instill more dialysate at the next exchange time.
4. A client receiving hemodialysis treatment arrives at the hospital with a blood pressure
of 200/100, a heart rate of 110, and a respiratory rate of 36. Oxygen saturation on room air
is 89%. He complains of shortness of breath, and +2 pedal edema is noted. His last
hemodialysis treatment was yesterday. Which of the following interventions should be
done first?
1. Administer oxygen
2. Elevate the foot of the bed
3. Restrict the clients fluids
4. Prepare the client for hemodialysis.
5. A client has a history of chronic renal failure and received hemodialysis treatments three
times per week through an arteriovenous (AV) fistula in the left arm. Which of the following
interventions is included in this clients plan of care?
1. Keep the AV fistula site dry.
2. Keep the AV fistula wrapped in gauze.
3. Take the blood pressure in the left arm
4. Assess the AV fistula for a bruit and thrill
6. Which of the following factors causes the nausea associated with renal failure?
1. Oliguria
2. Gastric ulcers
3. Electrolyte imbalances
4. Accumulation of waste products
7. Which of the following clients is at greatest risk for developing acute renal failure?
1. A dialysis client who gets influenza
2. A teenager who has an appendectomy
3. A pregnant woman who has a fractured femur
4. A client with diabetes who has a heart catherization
8. In a client in renal failure, which assessment finding may indicate hypocalcemia?

55

1. Headache
2. Serum calcium level of 5 mEq/L
3. Increased blood coagulation
4. Diarrhea
9. A nurse is assessing the patency of an arteriovenous fistula in the left arm of a client
who is receiving hemodialysis for the treatment of chronic renal failure. Which finding
indicates that the fistula is patent?
1. Absence of bruit on auscultation of the fistula.
2. Palpation of a thrill over the fistula
3. Presence of a radial pulse in the left wrist
4. Capillary refill time less than 3 seconds in the nail beds of the fingers on the left hand.
10. The client with chronic renal failure is at risk of developing dementia related to
excessive absorption of aluminum. The nurse teaches that this is the reason that the client
is being prescribed which of the following phosphate binding agents?
1. Alu-cap (aluminum hydroxide)
2. Tums (calcium carbonate)
3. Amphojel (aluminum hydroxide)
4. Basaljel (aluminum hydroxide)
11. The client newly diagnosed with chronic renal failure recently has begun hemodialysis.
Knowing that the client is at risk for disequilibrium syndrome, the nurse assesses the
client during dialysis for:
1. Hypertension, tachycardia, and fever
2. Hypotension, bradycardia, and hypothermia
3. restlessness, irritability, and generalized weakness
4. Headache, deteriorating level of consciousness, and twitching.
12. A client with chronic renal failure has completed a hemodialysis treatment. The nurse
would use which of the following standard indicators to evaluate the clients status after
dialysis?
1. Potassium level and weight
2. BUN and creatinine levels
3. VS and BUN
4. VS and weight.
13. The hemodialysis client with a left arm fistula is at risk for steal syndrome. The nurse
assesses this client for which of the following clinical manifestations?
1. Warmth, redness, and pain in the left hand.
2. Pallor, diminished pulse, and pain in the left hand.
3. Edema and reddish discoloration of the left arm
4. Aching pain, pallor, and edema in the left arm.
14. A client is admitted to the hospital and has a diagnosis of early stage chronic renal
failure. Which of the following would the nurse expect to note on assessment of the client?
1. Polyuria
2. Polydipsia
3. Oliguria
4. Anuria
15. The client with chronic renal failure returns to the nursing unit following a hemodialysis
treatment. On assessment the nurse notes that the clients temperature is 100.2. Which of
the following is the most appropriate nursing action?

56

1. Encourage fluids
2. Notify the physician
3. Monitor the site of the shunt for infection
4. Continue to monitor vital signs
16. The nurse is performing an assessment on a client who has returned from the dialysis
unit following hemodialysis. The client is complaining of a headache and nausea and is
extremely restless. Which of the following is the most appropriate nursing action?
1. Notify the physician
2. Monitor the client
3. Elevate the head of the bed
4. Medicate the client for nausea
17. The nurse is assisting a client on a low-potassium diet to select food items from the
menu. Which of the following food items, if selected by the client, would indicate an
understanding of this dietary restriction?
1. Cantaloupe
2. Spinach
3. Lima beans
4. Strawberries
18. The nurse is reviewing a list of components contained in the peritoneal dialysis solution
with the client. The client asks the nurse about the purpose of the glucose contained in the
solution. The nurse bases the response knowing that the glucose:
1. Prevents excess glucose from being removed from the client.
2. Decreases risk of peritonitis.
3. Prevents disequilibrium syndrome
4. Increases osmotic pressure to produce ultrafiltration.
19. The nurse is preparing to care for a client receiving peritoneal dialysis. Which of the
following would be included in the nursing plan of care to prevent the major complication
associated with peritoneal dialysis?
1. Monitor the clients level of consciousness
2. Maintain strict aseptic technique
3. Add heparin to the dialysate solution
4. Change the catheter site dressing daily
20. A client newly diagnosed with renal failure is receiving peritoneal dialysis. During the
infusion of the dialysate the client complains of abdominal pain. Which action by the nurse
is most appropriate?
1. Slow the infusion
2. Decrease the amount to be infused
3. Explain that the pain will subside after the first few exchanges
4. Stop the dialysis
21. The nurse is instructing a client with diabetes mellitus about peritoneal dialysis. The
nurse tells the client that it is important to maintain the dwell time for the dialysis at the
prescribed time because of the risk of:
1. Infection
2. Hyperglycemia
3. Fluid overload
4. Disequilibrium syndrome

57

22. The client with acute renal failure has a serum potassium level of 5.8 mEq/L. The nurse
would plan which of the following as a priority action?
1. Allow an extra 500 ml of fluid intake to dilute the electrolyte concentration.
2. Encourage increased vegetables in the diet
3. Place the client on a cardiac monitor
4. Check the sodium level
23. The client with chronic renal failure who is scheduled for hemodialysis this morning is
due to receive a daily dose of enalapril (Vasotec). The nurse should plan to administer this
medication:
1. Just before dialysis
2. During dialysis
3. On return from dialysis
4. The day after dialysis
24. The client with chronic renal failure has an indwelling catheter for peritoneal dialysis in
the abdomen. The client spills water on the catheter dressing while bathing. The nurse
should immediately:
1. Reinforce the dressing
2. Change the dressing
3. Flush the peritoneal dialysis catheter
4. Scrub the catheter with povidone-iodine
25. The client being hemodialyzed suddenly becomes short of breath and complains of
chest pain. The client is tachycardic, pale, and anxious. The nurse suspects air embolism.
The nurse should:
1. Continue the dialysis at a slower rate after checking the lines for air
2. Discontinue dialysis and notify the physician
3. Monitor vital signs every 15 minutes for the next hour
4. Bolus the client with 500 ml of normal saline to break up the air embolism.
26. The nurse has completed client teaching with the hemodialysis client about selfmonitoring between hemodialysis treatments. The nurse determines that the client best
understands the information given if the client states to record the daily:
1. Pulse and respiratory rate
2. Intake, output, and weight
3. BUN and creatinine levels
4. Activity log
27. The client with an arteriovenous shunt in place for hemodialysis is at risk for bleeding.
The nurse would do which of the following as a priority action to prevent this complication
from occurring?
1. Check the results of the PT time as they are ordered.
2. Observe the site once per shift
3. Check the shunt for the presence of a bruit and thrill
4. Ensure that small clamps are attached to the AV shunt dressing.
28. The nurse is monitoring a client receiving peritoneal dialysis and nurse notes that a
clients outflow is less than the inflow. Select actions that the nurse should take.
1. Place the client in good body alignment
2. Check the level of the drainage bag
3. Contact the physician

58

4. Check the peritoneal dialysis system for kinks


5. Reposition the client to his or her side.
29. The nurse assesses the client who has chronic renal failure and notes the following:
crackles in the lung bases, elevated blood pressure, and weight gain of 2 pounds in one
day. Based on these data, which of the following nursing diagnoses is appropriate?
1. Excess fluid volume related to the kidneys inability to maintain fluid balance.
2. Increased cardiac output related to fluid overload.
3. Ineffective tissue perfusion related to interrupted arterial blood flow.
4. Ineffective therapeutic Regimen Management related to lack of knowledge about therapy.
30. The nurse is caring for a hospitalized client who has chronic renal failure. Which of the
following nursing diagnoses are most appropriate for this client? Select all that apply.
1. Excess Fluid Volume
2. Imbalanced Nutrition; Less than Body Requirements
3. Activity Intolerance
4. Impaired Gas Exchange
5. Pain.
31. What is the primary disadvantage of using peritoneal dialysis for long term
management of chronic renal failure?
1. The danger of hemorrhage is high.
2. It cannot correct severe imbalances.
3. It is a time consuming method of treatment.
4. The risk of contracting hepatitis is high.
32. The dialysis solution is warmed before use in peritoneal dialysis primarily to:
1. Encourage the removal of serum urea.
2. Force potassium back into the cells.
3. Add extra warmth into the body.
4. Promote abdominal muscle relaxation.
33. During the clients dialysis, the nurse observes that the solution draining from the
abdomen is consistently blood tinged. The client has a permanent peritoneal catheter in
place. Which interpretation of this observation would be correct?
1. Bleeding is expected with a permanent peritoneal catheter
2. Bleeding indicates abdominal blood vessel damage
3. Bleeding can indicate kidney damage.
4. Bleeding is caused by too-rapid infusion of the dialysate.
34. Which of the following nursing interventions should be included in the clients care
plan during dialysis therapy?
1. Limit the clients visitors
2. Monitor the clients blood pressure
3. Pad the side rails of the bed
4. Keep the client NPO.
35. Aluminum hydroxide gel (Amphojel) is prescribed for the client with chronic renal
failure to take at home. What is the purpose of giving this drug to a client with chronic
renal failure?
1. To relieve the pain of gastric hyperacidity
2. To prevent Curlings stress ulcers
3. To bind phosphorus in the intestine
4. To reverse metabolic acidosis.

59

36. The nurse teaches the client with chronic renal failure when to take the aluminum
hydroxide gel. Which of the following statements would indicate that the client
understands the teaching?
1. Ill take it every 4 hours around the clock.
2. Ill take it between meals and at bedtime.
3. Ill take it when I have a sour stomach.
4. Ill take it with meals and bedtime snacks.
37. The client with chronic renal failure tells the nurse he takes magnesium hydroxide (milk
of magnesia) at home for constipation. The nurse suggests that the client switch to
psyllium hydrophilic mucilloid (Metamucil) because:
1. MOM can cause magnesium toxicity
2. MOM is too harsh on the bowel
3. Metamucil is more palatable
4. MOM is high in sodium
38. In planning teaching strategies for the client with chronic renal failure, the nurse must
keep in mind the neurologic impact of uremia. Which teaching strategy would be most
appropriate?
1. Providing all needed teaching in one extended session.
2. Validating frequently the clients understanding of the material.
3. Conducting a one-on-one session with the client.
4. Using videotapes to reinforce the material as needed.
39. The nurse helps the client with chronic renal failure develop a home diet plan with the
goal of helping the client maintain adequate nutritional intake. Which of the following diets
would be most appropriate for a client with chronic renal failure?
1. High carbohydrate, high protein
2. High calcium, high potassium, high protein
3. Low protein, low sodium, low potassium
4. Low protein, high potassium
40. A client with chronic renal failure has asked to be evaluated for a home continuous
ambulatory peritoneal dialysis (CAPD) program. The nurse should explain that the major
advantage of this approach is that it:
1. Is relatively low in cost
2. Allows the client to be more independent
3. Is faster and more efficient than standard peritoneal dialysis
4. Has fewer potential complications than standard peritoneal dialysis
41. The client asks whether her diet would change on CAPD. Which of the following would
be the nurses best response?
1. Diet restrictions are more rigid with CAPD because standard peritoneal dialysis is a more effective
technique.
2. Diet restrictions are the same for both CAPD and standard peritoneal dialysis.
3. Diet restrictions with CAPD are fewer than with standard peritoneal dialysis because dialysis is
constant.
4. Diet restrictions with CAPD are fewer than with standard peritoneal dialysis because CAPD works
more quickly.
42. Which of the following is the most significant sign of peritoneal infection?
1. Cloudy dialysate fluid
2. Swelling in the legs

60

3. Poor drainage of the dialysate fluid


4. Redness at the catheter insertion site
43. The main indicator of the need for hemodialysis is:
1. Ascites
2. Acidosis
3. Hypertension
4. Hyperkalemia
44. To gain access to the vein and artery, an AV shunt was used for Mr. Roberto. The most
serious problem with regards to the AV shunt is:
1. Septicemia
2. Clot formation
3. Exsanguination
4. Vessel sclerosis
45. When caring for Mr. Robertos AV shunt on his right arm, you should:
1. Cover the entire cannula with an elastic bandage
2. Notify the physician if a bruit and thrill are present
3. User surgical aseptic technique when giving shunt care
4. Take the blood pressure on the right arm instead

Answers and Rationale


1. Answer: 1. Osmosis allows for the removal of fluid from the blood by allowing it to pass through the
semipermeable membrane to an area of high concentrate (dialysate), and diffusion allows for passage
of particles (electrolytes, urea, and creatinine) from an area of higher concentration to an area of lower
concentration. Fluid passes to an area with a higher solute concentration. The pores of a
semipermeable membrane are small, thus preventing the flow of blood cells and protein molecules
through it.
2. Answer: 2. To prevent life-threatening complications, the client must follow the dialysis schedule.
Alcohol would further dry the clients skin more than it already is. The client should follow a lowpotassium diet because potassium levels increase in chronic renal failure. The client should know
hemodialysis is time-consuming and will definitely cause a change in current lifestyle.
3. Answer: 3. The first intervention should be to check for kinks and obstructions because that could be
preventing drainage. After checking for kinks, have the client change position to promote drainage.
Dont give the next scheduled exchange until the dialysate is drained because abdominal distention
will occur, unless the output is within parameters set by the physician. If unable to get more output
despite checking for kinks and changing the clients position, the nurse should then call the physician
to determine the proper intervention.
4. Answer: 1. Airway and oxygenation are always the first priority. Because the client is complaining of
shortness of breath and his oxygen saturation is only 89%, the nurse needs to try to increase his levels
by administering oxygen. The client is in pulmonary edema from fluid overload and will need to be
dialyzed and have his fluids restricted, but the first interventions should be aimed at the immediate
treatment of hypoxia. The foot of the bed may be elevated to reduce edema, but this isnt the priority.
5. Answer: 4. Assessment of the AV fistula for bruit and thrill is important because, if not present, it
indicates a non-functioning fistula. No blood pressures or venipunctures should be taken in the arm
with the AV fistula. When not being dialyzed, the AV fistula site may get wet. Immediately after a
dialysis treatment, the access site is covered with adhesive bandages.

61

6. Answer: 4. Although clients with renal failure can develop stress ulcers, the nausea is usually related
to the poisons of metabolic wastes that accumulate when the kidneys are unable to eliminate them.
The client has electrolyte imbalances and oliguria, but these dont directly cause nausea.
7. Answer: 4. Clients with diabetes are prone to renal insufficiency and renal failure. The contrast used
for heart catherization must be eliminated by the kidneys, which further stresses them and may
produce acute renal failure. A teenager who has an appendectomy and a pregnant woman with a
fractured femur isnt at increased risk for renal failure. A dialysis client already has end-stage renal
disease and wouldnt develop acute renal failure.
8. Answer: 4. In renal failure, calcium absorption from the intestine declines, leading to increased
smooth muscle contractions, causing diarrhea. CNS changes in renal failure rarely include headache. A
serum calcium level of 5 mEq/L indicates hypercalcemia. As renal failure progresses, bleeding
tendencies increase.
9. Answer: 2. The nurse assesses the patency of the fistula by palpating for the presence of a thrill or
auscultating for a bruit. The presence of a thrill and bruit indicate patency of the fistula. Although the
presence of a radial pulse in the left wrist and capillary refill time less than 3 seconds in the nail beds
of the fingers on the left hand are normal findings, they do not assess fistula patency.
10. Answer: 2. Phosphate binding agents that contain aluminum include Alu-caps, Basaljel, and
Amphojel. These products are made from aluminum hydroxide. Tums are made from calcium carbonate
and also bind phosphorus. Tums are prescribed to avoid the occurrence of dementia related to high
intake of aluminum. Phosphate binding agents are needed by the client in renal failure because the
kidneys cannot eliminate phosphorus.
11. Answer: 4. Disequilibrium syndrome is characterized by headache, mental confusion, decreasing
level of consciousness, nausea, and vomiting, twitching, and possible seizure activity. Disequilibrium
syndrome is caused by rapid removal of solutes from the body during hemodialysis. At the same time,
the blood-brain barrier interferes with the efficient removal of wastes from brain tissue. As a result,
water goes into cerebral cells because of the osmotic gradient, causing brain swelling and onset of
symptoms. The syndrome most often occurs in clients who are new to dialysis and is prevented by
dialyzing for shorter times or at reduced blood flow rates.
12. Answer: 4. Following dialysis, the clients vital signs are monitored to determine whether the client
is remaining hemodynamically stable. Weight is measured and compared with the clients predialysis
weight to determine effectiveness of fluid extraction. Laboratory studies are done as per protocol but
are not necessarily done after the hemodialysis treatment has ended.
13. Answer: 2. Steal syndrome results from vascular insufficiency after creation of a fistula. The client
exhibits pallor and a diminished pulse distal to the fistula. The client also complains of pain distal to
the fistula, which is due to tissue ischemia. Warmth, redness, and pain more likely would characterize a
problem with infection.
14. Answer: 1. Polyuria occurs early in chronic renal failure and if untreated can cause severe
dehydration. Polyuria progresses to anuria, and the client loses all normal functions of the kidney.
Oliguria and anuria are not early signs, and polydipsia is unrelated to chronic renal failure.
15. Answer: 4. The client may have an elevated temperature following dialysis because the dialysis
machine warms the blood slightly. If the temperature is elevated excessively and remains elevated,
sepsis would be suspected and a blood sample would be obtained as prescribed for culture and
sensitivity purposes.
16. Answer: 1. Disequilibrium syndrome may be due to the rapid decrease in BUN levels during
dialysis. These changes can cause cerebral edema that leads to increased intracranial pressure. The
client is exhibiting early signs of disequilibrium syndrome and appropriate treatments with

62

anticonvulsant medications and barbituates may be necessary to prevent a life-threatening situation.


The physician must be notified.
17. Answer: Cantaloupe (1/4 small), spinach (1/2 cooked) and strawberries (1 cups) are high
potassium foods and average 7 mEq per serving. Lima beans (1/3 c) averages 3 mEq per serving.
18. Answer: 4. Increasing the glucose concentration makes the solution increasingly more hypertonic.
The more hypertonic the solution, the greater the osmotic pressure for ultrafiltration and thus the
greater amount of fluid removed from the client during an exchange.
19. Answer: 2. The major complication of peritoneal dialysis is peritonitis. Strict aseptic technique is
required in caring for the client receiving this treatment. Although option 4 may assist in preventing
infection, this option relates to an external site.
20. Answer: 3. Pain during the inflow of dialysate is common during the first few exchanges because of
peritoneal irritation; however, the pain usually disappears after 1 to 2 weeks of treatment. The infusion
amount should not be decreased, and the infusion should not be slowed or stopped.
21. Answer: 2. An extended dwell time increases the risk of hyperglycemia in the client with diabetes
mellitus as a result of absorption of glucose from the dialysate and electrolyte changes. Diabetic
clients may require extra insulin when receiving peritoneal dialysis.
22. Answer: 3. The client with hyperkalemia is at risk for developing cardiac dysrhythmias and cardiac
arrest. Because of this the client should be placed on a cardiac monitor. Fluid intake is not increased
because it contributes to fluid overload and would not affect the serum potassium level significantly.
Vegetables are a natural source of potassium in the diet, and their use would not be increased. The
nurse may also assess the sodium level because sodium is another electrolyte commonly measured
with the potassium level. However, this is not a priority action at this time.
23. Answer: 3. Antihypertensive medications such as enalapril are given to the client following
hemodialysis. This prevents the client from becoming hypotensive during dialysis and also from having
the medication removed from the bloodstream by dialysis. No rationale exists for waiting a full day to
resume the medication. This would lead to ineffective control of the blood pressure.
24. Answer: 2. Clients with peritoneal dialysis catheters are at high risk for infection. A dressing that is
wet is a conduit for bacteria for bacteria to reach the catheter insertion site. The nurse assures that the
dressing is kept dry at all times. Reinforcing the dressing is not a safe practice to prevent infection in
this circumstance. Flushing the catheter is not indicated. Scrubbing the catheter with povidone-iodine
is done at the time of connection or disconnecting of peritoneal dialysis.
25. Answer: 2. If the client experiences air embolus during hemodialysis, the nurse should terminate
dialysis immediately, notify the physician, and administer oxygen as needed.
26. Answer: 2. The client on hemodialysis should monitor fluid status between hemodialysis treatments
by recording intake and output and measuring weight daily. Ideally, the hemodialysis client should not
gain more than 0.5 kg of weight per day.
27. Answer: 4. An AV shunt is a less common form of access site but carries a risk for bleeding when it
is used because two ends of an external cannula are tunneled subcutaneously into an artery and a
vein, and the ends of the cannula are joined. If accidental connection occurs, the client could lose
blood rapidly. For this reason, small clamps are attached to the dressing that covers the insertion site
to use if needed. The shunt site should be assessed at least every four hours.
28. Answer: 1, 2, 4, 5. If outflow drainage is inadequate, the nurse attempts to stimulate outflow by
changing the clients position. Turning the client to the other side or making sure that the client is in
good body alignment may assist with outflow drainage. The drainage bag needs to be lower than the
clients abdomen to enhance gravity drainage. The connecting tubing and the peritoneal dialysis
system is also checked for kinks or twisting and the clamps on the system are checked to ensure that
they are open. There is no reason to contact the physician.

63

29. Answer: 1. Crackles in the lungs, weight gain, and elevated blood pressure are indicators of excess
fluid volume, a common complication in chronic renal failure. The clients fluid status should be
monitored carefully for imbalances on an ongoing basis.
30. Answer: 1, 2, 3. Appropriate nursing diagnoses for clients with chronic renal failure include excess
fluid volume related to fluid and sodium retention; imbalanced nutrition, less than body requirements
related to anorexia, nausea, and vomiting; and activity intolerance related to fatigue. The nursing
diagnoses of impaired gas exchange and pain are not commonly related to chronic renal failure.
31. Answer: 3. The disadvantages of peritoneal dialysis in long-term management of chronic renal
failure is that is requires large blocks of time. The risk of hemorrhage or hepatitis is not high with PD.
PD is effective in maintaining a clients fluid and electrolyte balance.
32. Answer: 1. The main reason for warming the peritoneal dialysis solution is that the warm solution
helps dilate peritoneal vessels, which increases urea clearance. Warmed dialyzing solution also
contributes to client comfort by preventing chilly sensations, but this is a secondary reason for
warming the solution. The warmed solution does not force potassium into the cells or promote
abdominal muscle relaxation.
33. Answer: 2. Because the client has a permanent catheter in place, blood tinged drainage should not
occur. Persistent blood tinged drainage could indicate damage to the abdominal vessels, and the
physician should be notified. The bleeding is originating in the peritoneal cavity, not the kidneys. Too
rapid infusion of the dialysate can cause pain.
34. Answer: 2. Because hypotension is a complication of peritoneal dialysis, the nurse records intake
and output, monitors VS, and observes the clients behavior. The nurse also encourages visiting and
other diversional activities. A client on PD does not need to be placed in bed with padded side rails or
kept NPO.
35. Answer: 3. A client in renal failure develops hyperphosphatemia that causes a corresponding
excretion of the bodys calcium stores, leading to renal osteodystrophy. To decrease this loss,
aluminum hydroxide gel is prescribed to bind phosphates in the intestine and facilitate their excretion.
Gastric hyperacidity is not necessarily a problem associated with chronic renal failure. Antacids will not
prevent Curlings stress ulcers and do not affect metabolic acidosis.
36. Answer: 3. Aluminum hydroxide gel is administered to bind the phosphates in ingested foods and
must be given with or immediately after meals and snacks. There is no need for the client to take it on
a 24-hour schedule. It is not administered to treat hyperacidity in clients with CRF and therefore is not
prescribed between meals.
37. Answer: 1. Magnesium is normally excreted by the kidneys. When the kidneys fail, magnesium can
accumulate and cause severe neurologic problems. MOM is harsher than Metamucil, but magnesium
toxicity is a more serious problem. A client may find both MOM and Metamucil unpalatable. MOM is not
high in sodium.
38. Answer: 2. Uremia can cause decreased alertness, so the nurse needs to validate the clients
comprehension frequently. Because the clients ability to concentrate is limited, short lesions are most
effective. If family members are present at the sessions, they can reinforce the material. Written
materials that the client can review are superior to videotapes, because the clients may not be able to
maintain alertness during the viewing of the videotape.
39. Answer: 3. Dietary management for clients with chronic renal failure is usually designed to restrict
protein, sodium, and potassium intake. Protein intake is reduced because the kidney can no longer
excrete the byproducts of protein metabolism. The degree of dietary restriction depends on the degree
of renal impairment. The client should also receive a high carbohydrate diet along with appropriate
vitamin and mineral supplements. Calcium requirements remain 1,000 to 2,000 mg/day.

64

40. Answer: 2. The major benefit of CAPD is that it frees the client from daily dependence on dialysis
centers, home health care personnel, and machines for life-sustaining treatment. The independence is
a valuable outcome for some people. CAPD is costly and must be done daily. Side effects and
complications are similar to those of standard peritoneal dialysis.
41. Answer: 3. Dietary restrictions with CAPD are fewer than those with standard peritoneal dialysis
because dialysis is constant, not intermittent. The constant slow diffusion of CAPD helps prevent
accumulation of toxins and allows for a more liberal diet. CAPD does not work more quickly, but more
consistently. Both types of peritoneal dialysis are effective.
42. Answer: 1. Cloudy drainage indicates bacterial activity in the peritoneum. Other signs and
symptoms of infection are fever, hyperactive bowel sounds, and abdominal pain. Swollen legs may be
indicative of congestive heart failure. Poor drainage of dialysate fluid is probably the result of a kinked
catheter. Redness at the insertion site indicates local infection, not peritonitis. However, a local
infection that is left untreated can progress to the peritoneum.
43. Answer: 4. Hyperkalemia
44. Answer: 3. Exsanguination
45. Answer: 3. User surgical aseptic technique when giving shunt care

Introduction
A 25-item NCLEX-style examination about Neurological Disorders that covers topics like: Meningitis,
Level of Consciousness, Seizures and more.

Topics

Increased Intracranial Pressure

Level of Consciousness

Meningitis

Guidelines

Read each question carefully and choose the best answer.

You are given one minute per question. Spend your time wisely!

Answers and rationales are given below. Be sure to read them.

If you need more clarifications, please direct them to the comments section.

Questions
1. A client admitted to the hospital with a subarachnoid hemorrhage has complaints of
severe headache, nuchal rigidity, and projectile vomiting. The nurse knows lumbar
puncture (LP) would be contraindicated in this client in which of the following
circumstances?
1. Vomiting continues
2. Intracranial pressure (ICP) is increased
3. The client needs mechanical ventilation
4. Blood is anticipated in the cerebrospinal fluid (CSF)
2. A client with a subdural hematoma becomes restless and confused, with dilation of the
ipsilateral pupil. The physician orders mannitol for which of the following reasons?
1. To reduce intraocular pressure
2. To prevent acute tubular necrosis

65

3. To promote osmotic diuresis to decrease ICP


4. To draw water into the vascular system to increase blood pressure
3. A client with subdural hematoma was given mannitol to decrease intracranial pressure
(ICP). Which of the following results would best show the mannitol was effective?
1. Urine output increases
2. Pupils are 8 mm and nonreactive
3. Systolic blood pressure remains at 150 mm Hg
4. BUN and creatinine levels return to normal
4. Which of the following values is considered normal for ICP?
1. 0 to 15 mm Hg
2. 25 mm Hg
3. 35 to 45 mm Hg
4. 120/80 mm Hg
5. Which of the following symptoms may occur with a phenytoin level of 32 mg/dl?
1. Ataxia and confusion
2. Sodium depletion
3. Tonic-clonic seizure
4. Urinary incontinence
6. Which of the following signs and symptoms of increased ICP after head trauma would
appear first?
1. Bradycardia
2. Large amounts of very dilute urine
3. Restlessness and confusion
4. Widened pulse pressure
7. Problems with memory and learning would relate to which of the following lobes?
1. Frontal
2. Occipital
3. Parietal
4. Temporal
8. While cooking, your client couldnt feel the temperature of a hot oven. Which lobe could
be dysfunctional?
1. Frontal
2. Occipital
3. Parietal
4. Temporal
9. The nurse is assessing the motor function of an unconscious client. The nurse would plan
to use which of the following to test the clients peripheral response to pain?
1. Sternal rub
2. Pressure on the orbital rim
3. Squeezing the sternocleidomastoid muscle
4. Nail bed pressure
10. The client is having a lumbar puncture performed. The nurse would plan to place the
client in which position for the procedure?
1. Side-lying, with legs pulled up and head bent down onto the chest
2. Side-lying, with a pillow under the hip
3. Prone, in a slight Trendelenburgs position
4. Prone, with a pillow under the abdomen.

66

11. A nurse is assisting with caloric testing of the oculovestibular reflex of an unconscious
client. Cold water is injected into the left auditory canal. The client exhibits eye conjugate
movements toward the left followed by a rapid nystagmus toward the right. The nurse
understands that this indicates the client has:
1. A cerebral lesion
2. A temporal lesion
3. An intact brainstem
4. Brain death
12. The nurse is caring for the client with increased intracranial pressure. The nurse would
note which of the following trends in vital signs if the ICP is rising?
1. Increasing temperature, increasing pulse, increasing respirations, decreasing blood pressure.
2. Increasing temperature, decreasing pulse, decreasing respirations, increasing blood pressure.
3. Decreasing temperature, decreasing pulse, increasing respirations, decreasing blood pressure.
4. Decreasing temperature, increasing pulse, decreasing respirations, increasing blood pressure.
13. The nurse is evaluating the status of a client who had a craniotomy 3 days ago. The
nurse would suspect the client is developing meningitis as a complication of surgery if the
client exhibits:
1. A positive Brudzinskis sign
2. A negative Kernigs sign
3. Absence of nuchal rigidity
4. A Glascow Coma Scale score of 15
14. A client is arousing from a coma and keeps saying, Just stop the pain. The nurse
responds based on the knowledge that the human body typically and automatically
responds to pain first with attempts to:
1. Tolerate the pain
2. Decrease the perception of pain
3. Escape the source of pain
4. Divert attention from the source of pain.
15. During the acute stage of meningitis, a 3-year-old child is restless and irritable. Which
of the following would be most appropriate to institute?
1. Limiting conversation with the child
2. Keeping extraneous noise to a minimum
3. Allowing the child to play in the bathtub
4. Performing treatments quickly
16. Which of the following would lead the nurse to suspect that a child with meningitis has
developed disseminated intravascular coagulation?
1. Hemorrhagic skin rash
2. Edema
3. Cyanosis
4. Dyspnea on exertion
17. When interviewing the parents of a 2-year-old child, a history of which of the following
illnesses would lead the nurse to suspect pneumococcal meningitis?
1. Bladder infection
2. Middle ear infection
3. Fractured clavicle
4. Septic arthritis

67

18. The nurse is assessing a child diagnosed with a brain tumor. Which of the following
signs and symptoms would the nurse expect the child to demonstrate? Select all that
apply.
1. Head tilt
2. Vomiting
3. Polydipsia
4. Lethargy
5. Increased appetite
6. Increased pulse
19. A lumbar puncture is performed on a child suspected of having bacterial meningitis.
CSF is obtained for analysis. A nurse reviews the results of the CSF analysis and
determines that which of the following results would verify the diagnosis?
1. Cloudy CSF, decreased protein, and decreased glucose
2. Cloudy CSF, elevated protein, and decreased glucose
3. Clear CSF, elevated protein, and decreased glucose
4. Clear CSF, decreased pressure, and elevated protein
20. A nurse is planning care for a child with acute bacterial meningitis. Based on the mode
of transmission of this infection, which of the following would be included in the plan of
care?
1. No precautions are required as long as antibiotics have been started
2. Maintain enteric precautions
3. Maintain respiratory isolation precautions for at least 24 hours after the initiation of antibiotics
4. Maintain neutropenic precautions
21. A nurse is reviewing the record of a child with increased ICP and notes that the child
has exhibited signs of decerebrate posturing. On assessment of the child, the nurse would
expect to note which of the following if this type of posturing was present?
1. Abnormal flexion of the upper extremities and extension of the lower extremities
2. Rigid extension and pronation of the arms and legs
3. Rigid pronation of all extremities
4. Flaccid paralysis of all extremities
22. Which of the following assessment data indicated nuchal rigidity?
1. Positive Kernigs sign
2. Negative Brudzinskis sign
3. Positive homans sign
4. Negative Kernigs sign
23. Meningitis occurs as an extension of a variety of bacterial infections due to which of
the following conditions?
1. Congenital anatomic abnormality of the meninges
2. Lack of acquired resistance to the various etiologic organisms
3. Occlusion or narrowing of the CSF pathway
4. Natural affinity of the CNS to certain pathogens
24. Which of the following pathologic processes is often associated with aseptic
meningitis?
1. Ischemic infarction of cerebral tissue
2. Childhood diseases of viral causation such as mumps
3. Brain abscesses caused by a variety of pyogenic organisms
4. Cerebral ventricular irritation from a traumatic brain injury

68

25. You are preparing to admit a patient with a seizure disorder. Which of the following
actions can you delegate to LPN/LVN?
1. Complete admission assessment.
2. Set up oxygen and suction equipment.
3. Place a padded tongue blade at bedside.
4. Pad the side rails before patient arrives.

Answers and Rationale


1. Answer: 2. Sudden removal of CSF results in pressures lower in the lumbar area than the brain and
favors herniation of the brain; therefore, LP is contraindicated with increased ICP. Vomiting may be
caused by reasons other than increased ICP; therefore, LP isnt strictly contraindicated. An LP may be
performed on clients needing mechanical ventilation. Blood in the CSF is diagnostic for subarachnoid
hemorrhage and was obtained before signs and symptoms of ICP.
2. Answer: 3. Mannitol promotes osmotic diuresis by increasing the pressure gradient, drawing fluid
from intracellular to intravascular spaces. Although mannitol is used for all the reasons described, the
reduction of ICP in this client is a concern.
3. Answer: 1. Mannitol promotes osmotic diuresis by increasing the pressure gradient in the renal
tubes. Fixed and dilated pupils are symptoms of increased ICP or cranial nerve damage. No information
is given about abnormal BUN and creatinine levels or that mannitol is being given for renal dysfunction
or blood pressure maintenance.
4. Answer: 1. Normal ICP is 0-15 mm Hg.
5. Answer: 1. A therapeutic phenytoin level is 10 to 20 mg/dl. A level of 32 mg/dl indicates toxicity.
Symptoms of toxicity include confusion and ataxia. Phenytoin doesnt cause hyponatremia, seizure, or
urinary incontinence. Incontinence may occur during or after a seizure.
6. Answer: 3. The earliest symptom of elevated ICP is a change in mental status. Bradycardia, widened
pulse pressure, and bradypnea occur later. The client may void large amounts of very dilute urine if
theres damage to the posterior pituitary.
7. Answer: 4. The temporal lobe functions to regulate memory and learning problems because of the
integration of the hippocampus. The frontal lobe primarily functions to regulate thinking, planning, and
judgment. The occipital lobe functions regulate vision. The parietal lobe primarily functions with
sensory function.
8. Answer: 3. The parietal lobe regulates sensory function, which would include the ability to sense hot
or cold objects. The frontal lobe regulates thinking, planning, and judgment, and the occipital lobe is
primarily responsible for vision function. The temporal lobe regulates memory.
9. Answer: 4. Motor testing on the unconscious client can be done only by testing response to painful
stimuli. Nail Bed pressure tests a basic peripheral response. Cerebral responses to pain are testing
using sternal rub, placing upward pressure on the orbital rim, or squeezing the clavicle or
sternocleidomastoid muscle.
10. Answer: 1. The client undergoing lumbar puncture is positioned lying on the side, with the legs
pulled up to the abdomen, and with the head bent down onto the chest. This position helps to open the
spaces between the vertebrae.
11. Answer: 3. Caloric testing provides information about differentiating between cerebellar and
brainstem lesions. After determining patency of the ear canal, cold or warm water is injected in the
auditory canal. A normal response that indicates intact function of cranial nerves III, IV, and VIII is
conjugate eye movements toward the side being irrigated, followed by rapid nystagmus to the
opposite side. Absent or disconjugate eye movements indicate brainstem damage.

69

12. Answer: 2. A change in vital signs may be a late sign of increased intracranial pressure. Trends
include increasing temperature and blood pressure and decreasing pulse and respirations. Respiratory
irregularities also may arise.
13. Answer: 1. Signs of meningeal irritation compatible with meningitis include nuchal rigidity, positive
Brudzinskis sign, and positive Kernigs sign. Nuchal rigidity is characterized by a stiff neck and
soreness, which is especially noticeable when the neck is fixed. Kernigs sign is positive when the client
feels pain and spasm of the hamstring muscles when the knee and thigh are extended from a flexedright angle position. Brudzinskis sign is positive when the client flexes the hips and knees in response
to the nurse gently flexing the head and neck onto the chest. A Glascow Coma Scale of 15 is a perfect
score and indicates the client is awake and alert with no neurological deficits.
14. Answer: 3. The clients innate responses to pain are directed initially toward escaping from the
source of pain. Variations in individuals tolerance and perception of pain are apparent only in
conscious clients, and only conscious clients are able to employ distraction to help relieve pain.
15. Answer: 2. A child in the acute stage of meningitis is irritable and hypersensitive to loud noise and
light. Therefore, extraneous noise should be minimized and bright lights avoided as much as possible.
There is no need to limit conversations with the child. However, the nurse should speak in a calm,
gentle, reassuring voice. The child needs gentle and calm bathing. Because of the acuteness of the
infection, sponge baths would be more appropriate than tub baths. Although treatments need to be
completed as quickly as possible to prevent overstressing the child, any treatments should be
performed carefully and at a pace that avoids sudden movements to prevent startling the child and
subsequently increasing intracranial pressure.
16. Answer: 1. DIC is characterized by skin petechiae and a purpuric skin rash caused by spontaneous
bleeding into the tissues. An abnormal coagulation phenomenon causes the condition.
17. Answer: 2. Organisms that cause bacterial meningitis, such as pneumococci or meningococci, are
commonly spread in the body by vascular dissemination from a middle ear infection. The meningitis
may also be a direct extension from the paranasal and mastoid sinuses. The causative organism is a
pneumococcus. A chronically draining ear is frequently also found.
18. Answer: 1, 2, 4. Head tilt, vomiting, and lethargy are classic signs assessed in a child with a brain
tumor. Clinical manifestations are the result of location and size of the tumor.
19. Answer: 2. A diagnosis of meningitis is made by testing CSF obtained by lumbar puncture. In the
case of bacterial meningitis, findings usually include an elevated pressure, turbid or cloudy CSF,
elevated leukocytes, elevated protein, and decreased glucose levels.
20. Answer: 3. A major priority of nursing care for a child suspected of having meningitis is to
administer the prescribed antibiotic as soon as it is ordered. The child is also placed on respiratory
isolation for at least 24 hours while culture results are obtained and the antibiotic is having an effect.
21. Answer: 2. Decerebrate posturing is characterized by the rigid extension and pronation of the arms
and legs.
22. Answer: 1. A positive Kernigs sign indicated nuchal rigidity, caused by an irritative lesion of the
subarachnoid space. Brudzinskis sign is also indicative of the condition.
23. Answer: 2. Extension of a variety of bacterial infections is a major causative factor of meningitis
and occurs as a result of a lack of acquired resistance to the etiologic organisms. Preexisting CNS
anomalies are factors that contribute to susceptibility.
24. Answer: 2. Aseptic meningitis is caused principally by viruses and is often associated with other
diseases such as measles, mumps, herpes, and leukemia. Incidences of brain abscess are high in
bacterial meningitis, and ischemic infarction of cerebral tissue can occur with tubercular meningitis.
Traumatic brain injury could lead to bacterial (not viral) meningitis.

70

25. Answer: 2 The LPN/LVN can set up the equipment for oxygen and suctioning. The RN should
perform the complete initial assessment. Padded side rails are controversial in terms of whether they
actually provide safety and ay embarrass the patient and family. Tongue blades should not be at the
bedside and should never be inserted into the patients mouth after a seizure begins. Focus:
Delegation/supervision.

Introduction
This is a 50-item NCLEX style examination all about diseases affecting the Neurological System. Topics
included here are: Seizure, Head Trauma, Spinal Cord Injury and more.

Topics

Seizures

Head Trauma

Spinal Cord Injury

Guidelines

Read each question carefully and choose the best answer.

You are given one minute per question. Spend your time wisely!

Answers and rationales are given below. Be sure to read them.

If you need more clarifications, please direct them to the comments section.

Questions
1. An 18-year-old client is admitted with a closed head injury sustained in a MVA. His
intracranial pressure (ICP) shows an upward trend. Which intervention should the nurse
perform first?
1. Reposition the client to avoid neck flexion
2. Administer 1 g Mannitol IV as ordered
3. Increase the ventilators respiratory rate to 20 breaths/minute
4. Administer 100 mg of pentobarbital IV as ordered.
2. A client with a subarachnoid hemorrhage is prescribed a 1,000-mg loading dose of
Dilantin IV. Which consideration is most important when administering this dose?
1. Therapeutic drug levels should be maintained between 20 to 30 mg/ml.
2. Rapid dilantin administration can cause cardiac arrhythmias.
3. Dilantin should be mixed in dextrose in water before administration.
4. Dilantin should be administered through an IV catheter in the clients hand.
3. A client with head trauma develops a urine output of 300 ml/hr, dry skin, and dry mucous
membranes. Which of the following nursing interventions is the most appropriate to
perform initially?
1. Evaluate urine specific gravity
2. Anticipate treatment for renal failure
3. Provide emollients to the skin to prevent breakdown
4. Slow down the IV fluids and notify the physician
4. When evaluating an ABG from a client with a subdural hematoma, the nurse notes the
PaCO2 is 30 mm Hg. Which of the following responses best describes this result?

71

1. Appropriate; lowering carbon dioxide (CO2) reduces intracranial pressure (ICP).


2. Emergent; the client is poorly oxygenated.
3. Normal
4. Significant; the client has alveolar hypoventilation.
5. A client who had a transsphenoidal hypophysectomy should be watched carefully for
hemorrhage, which may be shown by which of the following signs?
1. Bloody drainage from the ears
2. Frequent swallowing
3. Guaiac-positive stools
4. Hematuria
6. After a hypophysectomy, vasopressin is given IM for which of the following reasons?
1. To treat growth failure
2. To prevent syndrome of inappropriate antidiuretic hormone (SIADH)
3. To reduce cerebral edema and lower intracranial pressure
4. To replace antidiuretic hormone (ADH) normally secreted by the pituitary.
7. A client comes into the ER after hitting his head in an MVA. Hes alert and oriented.
Which of the following nursing interventions should be done first?
1. Assess full ROM to determine extent of injuries
2. Call for an immediate chest x-ray
3. Immobilize the clients head and neck
4. Open the airway with the head-tilt chin-lift maneuver
8. A client with a C6 spinal injury would most likely have which of the following symptoms?
1. Aphasia
2. Hemiparesis
3. Paraplegia
4. Tetraplegia
9. A 30-year-old was admitted to the progressive care unit with a C5 fracture from a
motorcycle accident. Which of the following assessments would take priority?
1. Bladder distension
2. Neurological deficit
3. Pulse ox readings
4. The clients feelings about the injury
10. While in the ER, a client with C8 tetraplegia develops a blood pressure of 80/40, pulse
48, and RR of 18. The nurse suspects which of the following conditions?
1. Autonomic dysreflexia
2. Hemorrhagic shock
3. Neurogenic shock
4. Pulmonary embolism
11. A client is admitted with a spinal cord injury at the level of T12. He has limited
movement of his upper extremities. Which of the following medications would be used to
control edema of the spinal cord?
1. Acetazolamide (Diamox)
2. Furosemide (Lasix)
3. Methylprednisolone (Solu-Medrol)
4. Sodium bicarbonate

72

12. A 22-year-old client with quadriplegia is apprehensive and flushed, with a blood
pressure of 210/100 and a heart rate of 50 bpm. Which of the following nursing
interventions should be done first?
1. Place the client flat in bed
2. Assess patency of the indwelling urinary catheter
3. Give one SL nitroglycerin tablet
4. Raise the head of the bed immediately to 90 degrees
13. A client with a cervical spine injury has Gardner-Wells tongs inserted for which of the
following reasons?
1. To hasten wound healing
2. To immobilize the cervical spine
3. To prevent autonomic dysreflexia
4. To hold bony fragments of the skull together
14. Which of the following interventions describes an appropriate bladder program for a
client in rehabilitation for spinal cord injury?
1. Insert an indwelling urinary catheter to straight drainage
2. Schedule intermittent catheterization every 2 to 4 hours
3. Perform a straight catheterization every 8 hours while awake
4. Perform Credes maneuver to the lower abdomen before the client voids.
15. A client is admitted to the ER for head trauma is diagnosed with an epidural hematoma.
The underlying cause of epidural hematoma is usually related to which of the following
conditions?
1. Laceration of the middle meningeal artery
2. Rupture of the carotid artery
3. Thromboembolism from a carotid artery
4. Venous bleeding from the arachnoid space
16. A 23-year-old client has been hit on the head with a baseball bat. The nurse notes clear
fluid draining from his ears and nose. Which of the following nursing interventions should
be done first?
1. Position the client flat in bed
2. Check the fluid for dextrose with a dipstick
3. Suction the nose to maintain airway patency
4. Insert nasal and ear packing with sterile gauze
17. When discharging a client from the ER after a head trauma, the nurse teaches the
guardian to observe for a lucid interval. Which of the following statements best described
a lucid interval?
1. An interval when the clients speech is garbled
2. An interval when the client is alert but cant recall recent events
3. An interval when the client is oriented but then becomes somnolent
4. An interval when the client has a warning symptom, such as an odor or visual disturbance.
18. Which of the following clients on the rehab unit is most likely to develop autonomic
dysreflexia?
1. A client with a brain injury
2. A client with a herniated nucleus pulposus
3. A client with a high cervical spine injury
4. A client with a stroke

73

19. Which of the following conditions indicates that spinal shock is resolving in a client
with C7 quadriplegia?
1. Absence of pain sensation in chest
2. Spasticity
3. Spontaneous respirations
4. Urinary continence
20. A nurse assesses a client who has episodes of autonomic dysreflexia. Which of the
following conditions can cause autonomic dysreflexia?
1. Headache
2. Lumbar spinal cord injury
3. Neurogenic shock
4. Noxious stimuli
21. During an episode of autonomic dysreflexia in which the client becomes hypertensive,
the nurse should perform which of the following interventions?
1. Elevate the clients legs
2. Put the client flat in bed
3. Put the client in the Trendelenburgs position
4. Put the client in the high-Fowlers position
22. A client with a T1 spinal cord injury arrives at the emergency department with a BP of
82/40, pulse 34, dry skin, and flaccid paralysis of the lower extremities. Which of the
following conditions would most likely be suspected?
1. Autonomic dysreflexia
2. Hypervolemia
3. Neurogenic shock
4. Sepsis
23. A client has a cervical spine injury at the level of C5. Which of the following conditions
would the nurse anticipate during the acute phase?
1. Absent corneal reflex
2. Decerebrate posturing
3. Movement of only the right or left half of the body
4. The need for mechanical ventilation
24. A client with C7 quadriplegia is flushed and anxious and complains of a pounding
headache. Which of the following symptoms would also be anticipated?
1. Decreased urine output or oliguria
2. Hypertension and bradycardia
3. Respiratory depression
4. Symptoms of shock
25. A 40-year-old paraplegic must perform intermittent catheterization of the bladder.
Which of the following instructions should be given?
1. Clean the meatus from back to front.
2. Measure the quantity of urine.
3. Gently rotate the catheter during removal.
4. Clean the meatus with soap and water.
26. An 18-year-old client was hit in the head with a baseball during practice. When
discharging him to the care of his mother, the nurse gives which of the following
instructions?

74

1. Watch him for keyhole pupil the next 24 hours.


2. Expect profuse vomiting for 24 hours after the injury.
3. Wake him every hour and assess his orientation to person, time, and place.
4. Notify the physician immediately if he has a headache.
27. Which neurotransmitter is responsible for may of the functions of the frontal lobe?
1. Dopamine
2. GABA
3. Histamine
4. Norepinephrine
28. The nurse is discussing the purpose of an electroencephalogram (EEG) with the family
of a client with massive cerebral hemorrhage and loss of consciousness. It would be most
accurate for the nurse to tell family members that the test measures which of the following
conditions?
1. Extent of intracranial bleeding
2. Sites of brain injury
3. Activity of the brain
4. Percent of functional brain tissue
29. A client arrives at the ER after slipping on a patch of ice and hitting her head. A CT scan
of the head shows a collection of blood between the skull and dura mater. Which type of
head injury does this finding suggest?
1. Subdural hematoma
2. Subarachnoid hemorrhage
3. Epidural hematoma
4. Contusion
30. After falling 20, a 36-year-old man sustains a C6 fracture with spinal cord transaction.
Which other findings should the nurse expect?
1. Quadriplegia with gross arm movement and diaphragmatic breathing
2. Quadriplegia and loss of respiratory function
3. Paraplegia with intercostal muscle loss
4. Loss of bowel and bladder control
31. A 20-year-old client who fell approximately 30 is unresponsive and breathless. A
cervical spine injury is suspected. How should the first-responder open the clients airway
for rescue breathing?
1. By inserting a nasopharyngeal airway
2. By inserting a oropharyngeal airway
3. By performing a jaw-thrust maneuver
4. By performing the head-tilt, chin-lift maneuver
32. The nurse is caring for a client with a T5 complete spinal cord injury. Upon assessment,
the nurse notes flushed skin, diaphoresis above the T5, and a blood pressure of 162/96.
The client reports a severe, pounding headache. Which of the following nursing
interventions would be appropriate for this client? Select all that apply.
1. Elevate the HOB to 90 degrees
2. Loosen constrictive clothing
3. Use a fan to reduce diaphoresis
4. Assess for bladder distention and bowel impaction
5. Administer antihypertensive medication
6. Place the client in a supine position with legs elevated

75

33. The client with a head injury has been urinating copious amounts of dilute urine
through the Foley catheter. The clients urine output for the previous shift was 3000 ml.
The nurse implements a new physician order to administer:
1. Desmopressin (DDAVP, stimate)
2. Dexamethasone (Decadron)
3. Ethacrynic acid (Edecrin)
4. Mannitol (Osmitrol)
34. The nurse is caring for the client in the ER following a head injury. The client
momentarily lost consciousness at the time of the injury and then regained it. The client
now has lost consciousness again. The nurse takes quick action, knowing this is compatible
with:
1. Skull fracture
2. Concussion
3. Subdural hematoma
4. Epidural hematoma
35. The nurse is caring for a client who suffered a spinal cord injury 48 hours ago. The
nurse monitors for GI complications by assessing for:
1. A flattened abdomen
2. Hematest positive nasogastric tube drainage
3. Hyperactive bowel sounds
4. A history of diarrhea
36. A client with a spinal cord injury is prone to experiencing autonomic dysreflexia. The
nurse would avoid which of the following measures to minimize the risk of recurrence?
1. Strict adherence to a bowel retraining program
2. Limiting bladder catheterization to once every 12 hours
3. Keeping the linen wrinkle-free under the client
4. Preventing unnecessary pressure on the lower limbs
37. The nurse is planning care for the client in spinal shock. Which of the following actions
would be least helpful in minimizing the effects of vasodilation below the level of the
injury?
1. Monitoring vital signs before and during position changes
2. Using vasopressor medications as prescribed
3. Moving the client quickly as one unit
4. Applying Teds or compression stockings.
38. The nurse is caring for a client admitted with spinal cord injury. The nurse minimizes
the risk of compounding the injury most effectively by:
1. Keeping the client on a stretcher
2. Logrolling the client on a firm mattress
3. Logrolling the client on a soft mattress
4. Placing the client on a Stryker frame
39. The nurse is evaluating neurological signs of the male client in spinal shock following
spinal cord injury. Which of the following observations by the nurse indicates that spinal
shock persists?
1. Positive reflexes
2. Hyperreflexia
3. Inability to elicit a Babinskis reflex
4. Reflex emptying of the bladder

76

40. A client with a spinal cord injury suddenly experiences an episode of autonomic
dysreflexia. After checking the clients vital signs, list in order of priority, the nurses
actions (Number 1 being the first priority and number 5 being the last priority).
1. Check for bladder distention
2. Raise the head of the bed
3. Contact the physician
4. Loosen tight clothing on the client
5. Administer an antihypertensive medication
41. A client is at risk for increased ICP. Which of the following would be a priority for the
nurse to monitor?
1. Unequal pupil size
2. Decreasing systolic blood pressure
3. Tachycardia
4. Decreasing body temperature
42. Which of the following respiratory patterns indicate increasing ICP in the brain stem?
1. Slow, irregular respirations
2. Rapid, shallow respirations
3. Asymmetric chest expansion
4. Nasal flaring
43. Which of the following nursing interventions is appropriate for a client with an ICP of
20 mm Hg?
1. Give the client a warming blanket
2. Administer low-dose barbiturate
3. Encourage the client to hyperventilate
4. Restrict fluids
44. A client has signs of increased ICP. Which of the following is an early indicator of
deterioration in the clients condition?
1. Widening pulse pressure
2. Decrease in the pulse rate
3. Dilated, fixed pupil
4. Decrease in LOC
45. A client who is regaining consciousness after a craniotomy becomes restless and
attempts to pull out her IV line. Which nursing intervention protects the client without
increasing her ICP?
1. Place her in a jacket restraint
2. Wrap her hands in soft mitten restraints
3. Tuck her arms and hands under the draw sheet
4. Apply a wrist restraint to each arm
46. Which of the following describes decerebrate posturing?
1. Internal rotation and adduction of arms with flexion of elbows, wrists, and fingers
2. Back hunched over, rigid flexion of all four extremities with supination of arms and plantar flexion of
the feet
3. Supination of arms, dorsiflexion of feet
4. Back arched; rigid extension of all four extremities.
47. A client receiving vent-assisted mode ventilation begins to experience cluster
breathing after recent intracranial occipital bleeding. Which action would be most
appropriate?

77

1. Count the rate to be sure the ventilations are deep enough to be sufficient
2. Call the physician while another nurse checks the vital signs and ascertains the patients Glasgow
Coma score.
3. Call the physician to adjust the ventilator settings.
4. Check deep tendon reflexes to determine the best motor response
48. In planning the care for a client who has had a posterior fossa (infratentorial)
craniotomy, which of the following is contraindicated when positioning the client?
1. Keeping the client flat on one side or the other
2. Elevating the head of the bed to 30 degrees
3. Log rolling or turning as a unit when turning
4. Keeping the head in neutral position
49. A client has been pronounced brain dead. Which findings would the nurse assess?
Check all that apply.
1. Decerebrate posturing
2. Dilated nonreactive pupils
3. Deep tendon reflexes
4. Absent corneal reflex
50. A 23-year-old patient with a recent history of encephalitis is admitted to the medical
unit with new onset generalized tonic-clonic seizures. Which nursing activities included in
the patients care will be best to delegate to an LPN/LVN whom you are supervising?
(Choose all that apply).
1. Document the onset time, nature of seizure activity, and postictal behaviors for all seizures.
2. Administer phenytoin (Dilantin) 200 mg PO daily.
3. Teach patient about the need for good oral hygiene.
4. Develop a discharge plan, including physician visits and referral to the Epilepsy Foundation.

Answers and Rationale


1. Answer: 1. The nurse should first attempt nursing interventions, such as repositioning the client to
avoid neck flexion, which increases venous return and lowers ICP. If nursing measures prove
ineffective, notify the physician, who may prescribe mannitol, pentobarbital, or hyperventilation
therapy.
2. Answer: 2. Dilantin IV shouldnt be given at a rate exceeding 50 mg/minute. Rapid administration
can depress the myocardium, causing arrhythmias. Therapeutic drug levels range from 10 to 20
mg/ml. Dilantin shouldnt be mixed in solution for administration. However, because its compatible
with normal saline solution, it can be injected through an IV line containing normal saline. When given
through an IV catheter hand, dilantin may cause purple glove syndrome.
3. Answer: 1. Urine output of 300 ml/hr may indicate diabetes insipidus, which is a failure of the
pituitary to produce anti-diuretic hormone. This may occur with increased intracranial pressure and
head trauma; the nurse evaluates for low urine specific gravity, increased serum osmolarity, and
dehydration. Theres no evidence that the client is experiencing renal failure. Providing emollients to
prevent skin breakdown is important, but doesnt need to be performed immediately. Slowing the rate
of IV fluid would contribute to dehydration when polyuria is present.
4. Answer: 1. A normal PaCO2 value is 35 to 45 mm Hg. CO2 has vasodilating properties; therefore,
lowering PaCO2 through hyperventilation will lower ICP caused by dilated cerebral vessels.
Oxygenation is evaluated through PaO2 and oxygen saturation. Alveolar hypoventilation would be
reflected in an increased PaCO2.

78

5. Answer: 2. Frequent swallowing after brain surgery may indicate fluid or blood leaking from the
sinuses into the oropharynx. Blood or fluid draining from the ear may indicate a basilar skull fracture.
6. Answer: 4. After hypophysectomy, or removal of the pituitary gland, the body cant synthesize ADH.
Somatropin or growth hormone, not vasopressin is used to treat growth failure. SIADH results from
excessive ADH secretion. Mannitol or corticosteroids are used to decrease cerebral edema.
7. Answer: 3. All clients with a head injury are treated as if a cervical spine injury is present until x-rays
confirm their absence. ROM would be contraindicated at this time. There is no indication that the client
needs a chest x-ray. The airway doesnt need to be opened since the client appears alert and not in
respiratory distress. In addition, the head-tilt chin-lift maneuver wouldnt be used until the cervical
spine injury is ruled out.
8. Answer: 4. Tetraplegia occurs as a result of cervical spine injuries. Paraplegia occurs as a result of
injury to the thoracic cord and below.
9. Answer: 3. After a spinal cord injury, ascending cord edema may cause a higher level of injury. The
diaphragm is innervated at the level of C4, so assessment of adequate oxygenation and ventilation is
necessary. Although the other options would be necessary at a later time, observation for respiratory
failure is the priority.
10. Answer: 3. Symptoms of neurogenic shock include hypotension, bradycardia, and warm, dry skin
due to the loss of adrenergic stimulation below the level of the lesion. Hypertension, bradycardia,
flushing, and sweating of the skin are seen with autonomic dysreflexia. Hemorrhagic shock presents
with anxiety, tachycardia, and hypotension; this wouldnt be suspected without an injury. Pulmonary
embolism presents with chest pain, hypotension, hypoxemia, tachycardia, and hemoptysis; this may
be a later complication of spinal cord injury due to immobility.
11. Answer: 3. High doses of Solu-Medrol are used within 24 hours of spinal injury to reduce cord
swelling and limit neurological deficit. The other drugs arent indicated in this circumstance.
12. Answer: 4. Anxiety, flushing above the level of the lesion, piloerection, hypertension, and
bradycardia are symptoms of autonomic dysreflexia, typically caused by such noxious stimuli such as a
full bladder, fecal impaction, or decubitus ulcer. Putting the client flat will cause the blood pressure to
increase even more. The indwelling urinary catheter should be assessed immediately after the HOB is
raised. Nitroglycerin is given to reduce chest pain and reduce preload; it isnt used for hypertension or
dysreflexia.
13. Answer: 2. Gardner-Wells, Vinke, and Crutchfield tongs immobilize the spine until surgical
stabilization is accomplished.
14. Answer: 2. Intermittent catherization should begin every 2 to 4 hours early in the treatment. When
residual volume is less than 400 ml, the schedule may advance to every 4 to 6 hours. Indwelling
catheters may predispose the client to infection and are removed as soon as possible. Credes
maneuver is not used on people with spinal cord injury.
15. Answer: 1. Epidural hematoma or extradural hematoma is usually caused by laceration of the
middle meningeal artery. An embolic stroke is a thromboembolism from a carotid artery that ruptures.
Venous bleeding from the arachnoid space is usually observed with subdural hematoma.
16. Answer: 2. Clear fluid from the nose or ear can be determined to be cerebral spinal fluid or mucous
by the presence of dextrose. Placing the client flat in bed may increase ICP and promote pulmonary
aspiration. The nose wouldnt be suctioned because of the risk for suctioning brain tissue through the
sinuses. Nothing is inserted into the ears or nose of a client with a skull fracture because of the risk of
infection.
17. Answer: 3. A lucid interval is described as a brief period of unconsciousness followed by alertness;
after several hours, the client again loses consciousness. Garbled speech is known as dysarthria. An

79

interval in which the client is alert but cant recall recent events is known as amnesia. Warning
symptoms or auras typically occur before seizures.
18. Answer: 3. Autonomic dysreflexia refers to uninhibited sympathetic outflow in clients with spinal
cord injuries about the level of T10. The other clients arent prone to dysreflexia.
19. Answer: 3. Spasticity, the return of reflexes, is a sign of resolving shock. Spinal or neurogenic shock
is characterized by hypotension, bradycardia, dry skin, flaccid paralysis, or the absence of reflexes
below the level of injury. The absence of pain sensation in the chest doesnt apply to spinal shock.
Spinal shock descends from the injury, and respiratory difficulties occur at C4 and above.
20. Answer: 4. Noxious stimuli, such as a full bladder, fecal impaction, or a decub ulcer, may cause
autonomic dysreflexia. A headache is a symptom of autonomic dysreflexia, not a cause. Autonomic
dysreflexia is most commonly seen with injuries at T10 or above. Neurogenic shock isnt a cause of
dysreflexia.
21. Answer: 4. Putting the client in the high-Fowlers position will decrease cerebral blood flow,
decreasing hypertension. Elevating the clients legs, putting the client flat in bed, or putting the bed in
the Trendelenburgs position places the client in positions that improve cerebral blood flow, worsening
hypertension.
22. Answer: 3. Loss of sympathetic control and unopposed vagal stimulation below the level of injury
typically cause hypotension, bradycardia, pallor, flaccid paralysis, and warm, dry skin in the client in
neurogenic shock. Hypervolemia is indicated by rapid and bounding pulse and edema. Autonomic
dysreflexia occurs after neurogenic shock abates. Signs of sepsis would include elevated temperature,
increased heart rate, and increased respiratory rate.
23. Answer: 4. The diaphragm is stimulated by nerves at the level of C4. Initially, this client may need
mechanical ventilation due to cord edema. This may resolve in time. Absent corneal reflexes,
decerebrate posturing, and hemiplegia occur with brain injuries, not spinal cord injuries.
24. Answer: 2. Hypertension, bradycardia, anxiety, blurred vision, and flushing above the lesion occur
with autonomic dysreflexia due to uninhibited sympathetic nervous system discharge. The other
options are incorrect.
25. Answer: 4. Intermittent catheterization may be performed chronically with clean technique, using
soap and water to clean the urinary meatus. The meatus is always cleaned from front to back in a
woman, or in expanding circles working outward from the meatus in a man. It isnt necessary to
measure the urine. The catheter doesnt need to be rotated during removal.
26. Answer: 3. Changes in LOC may indicate expanding lesions such as subdural hematoma;
orientation and LOC are assessed frequently for 24 hours. A keyhole pupil is found after iridectomy.
Profuse or projectile vomiting is a symptom of increased ICP and should be reported immediately. A
slight headache may last for several days after concussion; severe or worsening headaches should be
reported.
27. Answer: 1. The frontal lobe primarily functions to regulate thinking, planning, and affect. Dopamine
is known to circulate widely throughout this lobe, which is why its such an important neurotransmitter
in schizophrenia.
28. Answer: 3. An EEG measures the electrical activity of the brain. Extent of intracranial bleeding and
location of the injury site would be determined by CT or MRI. Percent of functional brain tissue would
be determined by a series of tests.
29. Answer: 3. An epidural hematoma occurs when blood collects between the skull and the dura
mater. In a subdural hematoma, venous blood collects between the dura mater and the arachnoid
mater. In a subarachnoid hemorrhage, blood collects between the pia mater and arachnoid membrane.
A contusion is a bruise on the brains surface.

80

30. Answer: 1. A client with a spinal cord injury at levels C5 to C6 has quadriplegia with gross arm
movement and diaphragmatic breathing. Injury levels C1 to C4 leads to quadriplegia with total loss of
respiratory function. Paraplegia with intercostal muscle loss occurs with injuries at T1 to L2. Injuries
below L2 cause paraplegia and loss of bowel and bladder control.
31. Answer: 3. If the client has a suspected cervical spine injury, a jaw-thrust maneuver should be used
to open the airway. If the tongue or relaxed throat muscles are obstructing the airway, a
nasopharyngeal or oropharyngeal airway can be inserted; however, the client must have spontaneous
respirations when the airway is open. The head-tilt, chin-lift maneuver requires neck hyperextension,
which can worsen the cervical spine injury.
32. Answer: 1, 2, 4, 5. The client has signs and symptoms of autonomic dysreflexia. The potentially lifethreatening condition is caused by an uninhibited response from the sympathetic nervous system
resulting from a lack of control over the autonomic nervous system. The nurse should immediately
elevate the HOB to 90 degrees and place extremities dependently to decrease venous return to the
heart and increase venous return from the brain. Because tactile stimuli can trigger autonomic
dysreflexia, any constrictive clothing should be loosened. The nurse should also assess for distended
bladder and bowel impaction, which may trigger autonomic dysreflexia, and correct any problems.
Elevated blood pressure is the most life-threatening complication of autonomic dysreflexia because it
can cause stroke, MI, or seizures. If removing the triggering event doesnt reduce the clients blood
pressure, IV antihypertensives should be administered. A fan shouldnt be used because cold drafts
may trigger autonomic dysreflexia.
33. Answer: 1. A complication of a head injury is diabetes insipidus, which can occur with insult to the
hypothalamus, the antidiuretic storage vesicles, or the posterior pituitary gland. Urine output that
exceeds 9 L per day generally requires treatment with desmopressin. Dexamethasone, a
glucocorticoid, is administered to treat cerebral edema. This medication may be ordered for the head
injured patient. Ethacrynic acid and mannitol are diuretics, which would be contraindicated.
34. Answer: 4. The changes in neurological signs from an epidural hematoma begin with a loss of
consciousness as arterial blood collects in the epidural space and exerts pressure. The client regains
consciousness as the cerebral spinal fluid is reabsorbed rapidly to compensate for the rising
intracranial pressure. As the compensatory mechanisms fail, even small amounts of additional blood
can cause the intracranial pressure to rise rapidly, and the clients neurological status deteriorates
quickly.
35. Answer: 2. After spinal cord injury, the client can develop paralytic ileus, which is characterized by
the absence of bowel sounds and abdominal distention. Development of a stress ulcer can be detected
by hematest positive NG tube aspirate or stool. A history of diarrhea is irrelevant.
36. Answer: 2. The most frequent cause of autonomic dysreflexia is a distended bladder. Straight
catherization should be done every 4 to 6 hours, and Foley catheters should be checked frequently to
prevent kinks in the tubing. Constipation and fecal impaction are other causes, so maintaining bowel
regularity is important. Other causes include stimulation of the skin from tactile, thermal, or painful
stimuli. The nurse administers care to minimize risk in these areas.
37. Answer: 3. Reflex vasodilation below the level of the spinal cord injury places the client at risk for
orthostatic hypotension, which may be profound. Measures to minimize this include measuring vital
signs before and during position changes, use of a tilt-table with early mobilization, and changing the
clients position slowly. Venous pooling can be reduced by using Teds (compression stockings) or
pneumatic boots. Vasopressor medications are administered per protocol.
38. Answer: 4. Spinal immobilization is necessary after spinal cord injury to prevent further damage
and insult to the spinal cord. Whenever possible, the client is placed on a Stryker frame, which allows

81

the nurse to turn the client to prevent complications of immobility, while maintaining alignment of the
spine. If a Stryker frame is not available, a firm mattress with a bed board should be used.
39. Answer: 3. Resolution of spinal shock is occurring when there is a return of reflexes (especially
flexors to noxious cutaneous stimuli), a state of hyperreflexia rather than flaccidity, reflex emptying of
the bladder, and a positive Babinskis reflex.
40. Answer: 3, 1, 4, 2, 5. Autonomic dysreflexia is characterized by severe hypertension, bradycardia,
severe headache, nasal stuffiness, and flushing. The cause is a noxious stimulus, most often a
distended bladder or constipation. Autonomic dysreflexia is a neurological emergency and must be
treated promptly to prevent a hypertensive stroke. Immediate nursing actions are to sit the client up in
bed in a high-Fowlers position and remove the noxious stimulus. The nurse should loosen any tight
clothing and then check for bladder distention. If the client has a foley catheter, the nurse should
check for kinks in the tubing. The nurse also would check for a fecal impaction and disimpact if
necessary. The physician is contacted especially if these actions do not relieve the signs and
symptoms. Antihypertensive medications may be prescribed by the physician to minimize cerebral
hypertension.
41. Answer: 1. Increasing ICP causes unequal pupils as a result of pressure on the third cranial nerve.
Increasing ICP causes an increase in the systolic pressure, which reflects the additional pressure
needed to perfuse the brain. It increases the pressure on the vagus nerve, which produces
bradycardia, and it causes an increase in body temperature from hypothalamic damage.
42. Answer: 1. Neural control of respiration takes place in the brain stem. Deterioration and pressure
produce irregular respiratory patterns. Rapid, shallow respirations, asymmetric chest movements, and
nasal flaring are more characteristic of respiratory distress or hypoxia.
43. Answer: 3. Normal ICP is 15 mm Hg or less. Hyperventilation causes vasoconstriction, which
reduces CSF and blood volume, two important factors for reducing a sustained ICP of 20 mm Hg. A
cooling blanket is used to control the elevation of temperature because a fever increases the metabolic
rate, which in turn increases ICP. High doses of barbiturates may be used to reduce the increased
cellular metabolic demands. Fluid volume and inotropic drugs are used to maintain cerebral perfusion
by supporting the cardiac output and keeping the cerebral perfusion pressure greater than 80 mm Hg.
44. Answer: 4. A decrease in the clients LOC is an early indicator of deterioration of the clients
neurological status. Changes in LOC, such as restlessness and irritability, may be subtle. Widening of
the pulse pressure, decrease in the pulse rate, and dilated, fixed pupils occur later if the increased ICP
is not treated.
45. Answer: 2. It is best for the client to wear mitts which help prevent the client from pulling on the IV
without causing additional agitation. Using a jacket or wrist restraint or tucking the clients arms and
hands under the draw sheet restrict movement and add to feelings of being confined, all of which
would increase her agitation and increase ICP.
46. Answer: 4. Decerebrate posturing occurs in patients with damage to the upper brain stem,
midbrain, or pons and is demonstrated clinically by arching of the back, rigid extension of the
extremities, pronation of the arms, and plantar flexion of the feet. Internal rotation and adduction of
arms with flexion of the elbows, wrists, and fingers described decorticate posturing, which indicates
damage to corticospinal tracts and cerebral hemispheres.
47. Answer: 2. Cluster breathing consists of clusters of irregular breaths followed by periods of apnea
on an irregular basis. A lesion in the upper medulla or lower pons is usually the cause of cluster
breathing. Because the client had a bleed in the occipital lobe, which is superior and posterior to the
pons and medulla, clinical manifestations that indicate a new lesion are monitored very closely in case
another bleed ensues. The physician is notified immediately so that treatment can begin before

82

respirations cease. Another nurse needs to assess vital signs and score the client according to the
GCS, but time is also of the essence. Checking deep tendon reflexes is one part of the GCS analysis.
48. Answer: 2. Elevating the HOB to 30 degrees is contraindicated for infratentorial craniotomies
because it could cause herniation of the brain down onto the brainstem and spinal cord, resulting in
sudden death. Elevation of the head of the bed to 30 degrees with the head turned to the side
opposite of the incision, if not contraindicated by the ICP; is used for supratentorial craniotomies.
49. Answer: 2, 3, 4. A client who is brain dead typically demonstrates nonreactive dilated pupils and
nonreactive or absent corneal and gag reflexes. The client may still have spinal reflexes such as deep
tendon and Babinski reflexes in brain death. Decerebrate or decorticate posturing would not be seen.
50. Answer: B Administration of medications is included in LPN education and scope of practice.
Collection of data about the seizure activity may be accomplished by an LPN/LVN who observes initial
seizure activity. An LPN/LVN would know to call the supervising RN immediately if a patient started to
seize. Documentation of the seizure, patient teaching, and planning of care are complex activities that
require RN level education and scope of practice.

Introduction
A 10-item examination about Neurological Disorders. The questions in this exam are a perfect for your
review for the NCLEX.

Topics

Increased Intracranial Pressure

Level of Consciousness

Meningitis

Guidelines

Read each question carefully and choose the best answer.

You are given one minute per question. Spend your time wisely!

Answers and rationales are given below. Be sure to read them.

If you need more clarifications, please direct them to the comments section.

Questions
1. Regular oral hygiene is an essential intervention for the client who has had a stroke.
Which of the following nursing measures is inappropriate when providing oral hygiene?
1. Placing the client on the back with a small pillow under the head.
2. Keeping portable suctioning equipment at the bedside.
3. Opening the clients mouth with a padded tongue blade.
4. Cleaning the clients mouth and teeth with a toothbrush.
2. A 78 year old client is admitted to the emergency department with numbness and
weakness of the left arm and slurred speech. Which nursing intervention is priority?
1. Prepare to administer recombinant tissue plasminogen activator (rt-PA).
2. Discuss the precipitating factors that caused the symptoms.

83

3. Schedule for A STAT computer tomography (CT) scan of the head.


4. Notify the speech pathologist for an emergency consult.
3. A client arrives in the emergency department with an ischemic stroke and receives
tissue plasminogen activator (t-PA) administration. Which is the priority nursing
assessment?
1. Current medications.
2. Complete physical and history.
3. Time of onset of current stroke.
4. Upcoming surgical procedures.
4. During the first 24 hours after thrombolytic therapy for ischemic stroke, the primary
goal is to control the clients:
1. Pulse
2. Respirations
3. Blood pressure
4. Temperature
5. What is a priority nursing assessment in the first 24 hours after admission of the client
with a thrombotic stroke?
1. Cholesterol level
2. Pupil size and pupillary response
3. Vowel sounds
4. Echocardiogram
6. What is the expected outcome of thrombolytic drug therapy?
1. Increased vascular permeability.
2. Vasoconstriction.
3. Dissolved emboli.
4. Prevention of hemorrhage
7. The client diagnosed with atrial fibrillation has experienced a transient ischemic attack
(TIA). Which medication would the nurse anticipate being ordered for the client on
discharge?
1. An oral anticoagulant medication.
2. A beta-blocker medication.
3. An anti-hyperuricemic medication.
4. A thrombolytic medication.
8. Which client would the nurse identify as being most at risk for experiencing a CVA?
1. A 55-year-old African American male.
2. An 84-year-old Japanese female.
3. A 67-year-old Caucasian male.
4. A 39-year-old pregnant female.
9. Which assessment data would indicate to the nurse that the client would be at risk for a
hemorrhagic stroke?
1. A blood glucose level of 480 mg/dl.
2. A right-sided carotid bruit.
3. A blood pressure of 220/120 mmHg.
4. The presence of bronchogenic carcinoma.
10. The nurse and unlicensed assistive personnel (UAP) are caring for a client with rightsided paralysis. Which action by the UAP requires the nurse to intervene?

84

1. The assistant places a gait belt around the clients waist prior to ambulating.
2. The assistant places the client on the back with the clients head to the side.
3. The assistant places her hand under the clients right axilla to help him/her move up in bed.
4. The assistant praises the client for attempting to perform ADLs independently.

Answers and Rationale


1. Answer: 1. A helpless client should be positioned on the side, not on the back. This lateral position
helps secretions escape from the throat and mouth, minimizing the risk of aspiration. It may be
necessary to suction, so having suction equipment at the bedside is necessary. Padded tongue blades
are safe to use. A toothbrush is appropriate to use.
2. Answer: 3. A CT scan will determine if the client is having a stroke or has a brain tumor or another
neurological disorder. This would also determine if it is a hemorrhagic or ischemic accident and guide
the treatment, because only an ischemic stroke can use rt-PA. This would make (1) not the priority
since if a stroke was determined to be hemorrhagic, rt-PA is contraindicated. Discuss the precipitating
factors for teaching would not be a priority and slurred speech would as indicate interference for
teaching. Referring the client for speech therapy would be an intervention after the CVA emergency
treatment is administered according to protocol.
3. Answer: 3. The time of onset of a stroke to t-PA administration is critical. Administration within 3
hours has better outcomes. A complete history is not possible in emergency care. Upcoming surgical
procedures will need to be delay if t-PA is administered. Current medications are relevant, but onset of
current stroke takes priority.
4. Answer: 3. Controlling the blood pressure is critical because an intracerebral hemorrhage is the
major adverse effect of thrombolytic therapy. Blood pressure should be maintained according to
physician and is specific to the clients ischemic tissue needs and risks of bleeding from treatment.
Other vital signs are monitored, but the priority is blood pressure.
5. Answer: 2. It is crucial to monitor the pupil size and pupillary response to indicate changes around
the cranial nerves. Cholesterol level is an assessment to be addressed for long-term healthy lifestyle
rehabilitation. Bowel sounds need to be assessed because an ileus or constipation can develop, but is
not a priority in the first 24 hours. An echocardiogram is not needed for the client with a thrombotic
stroke.
6. Answer: 3. Thrombolytic therapy is use to dissolve emboli and reestablish cerebral perfusion.
7. Answer: 1. Thrombi form secondary to atrial fibrillation, therefore, an anticoagulant would be
anticipated to prevent thrombi formation; and oral (warfarin [Coumadin]) at discharge verses
intravenous. Beta blockers slow the heart rate and lower the blood pressure. Anti-hyperuricemic
medication is given to clients with gout. Thrombolytic medication might have been given at initial
presentation but would not be a drug prescribed at discharge.
8. Answer: 1. Africana Americans have twice the rate of CVAs as Caucasians; males are more likely to
have strokes than females except in advanced years. Orientals have a lower risk, possibly due to their
high omega-3 fatty acids. Pregnancy is a minimal risk factor for CVA.
9. Answer: 3. Uncontrolled hypertension is a risk factor for hemorrhagic stroke, which is a rupture blood
vessel in the cranium. A bruit in the carotid artery would predispose a client to an embolic or ischemic
stroke. High blood glucose levels could predispose a patient to ischemic stroke, but not hemorrhagic.
Cancer is not a precursor to stroke.
10. Answer: 3. This action is inappropriate and would require intervention by the nurse because pulling
on a flaccid shoulder joint could cause shoulder dislocation; as always use a lift sheet for the client and
nurse safety. All the other actions are appropriate.

85

Introduction
A 60-item NCLEX-style examination all about Obstetrical Nursing. This exam has questions that
will help you prepare for your boards or NCLEX.

Topics

Labor and Delivery

Fetal Heart Rate


Questions about oxytocin
Disseminated Intravascular Coagulation (DIC)
Leopolds Maneuver

Guidelines

Read each question carefully and choose the best answer.

You are given one minute per question. Spend your time wisely!
Answers and rationales are given below. Be sure to read them.
If you need more clarifications, please direct them to the comments section.

Questions
1. A nurse is caring for a client in labor. The nurse determines that the client is beginning in
the 2nd stage of labor when which of the following assessments is noted?
1. The client begins to expel clear vaginal fluid
2. The contractions are regular
3. The membranes have ruptured
4. The cervix is dilated completely
2. A nurse in the labor room is caring for a client in the active phases of labor. The nurse is
assessing the fetal patterns and notes a late deceleration on the monitor strip. The most
appropriate nursing action is to:
1. Place the mother in the supine position
2. Document the findings and continue to monitor the fetal patterns
86

3. Administer oxygen via face mask


4. Increase the rate of pitocin IV infusion
3. A nurse is performing an assessment of a client who is scheduled for a cesarean delivery.
Which assessment finding would indicate a need to contact the physician?
1. Fetal heart rate of 180 beats per minute
2. White blood cell count of 12,000
3. Maternal pulse rate of 85 beats per minute
4. Hemoglobin of 11.0 g/dL
4. A client in labor is transported to the delivery room and is prepared for a cesarean
delivery. The client is transferred to the delivery room table, and the nurse places the client
in the:
1. Trendelenburgs position with the legs in stirrups
2. Semi-Fowler position with a pillow under the knees
3. Prone position with the legs separated and elevated
4. Supine position with a wedge under the right hip
5. A nurse is caring for a client in labor and prepares to auscultate the fetal heart rate by
using a Doppler ultrasound device. The nurse most accurately determines that the fetal
heart sounds are heard by:
1. Noting if the heart rate is greater than 140 BPM
2. Placing the diaphragm of the Doppler on the mother abdomen
3. Performing Leopolds maneuvers first to determine the location of the fetal heart
4. Palpating the maternal radial pulse while listening to the fetal heart rate
6. A nurse is caring for a client in labor who is receiving Pitocin by IV infusion to stimulate
uterine contractions. Which assessment finding would indicate to the nurse that the
infusion needs to be discontinued?
1. Three contractions occurring within a 10-minute period
2. A fetal heart rate of 90 beats per minute
3. Adequate resting tone of the uterus palpated between contractions
4. Increased urinary output
7. A nurse is beginning to care for a client in labor. The physician has prescribed an IV
infusion of Pitocin. The nurse ensures that which of the following is implemented before
initiating the infusion?
87

1. Placing the client on complete bed rest


2. Continuous electronic fetal monitoring
3. An IV infusion of antibiotics
4. Placing a code cart at the clients bedside
8. A nurse is monitoring a client in active labor and notes that the client is having
contractions every 3 minutes that last 45 seconds. The nurse notes that the fetal heart rate
between contractions is 100 BPM. Which of the following nursing actions is most
appropriate?
1. Encourage the clients coach to continue to encourage breathing exercises
2. Encourage the client to continue pushing with each contraction
3. Continue monitoring the fetal heart rate
4. Notify the physician or nurse midwife
9. A nurse is caring for a client in labor and is monitoring the fetal heart rate patterns. The
nurse notes the presence of episodic accelerations on the electronic fetal monitor tracing.
Which of the following actions is most appropriate?
1. Document the findings and tell the mother that the monitor indicates fetal well-being
2. Take the mothers vital signs and tell the mother that bed rest is required to conserve oxygen.
3. Notify the physician or nurse midwife of the findings.
4. Reposition the mother and check the monitor for changes in the fetal tracing
10. A nurse is admitting a pregnant client to the labor room and attaches an external
electronic fetal monitor to the clients abdomen. After attachment of the monitor, the initial
nursing assessment is which of the following?
1. Identifying the types of accelerations
2. Assessing the baseline fetal heart rate
3. Determining the frequency of the contractions
4. Determining the intensity of the contractions
11. A nurse is reviewing the record of a client in the labor room and notes that the nurse
midwife has documented that the fetus is at (-1) station. The nurse determines that the fetal
presenting part is:
1. 1 cm above the ischial spine
2. 1 fingerbreadth below the symphysis pubis
3. 1 inch below the coccyx
4. 1 inch below the iliac crest
88

12. A pregnant client is admitted to the labor room. An assessment is performed, and the
nurse notes that the clients hemoglobin and hematocrit levels are low, indicating anemia.
The nurse determines that the client is at risk for which of the following?
1. A loud mouth
2. Low self-esteem
3. Hemorrhage
4. Postpartum infections
13. A nurse assists in the vaginal delivery of a newborn infant. After the delivery, the nurse
observes the umbilical cord lengthen and a spurt of blood from the vagina. The nurse
documents these observations as signs of:
1. Hematoma
2. Placenta previa
3. Uterine atony
4. Placental separation
14. A client arrives at a birthing center in active labor. Her membranes are still intact, and
the nurse-midwife prepares to perform an amniotomy. A nurse who is assisting the nursemidwife explains to the client that after this procedure, she will most likely have:
1. Less pressure on her cervix
2. Increased efficiency of contractions
3. Decreased number of contractions
4. The need for increased maternal blood pressure monitoring
15. A nurse is monitoring a client in labor. The nurse suspects umbilical cord compression
if which of the following is noted on the external monitor tracing during a contraction?
1. Early decelerations
2. Variable decelerations
3. Late decelerations
4. Short-term variability
16. A nurse explains the purpose of effleurage to a client in early labor. The nurse tells the
client that effleurage is:
1. A form of biofeedback to enhance bearing down efforts during delivery
2. Light stroking of the abdomen to facilitate relaxation during labor and provide tactile
stimulation to the fetus
3. The application of pressure to the sacrum to relieve a backache
89

4. Performed to stimulate uterine activity by contracting a specific muscle group while other
parts of the body rest
17. A nurse is caring for a client in the second stage of labor. The client is experiencing
uterine contractions every 2 minutes and cries out in pain with each contraction. The nurse
recognizes this behavior as:
1. Exhaustion
2. Fear of losing control
3. Involuntary grunting
4. Valsalvas maneuver
18. A nurse is monitoring a client in labor who is receiving Pitocin and notes that the client
is experiencing hypertonic uterine contractions. List in order of priority the actions that the
nurse takes.
1. Stop of Pitocin infusion
2. Perform a vaginal examination
3. Reposition the client
4. Check the clients blood pressure and heart rate
5. Administer oxygen by face mask at 8 to 10 L/min
19. A nurse is assigned to care for a client with hypotonic uterine dysfunction and signs of a
slowing labor. The nurse is reviewing the physicians orders and would expect to note
which of the following prescribed treatments for this condition?
1. Medication that will provide sedation
2. Increased hydration
3. Oxytocin (Pitocin) infusion
4. Administration of a tocolytic medication
20. A nurse in the labor room is preparing to care for a client with hypertonic uterine
dysfunction. The nurse is told that the client is experiencing uncoordinated contractions
that are erratic in their frequency, duration, and intensity. The priority nursing
intervention would be to:
1. Monitor the Pitocin infusion closely
2. Provide pain relief measures
3. Prepare the client for an amniotomy
4. Promote ambulation every 30 minutes

90

21. A nurse is developing a plan of care for a client experiencing dystocia and includes
several nursing interventions in the plan of care. The nurse prioritizes the plan of care and
selects which of the following nursing interventions as the highest priority?
1. Keeping the significant other informed of the progress of the labor
2. Providing comfort measures
3. Monitoring fetal heart rate
4. Changing the clients position frequently
22. A maternity nurse is preparing to care for a pregnant client in labor who will be
delivering twins. The nurse monitors the fetal heart rates by placing the external fetal
monitor:
1. Over the fetus that is most anterior to the mothers abdomen
2. Over the fetus that is most posterior to the mothers abdomen
3. So that each fetal heart rate is monitored separately
4. So that one fetus is monitored for a 15-minute period followed by a 15 minute fetal monitoring
period for the second fetus
23. A nurse in the postpartum unit is caring for a client who has just delivered a newborn
infant following a pregnancy with placenta previa. The nurse reviews the plan of care and
prepares to monitor the client for which of the following risks associated with placenta
previa?
1. Disseminated intravascular coagulation
2. Chronic hypertension
3. Infection
4. Hemorrhage
24. A nurse in the delivery room is assisting with the delivery of a newborn infant. After the
delivery of the newborn, the nurse assists in delivering the placenta. Which observation
would indicate that the placenta has separated from the uterine wall and is ready for
delivery?
1. The umbilical cord shortens in length and changes in color
2. A soft and boggy uterus
3. Maternal complaints of severe uterine cramping
4. Changes in the shape of the uterus

91

25. A nurse in the labor room is performing a vaginal assessment on a pregnant client in
labor. The nurse notes the presence of the umbilical cord protruding from the vagina.
Which of the following would be the initial nursing action?
1. Place the client in Trendelenburgs position
2. Call the delivery room to notify the staff that the client will be transported immediately
3. Gently push the cord into the vagina
4. Find the closest telephone and stat page the physician
26. A maternity nurse is caring for a client with abruptio placenta and is monitoring the
client for disseminated intravascular coagulopathy. Which assessment finding is least likely
to be associated with disseminated intravascular coagulation?
1. Swelling of the calf in one leg
2. Prolonged clotting times
3. Decreased platelet count
4. Petechiae, oozing from injection sites, and hematuria
27. A nurse is assessing a pregnant client in the 2nd trimester of pregnancy who was
admitted to the maternity unit with a suspected diagnosis of abruptio placentae. Which of
the following assessment findings would the nurse expect to note if this condition is
present?
1. Absence of abdominal pain
2. A soft abdomen
3. Uterine tenderness/pain
4. Painless, bright red vaginal bleeding
28. A maternity nurse is preparing for the admission of a client in the 3rd trimester of
pregnancy that is experiencing vaginal bleeding and has a suspected diagnosis of placenta
previa. The nurse reviews the physicians orders and would question which order?
1. Prepare the client for an ultrasound
2. Obtain equipment for external electronic fetal heart monitoring
3. Obtain equipment for a manual pelvic examination
4. Prepare to draw a Hgb and Hct blood sample
29. An ultrasound is performed on a client at term gestation that is experiencing moderate
vaginal bleeding. The results of the ultrasound indicate that an abruptio placenta is
present. Based on these findings, the nurse would prepare the client for:

92

1. Complete bed rest for the remainder of the pregnancy


2. Delivery of the fetus
3. Strict monitoring of intake and output
4. The need for weekly monitoring of coagulation studies until the time of delivery
30. A nurse in a labor room is assisting with the vaginal delivery of a newborn infant. The
nurse would monitor the client closely for the risk of uterine rupture if which of the
following occurred?
1. Hypotonic contractions
2. Forceps delivery
3. Schultz delivery
4. Weak bearing down efforts
31. A client is admitted to the birthing suite in early active labor. The priority nursing
intervention on admission of this client would be:
1. Auscultating the fetal heart
2. Taking an obstetric history
3. Asking the client when she last ate
4. Ascertaining whether the membranes were ruptured
32. A client who is gravida 1, para 0 is admitted in labor. Her cervix is 100% effaced, and
she is dilated to 3 cm. Her fetus is at +1 station. The nurse is aware that the fetus head is:
1. Not yet engaged
2. Entering the pelvic inlet
3. Below the ischial spines
4. Visible at the vaginal opening
33. After doing Leopolds maneuvers, the nurse determines that the fetus is in the ROP
position. To best auscultate the fetal heart tones, the Doppler is placed:
1. Above the umbilicus at the midline
2. Above the umbilicus on the left side
3. Below the umbilicus on the right side
4. Below the umbilicus near the left groin
34. The physician asks the nurse the frequency of a laboring clients contractions. The
nurse assesses the clients contractions by timing from the beginning of one contraction:

93

1. Until the time it is completely over


2. To the end of a second contraction
3. To the beginning of the next contraction
4. Until the time that the uterus becomes very firm
35. The nurse observes the clients amniotic fluid and decides that it appears normal,
because it is:
1. Clear and dark amber in color
2. Milky, greenish yellow, containing shreds of mucus
3. Clear, almost colorless, and containing little white specks
4. Cloudy, greenish-yellow, and containing little white specks
36. At 38 weeks gestation, a client is having late decelerations. The fetal pulse oximeter
shows 75% to 85%. The nurse should:
1. Discontinue the catheter, if the reading is not above 80%
2. Discontinue the catheter, if the reading does not go below 30%
3. Advance the catheter until the reading is above 90% and continue monitoring
4. Reposition the catheter, recheck the reading, and if it is 55%, keep monitoring
37. When examining the fetal monitor strip after rupture of the membranes in a laboring
client, the nurse notes variable decelerations in the fetal heart rate. The nurse should:
1. Stop the oxytocin infusion
2. Change the clients position
3. Prepare for immediate delivery
4. Take the clients blood pressure
38. When monitoring the fetal heart rate of a client in labor, the nurse identifies an
elevation of 15 beats above the baseline rate of 135 beats per minute lasting for 15 seconds.
This should be documented as:
1. An acceleration
2. An early elevation
3. A sonographic motion
4. A tachycardic heart rate
39. A laboring client complains of low back pain. The nurse replies that this pain occurs
most when the position of the fetus is:

94

1. Breech
2. Transverse
3. Occiput anterior
4. Occiput posterior
40. The breathing technique that the mother should be instructed to use as the fetus head
is crowning is:
1. Blowing
2. Slow chest
3. Shallow
4. Accelerated-decelerated
41. During the period of induction of labor, a client should be observed carefully for signs
of:
1. Severe pain
2. Uterine tetany
3. Hypoglycemia
4. Umbilical cord prolapse
42. A client arrives at the hospital in the second stage of labor. The fetus head is crowning,
the client is bearing down, and the birth appears imminent. The nurse should:
1. Transfer her immediately by stretcher to the birthing unit
2. Tell her to breathe through her mouth and not to bear down
3. Instruct the client to pant during contractions and to breathe through her mouth
4. Support the perineum with the hand to prevent tearing and tell the client to pant
43. A laboring client is to have a pudendal block. The nurse plans to tell the client that once
the block is working she:
1. Will not feel the episiotomy
2. May lose bladder sensation
3. May lose the ability to push
4. Will no longer feel contractions
44. Which of the following observations indicates fetal distress?
1. Fetal scalp pH of 7.14
2. Fetal heart rate of 144 beats/minute
95

3. Acceleration of fetal heart rate with contractions


4. Presence of long term variability
45. Which of the following fetal positions is most favorable for birth?
1. Vertex presentation
2. Transverse lie
3. Frank breech presentation
4. Posterior position of the fetal head
46. A laboring client has external electronic fetal monitoring in place. Which of the
following assessment data can be determined by examining the fetal heart rate strip
produced by the external electronic fetal monitor?
1. Gender of the fetus
2. Fetal position
3. Labor progress
4. Oxygenation
47. A laboring client is in the first stage of labor and has progressed from 4 to 7 cm in
cervical dilation. In which of the following phases of the first stage does cervical dilation
occur most rapidly?
1. Preparatory phase
2. Latent phase
3. Active phase
4. Transition phase
48. A multiparous client who has been in labor for 2 hours states that she feels the urge to
move her bowels. How should the nurse respond?
1. Let the client get up to use the potty
2. Allow the client to use a bedpan
3. Perform a pelvic examination
4. Check the fetal heart rate
49. Labor is a series of events affected by the coordination of the five essential factors. One
of these is the passenger (fetus). Which are the other four factors?
1. Contractions, passageway, placental position and function, pattern of care
2. Contractions, maternal response, placental position, psychological response
96

3. Passageway, contractions, placental position and function, psychological response


4. Passageway, placental position and function, paternal response, psychological response
50. Fetal presentation refers to which of the following descriptions?
1. Fetal body part that enters the maternal pelvis first
2. Relationship of the presenting part to the maternal pelvis
3. Relationship of the long axis of the fetus to the long axis of the mother
4. A classification according to the fetal part
51. A client is admitted to the L & D suite at 36 weeks gestation. She has a history of Csection and complains of severe abdominal pain that started less than 1 hour earlier. When
the nurse palpates tetanic contractions, the client again complains of severe pain. After the
client vomits, she states that the pain is better and then passes out. Which is the probable
cause of her signs and symptoms?
1. Hysteria compounded by the flu
2. Placental abruption
3. Uterine rupture
4. Dysfunctional labor
52. Upon completion of a vaginal examination on a laboring woman, the nurse records:
50%, 6 cm, -1. Which of the following is a correct interpretation of the data?
1. Fetal presenting part is 1 cm above the ischial spines
2. Effacement is 4 cm from completion
3. Dilation is 50% completed
4. Fetus has achieved passage through the ischial spines
53. Which of the following findings meets the criteria of a reassuring FHR pattern?
1. FHR does not change as a result of fetal activity
2. Average baseline rate ranges between 100 140 BPM
3. Mild late deceleration patterns occur with some contractions
4. Variability averages between 6 10 BPM
54. Late deceleration patterns are noted when assessing the monitor tracing of a woman
whose labor is being induced with an infusion of Pitocin. The woman is in a side-lying
position, and her vital signs are stable and fall within a normal range. Contractions are
intense, last 90 seconds, and occur every 1 1/2 to 2 minutes. The nurses immediate action
would be to:
97

1. Change the womans position


2. Stop the Pitocin
3. Elevate the womans legs
4. Administer oxygen via a tight mask at 8 to 10 liters/minute
55. The nurse should realize that the most common and potentially harmful maternal
complication of epidural anesthesia would be:
1. Severe postpartum headache
2. Limited perception of bladder fullness
3. Increase in respiratory rate
4. Hypotension
56. Perineal care is an important infection control measure. When evaluating a
postpartum womans perineal care technique, the nurse would recognize the need for
further instruction if the woman:
1. Uses soap and warm water to wash the vulva and perineum
2. Washes from symphysis pubis back to episiotomy
3. Changes her perineal pad every 2 3 hours
4. Uses the peribottle to rinse upward into her vagina
57. Which measure would be least effective in preventing postpartum hemorrhage?
1. Administer Methergine 0.2 mg every 6 hours for 4 doses as ordered
2. Encourage the woman to void every 2 hours
3. Massage the fundus every hour for the first 24 hours following birth
4. Teach the woman the importance of rest and nutrition to enhance healing
58. When making a visit to the home of a postpartum woman one week after birth, the
nurse should recognize that the woman would characteristically:
1. Express a strong need to review events and her behavior during the process of labor and birth
2. Exhibit a reduced attention span, limiting readiness to learn
3. Vacillate between the desire to have her own nurturing needs met and the need to take charge
of her own care and that of her newborn
4. Have reestablished her role as a spouse/partner
59. Four hours after a difficult labor and birth, a primiparous woman refuses to feed her
baby, stating that she is too tired and just wants to sleep. The nurse should:

98

1. Tell the woman she can rest after she feeds her baby
2. Recognize this as a behavior of the taking-hold stage
3. Record the behavior as ineffective maternal-newborn attachment
4. Take the baby back to the nursery, reassuring the woman that her rest is a priority at this time
60. Parents can facilitate the adjustment of their other children to a new baby by:
1. Having the children choose or make a gift to give to the new baby upon its arrival home
2. Emphasizing activities that keep the new baby and other children together
3. Having the mother carry the new baby into the home so she can show the other children the
new baby
4. Reducing stress on other children by limiting their involvement in the care of the new baby

Answers and Rationale


Gauge your performance by counter checking your answers to the answers below. Learn more
about the question by reading the rationale. If you have any disputes or questions, please direct
them to the comments section.
1. Answer: 4. The cervix is dilated completely. The second stage of labor begins when the
cervix is dilated completely and ends with the birth of the neonate.
2. Answer: 3. Administer oxygen via face mask. Late decelerations are due to uteroplacental
insufficiency as the result of decreased blood flow and oxygen to the fetus during the uterine
contractions. This causes hypoxemia; therefore oxygen is necessary. The supine position is
avoided because it decreases uterine blood flow to the fetus. The client should be turned to her
side to displace pressure of the gravid uterus on the inferior vena cava. An intravenous pitocin
infusion is discontinued when a late deceleration is noted.
3. Answer: 1. Fetal heart rate of 180 beats per minute. A normal fetal heart rate is 120-160
beats per minute. A count of 180 beats per minute could indicate fetal distress and would warrant
physician notification. By full term, a normal maternal hemoglobin range is 11-13 g/dL as a
result of the hemodilution caused by an increase in plasma volume during pregnancy.
4. Answer: 4. Supine position with a wedge under the right hip. Vena cava and descending
aorta compression by the pregnant uterus impedes blood return from the lower trunk and
extremities. This leads to decreasing cardiac return, cardiac output, and blood flow to the uterus
and the fetus. The best position to prevent this would be side-lying with the uterus displaced off
of abdominal vessels. Positioning for abdominal surgery necessitates a supine position; however,
a wedge placed under the right hip provides displacement of the uterus.

99

5. Answer: 4. Palpating the maternal radial pulse while listening to the fetal heart rate. The
nurse simultaneously should palpate the maternal radial or carotid pulse and auscultate the fetal
heart rate to differentiate the two. If the fetal and maternal heart rates are similar, the nurse may
mistake the maternal heart rate for the fetal heart rate. Leopolds maneuvers may help the
examiner locate the position of the fetus but will not ensure a distinction between the two rates.
6. Answer: 2. A fetal heart rate of 90 beats per minute. A normal fetal heart rate is 120-160
BPM. Bradycardia or late or variable decelerations indicate fetal distress and the need to
discontinue to pitocin. The goal of labor augmentation is to achieve three good-quality
contractions in a 10-minute period.
7. Answer: 2. Continuous electronic fetal monitoring. Continuous electronic fetal monitoring
should be implemented during an IV infusion of Pitocin.
8. Answer: 4. Notify the physician or nurse midwife. A normal fetal heart rate is 120-160 beats
per minute. Fetal bradycardia between contractions may indicate the need for immediate medical
management, and the physician or nurse midwife needs to be notified.
9. Answer: 1. Document the findings and tell the mother that the monitor indicates fetal
well-being. Accelerations are transient increases in the fetal heart rate that often accompany
contractions or are caused by fetal movement. Episodic accelerations are thought to be a sign of
fetal-well being and adequate oxygen reserve.
10. Answer: 2. Assessing the baseline fetal heart rate. Assessing the baseline fetal heart rate is
important so that abnormal variations of the baseline rate will be identified if they occur. Options
1 and 3 are important to assess, but not as the first priority.
11. Answer: 1. 1 cm above the ischial spine. Station is the relationship of the presenting part to
an imaginary line drawn between the ischial spines, is measured in centimeters, and is noted as a
negative number above the line and a positive number below the line. At -1 station, the fetal
presenting part is 1 cm above the ischial spines.
12. Answer: 4. Postpartum infections. Anemic women have a greater likelihood of cardiac
decompensation during labor, postpartum infection, and poor wound healing. Anemia does not
specifically present a risk for hemorrhage. Having a loud mouth is only related to the person
typing up this test.
13. Answer: 4. Placental separation. As the placenta separates, it settles downward into the
lower uterine segment. The umbilical cord lengthens, and a sudden trickle or spurt of blood
appears.

100

14. Answer: 2. Increased efficiency of contractions. Amniotomy can be used to induce labor
when the condition of the cervix is favorable (ripe) or to augment labor if the process begins to
slow. Rupturing of membranes allows the fetal head to contact the cervix more directly and may
increase the efficiency of contractions.
15. Answer: 2. Variable decelerations. Variable decelerations occur if the umbilical cord
becomes compressed, thus reducing blood flow between the placenta and the fetus. Early
decelerations result from pressure on the fetal head during a contraction. Late decelerations are
an ominous pattern in labor because it suggests uteroplacental insufficiency during a contraction.
Short-term variability refers to the beat-to-beat range in the fetal heart rate.
16. Answer: 2. Light stroking of the abdomen to facilitate relaxation during labor and
provide tactile stimulation to the fetus. Effleurage is a specific type of cutaneous stimulation
involving light stroking of the abdomen and is used before transition to promote relaxation and
relieve mild to moderate pain. Effleurage provides tactile stimulation to the fetus.
17. Answer: 2. Fear of losing control. Pains, helplessness, panicking, and fear of losing control
are possible behaviors in the 2nd stage of labor.
18. Answer: 1, 4, 2. 5, 3. If uterine hypertonicity occurs, the nurse immediately would intervene
to reduce uterine activity and increase fetal oxygenation. The nurse would stop the Pitocin
infusion and increase the rate of the nonadditive solution, check maternal BP for hyper or
hypotension, position the woman in a side-lying position, and administer oxygen by snug face
mask at 8-10 L/min. The nurse then would attempt to determine the cause of the uterine
hypertonicity and perform a vaginal exam to check for prolapsed cord.
19. Answer: 3. Oxytocin (Pitocin) infusion. Therapeutic management for hypotonic uterine
dysfunction includes oxytocin augmentation and amniotomy to stimulate a labor that slows.
20. Answer: 2. Provide pain relief measures. Management of hypertonic labor depends on the
cause. Relief of pain is the primary intervention to promote a normal labor pattern.
21. Answer: 3. Monitoring fetal heart rate. The priority is to monitor the fetal heart rate.
22. Answer: 3. So that each fetal heart rate is monitored separately. In a client with a multifetal pregnancy, each fetal heart rate is monitored separately.
23. Answer: 4. Hemorrhage. Because the placenta is implanted in the lower uterine segment,
which does not contain the same intertwining musculature as the fundus of the uterus, this site is
more prone to bleeding.

101

24. Answer: 4. Changes in the shape of the uterus. Signs of placental separation include
lengthening of the umbilical cord, a sudden gush of dark blood from the introitus (vagina), a
firmly contracted uterus, and the uterus changing from a discoid (like a disk) to a globular (like a
globe) shape. The client may experience vaginal fullness, but not severe uterine cramping.
25. Answer: 1. Place the client in Trendelenburgs position. When cord prolapse occurs,
prompt actions are taken to relieve cord compression and increase fetal oxygenation. The mother
should be positioned with the hips higher than the head to shift the fetal presenting part toward
the diaphragm. The nurse should push the call light to summon help, and other staff members
should call the physician and notify the delivery room. No attempt should be made to replace the
cord. The examiner, however, may place a gloved hand into the vagina and hold the presenting
part off of the umbilical cord. Oxygen at 8 to 10 L/min by face mask is delivered to the mother to
increase fetal oxygenation.
26. Answer: 1. Swelling of the calf in one leg. DIC is a state of diffuse clotting in which
clotting factors are consumed, leading to widespread bleeding. Platelets are decreased because
they are consumed by the process; coagulation studies show no clot formation (and are thus
normal to prolonged); and fibrin plugs may clog the microvasculature diffusely, rather than in an
isolated area. The presence of petechiae, oozing from injection sites, and hematuria are signs
associated with DIC. Swelling and pain in the calf of one leg are more likely to be associated
with thrombophlebitis.
27. 3. Uterine tenderness/pain. In abruptio placentae, acute abdominal pain is present. Uterine
tenderness and pain accompanies placental abruption, especially with a central abruption and
trapped blood behind the placenta. The abdomen will feel hard and boardlike on palpation as the
blood penetrates the myometrium and causes uterine irritability. Observation of the fetal
monitoring often reveals increased uterine resting tone, caused by failure of the uterus to relax in
attempt to constrict blood vessels and control bleeding.
28. Answer: 3. Obtain equipment for a manual pelvic examination. Manual pelvic
examinations are contraindicated when vaginal bleeding is apparent in the 3rd trimester until a
diagnosis is made and placental previa is ruled out. Digital examination of the cervix can lead to
maternal and fetal hemorrhage. A diagnosis of placenta previa is made by ultrasound. The H/H
levels are monitored, and external electronic fetal heart rate monitoring is initiated. External fetal
monitoring is crucial in evaluating the fetus that is at risk for severe hypoxia.
29. Answer: 2. Delivery of the fetus. The goal of management in abruptio placentae is to
control the hemorrhage and deliver the fetus as soon as possible. Delivery is the treatment of
choice if the fetus is at term gestation or if the bleeding is moderate to severe and the mother or
fetus is in jeopardy.

102

30. Answer: 2. Forceps delivery. Excessive fundal pressure, forceps delivery, violent bearing
down efforts, tumultuous labor, and shoulder dystocia can place a woman at risk for traumatic
uterine rupture. Hypotonic contractions and weak bearing down efforts do not alone add to the
risk of rupture because they do not add to the stress on the uterine wall.
31. Answer: 1. Auscultating the fetal heart. Determining the fetal well-being supersedes all
other measures. If the FHR is absent or persistently decelerating, immediate intervention is
required.
32. Answer: 3. Below the ischial spines. A station of +1 indicates that the fetal head is 1 cm
below the ischial spines.
33. Answer: 3. Below the umbilicus on the right side. Fetal heart tones are best auscultated
through the fetal back; because the position is ROP (right occiput presenting), the back would be
below the umbilicus and on the right side.
34. Answer: 3. To the beginning of the next contraction. This is the way to determine the
frequency of the contractions
35. Answer: 3. Clear, almost colorless, and containing little white specks. By 36 weeks
gestation, normal amniotic fluid is colorless with small particles of vernix caseosa present.
36. Answer: 4. Reposition the catheter, recheck the reading, and if it is 55%, keep
monitoring. Adjusting the catheter would be indicated. Normal fetal pulse oximetry should be
between 30% and 70%. 75% to 85% would indicate maternal readings.
37. Answer: 2. Change the clients position. Variable decelerations usually are seen as a result
of cord compression; a change of position will relieve pressure on the cord.
38. Answer: 1. An acceleration. An acceleration is an abrupt elevation above the baseline of 15
beats per minute for 15 seconds; if the acceleration persists for more than 10 minutes it is
considered a change in baseline rate. A tachycardic FHR is above 160 beats per minute.
39. Answer: 4. Occiput posterior. A persistent occiput-posterior position causes intense back
pain because of fetal compression of the sacral nerves. Occiput anterior is the most common fetal
position and does not cause back pain.
40. Answer: 1. Blowing. Blowing forcefully through the mouth controls the strong urge to push
and allows for a more controlled birth of the head.

103

41. Answer: 2. Uterine tetany. Uterine tetany could result from the use of oxytocin to induce
labor. Because oxytocin promotes powerful uterine contractions, uterine tetany may occur. The
oxytocin infusion must be stopped to prevent uterine rupture and fetal compromise.
42. Answer: 4. Support the perineum with the hand to prevent tearing and tell the client to
pant. Gentle pressure is applied to the babys head as it emerges so it is not born too rapidly. The
head is never held back, and it should be supported as it emerges so there will be no vaginal
lacerations. It is impossible to push and pant at the same time.
43. Answer: 1. May lose the ability to push. A pudendal block provides anesthesia to the
perineum.
44. Answer: 1. Fetal scalp pH of 7.14. A fetal scalp pH below 7.25 indicates acidosis and fetal
hypoxia.
45. Answer: 1. Vertex presentation. Vertex presentation (flexion of the fetal head) is the
optimal presentation for passage through the birth canal. Transverse lie is an unacceptable fetal
position for vaginal birth and requires a C-section. Frank breech presentation, in which the
buttocks present first, can be a difficult vaginal delivery. Posterior positioning of the fetal head
can make it difficult for the fetal head to pass under the maternal symphysis pubis.
46. Answer: 4. Oxygenation. Oxygenation of the fetus may be indirectly assessed through fetal
monitoring by closely examining the fetal heart rate strip. Accelerations in the fetal heart rate
strip indicate good oxygenation, while decelerations in the fetal heart rate sometimes indicate
poor fetal oxygenation.
47. Answer: 3. Active phase. Cervical dilation occurs more rapidly during the active phase than
any of the previous phases. The active phase is characterized by cervical dilation that progresses
from 4 to 7 cm. The preparatory, or latent, phase begins with the onset of regular uterine
contractions and ends when rapid cervical dilation begins. Transition is defined as cervical
dilation beginning at 8 cm and lasting until 10 cm or complete dilation.
48. Answer: 3. Perform a pelvic examination. A complaint of rectal pressure usually indicates
a low presenting fetal part, signaling imminent delivery. The nurse should perform a pelvic
examination to assess the dilation of the cervix and station of the presenting fetal part.
49. Answer: 3. Passageway, contractions, placental position and function, psychological
response. The five essential factors (5 Ps) are passenger (fetus), passageway (pelvis), powers
(contractions), placental position and function, and psyche (psychological response of the
mother).

104

50. Answer: 1. Fetal body part that enters the maternal pelvis first. Presentation is the fetal
body part that enters the pelvis first; its classified by the presenting part; the three main
presentations are cephalic/occipital, breech, and shoulder. The relationship of the presenting fetal
part to the maternal pelvis refers to fetal position. The relationship of the long axis to the fetus to
the long axis of the mother refers to fetal lie; the three possible lies are longitudinal, transverse,
and oblique.
51. Answer: 3. Uterine rupture. Uterine rupture is a medical emergency that may occur before
or during labor. Signs and symptoms typically include abdominal pain that may ease after uterine
rupture, vomiting, vaginal bleeding, hypovolemic shock, and fetal distress. With placental
abruption, the client typically complains of vaginal bleeding and constant abdominal pain.
52. Answer: 1. Fetal presenting part is 1 cm above the ischial spines. Station of 1 indicates
that the fetal presenting part is above the ischial spines and has not yet passed through the pelvic
inlet. A station of zero would indicate that the presenting part has passed through the inlet and is
at the level of the ischial spines or is engaged. Passage through the ischial spines with internal
rotation would be indicated by a plus station, such as + 1. Progress of effacement is referred to
by percentages with 100% indicating full effacement and dilation by centimeters (cm) with 10
cm indicating full dilation.
53. Answer: 4. Variability averages between 6 10 BPM. Variability indicates a well
oxygenated fetus with a functioning autonomic nervous system. FHR should accelerate with fetal
movement. Baseline range for the FHR is 120 to 160 beats per minute. Late deceleration
patterns are never reassuring, though early and mild variable decelerations are expected,
reassuring findings.
54. Answer: 2. Stop the Pitocin. Late deceleration patterns noted are most likely related to
alteration in uteroplacental perfusion associated with the strong contractions described. The
immediate action would be to stop the Pitocin infusion since Pitocin is an oxytocic which
stimulates the uterus to contract. The woman is already in an appropriate position for
uteroplacental perfusion. Elevation of her legs would be appropriate if hypotension were present.
Oxygen is appropriate but not the immediate action.
55. Answer: 4. Hypotension. Epidural anesthesia can lead to vasodilation and a drop in blood
pressure that could interfere with adequate placental perfusion. The woman must be well
hydrated before and during epidural anesthesia to prevent this problem and maintain an adequate
blood pressure. Headache is not a side effect since the spinal fluid is not disturbed by this
anesthetic as it would be with a low spinal (saddle block) anesthesia; 2 is an effect of epidural
anesthesia but is not the most harmful. Respiratory depression is a potentially serious
complication.

105

56. Answer: 4. Uses the peribottle to rinse upward into her vagina. Responses 1, 2, and 3 are
all appropriate measures. The peri bottle should be used in a backward direction over the
perineum. The flow should never be directed upward into the vagina since debris would be
forced upward into the uterus through the still-open cervix.
57. Answer: 3. Massage the fundus every hour for the first 24 hours following birth. The
fundus should be massaged only when boggy or soft. Massaging a firm fundus could cause it to
relax. Responses 1, 2, and 4 are all effective measures to enhance and maintain contraction of
the uterus and to facilitate healing.
58. Answer: 3. Vacillate between the desire to have her own nurturing needs met and the
need to take charge of her own care and that of her newborn. One week after birth the
woman should exhibit behaviors characteristic of the taking-hold stage as described in response
3. This stage lasts for as long as 4 to 5 weeks after birth. Responses 1 and 2 are characteristic of
the taking-in stage, which lasts for the first few days after birth. Response 4 reflects the lettinggo stage, which indicates that psychosocial recovery is complete.
59. Answer: 4. Take the baby back to the nursery, reassuring the woman that her rest is a
priority at this time. Response 1 does not take into consideration the need for the new mother to
be nurtured and have her needs met during the taking-in stage. The behavior described is typical
of this stage and not a reflection of ineffective attachment unless the behavior persists. Mothers
need to reestablish their own well-being in order to effectively care for their baby.
60. Answer: 1. Having the children choose or make a gift to give to the new baby upon its
arrival home. Special time should be set aside just for the other children without interruption
from the newborn. Someone other than the mother should carry the baby into the home so she
can give full attention to greeting her other children. Children should be actively involved in the
care of the baby according to their ability without overwhelming them.

Introduction
A 55-item examination, NCLEX style, that challenges your knowledge about Postpartum Care.

Topics

Postpartum Care

Lochias!

Guidelines

Read each question carefully and choose the best answer.

You are given one minute per question. Spend your time wisely!

Answers and rationales are given below. Be sure to read them.

106

If you need more clarifications, please direct them to the comments section.

Questions
1. A postpartum nurse is preparing to care for a woman who has just delivered a healthy
newborn infant. In the immediate postpartum period the nurse plans to take the womans
vital signs:
1. Every 30 minutes during the first hour and then every hour for the next two hours.
2. Every 15 minutes during the first hour and then every 30 minutes for the next two hours.
3. Every hour for the first 2 hours and then every 4 hours
4. Every 5 minutes for the first 30 minutes and then every hour for the next 4 hours.
2. A postpartum nurse is taking the vital signs of a woman who delivered a healthy
newborn infant 4 hours ago. The nurse notes that the mothers temperature is 100.2*F.
Which of the following actions would be most appropriate?
1. Retake the temperature in 15 minutes
2. Notify the physician
3. Document the findings
4. Increase hydration by encouraging oral fluids
3. The nurse is assessing a client who is 6 hours PP after delivering a full-term healthy
infant. The client complains to the nurse of feelings of faintness and dizziness. Which of
the following nursing actions would be most appropriate?
1. Obtain hemoglobin and hematocrit levels
2. Instruct the mother to request help when getting out of bed
3. Elevate the mothers legs
4. Inform the nursery room nurse to avoid bringing the newborn infant to the mother until the feelings
of lightheadedness and dizziness have subsided.
4. A nurse is preparing to perform a fundal assessment on a postpartum client. The initial
nursing action in performing this assessment is which of the following?
1. Ask the client to turn on her side
2. Ask the client to lie flat on her back with the knees and legs flat and straight.
3. Ask the mother to urinate and empty her bladder
4. Massage the fundus gently before determining the level of the fundus.
5. The nurse is assessing the lochia on a 1 day PP patient. The nurse notes that the lochia
is red and has a foul-smelling odor. The nurse determines that this assessment finding is:
1. Normal
2. Indicates the presence of infection
3. Indicates the need for increasing oral fluids
4. Indicates the need for increasing ambulation
6. When performing a PP assessment on a client, the nurse notes the presence of clots in
the lochia. The nurse examines the clots and notes that they are larger than 1 cm. Which of
the following nursing actions is most appropriate?
1. Document the findings
2. Notify the physician
3. Reassess the client in 2 hours
4. Encourage increased intake of fluids.
7. A nurse in a PP unit is instructing a mother regarding lochia and the amount of expected
lochia drainage. The nurse instructs the mother that the normal amount of lochia may vary
but should never exceed the need for:

107

1. One peripad per day


2. Two peripads per day
3. Three peripads per day
4. Eight peripads per day
8. A PP nurse is providing instructions to a woman after delivery of a healthy newborn
infant. The nurse instructs the mother that she should expect normal bowel elimination to
return:
1. One the day of the delivery
2. 3 days PP
3. 7 days PP
4. within 2 weeks PP
9. Select all of the physiological maternal changes that occur during the PP period.
1. Cervical involution ceases immediately
2. Vaginal distention decreases slowly
3. Fundus begins to descend into the pelvis after 24 hours
4. Cardiac output decreases with resultant tachycardia in the first 24 hours
5. Digestive processes slow immediately.
10. A nurse is caring for a PP woman who has received epidural anesthesia and is
monitoring the woman for the presence of a vulva hematoma. Which of the following
assessment findings would best indicate the presence of a hematoma?
1. Complaints of a tearing sensation
2. Complaints of intense pain
3. Changes in vital signs
4. Signs of heavy bruising
11. A nurse is developing a plan of care for a PP woman with a small vulvar hematoma. The
nurse includes which specific intervention in the plan during the first 12 hours following
the delivery of this client?
1. Assess vital signs every 4 hours
2. Inform health care provider of assessment findings
3. Measure fundal height every 4 hours
4. Prepare an ice pack for application to the area.
12. A new mother received epidural anesthesia during labor and had a forceps delivery
after pushing 2 hours. At 6 hours PP, her systolic blood pressure has dropped 20 points,
her diastolic BP has dropped 10 points, and her pulse is 120 beats per minute. The client is
anxious and restless. On further assessment, a vulvar hematoma is verified. After notifying
the health care provider, the nurse immediately plans to:
1. Monitor fundal height
2. Apply perineal pressure
3. Prepare the client for surgery.
4. Reassure the client
13. A nurse is monitoring a new mother in the PP period for signs of hemorrhage. Which of
the following signs, if noted in the mother, would be an early sign of excessive blood loss?
1. A temperature of 100.4*F
2. An increase in the pulse from 88 to 102 BPM
3. An increase in the respiratory rate from 18 to 22 breaths per minute
4. A blood pressure change from 130/88 to 124/80 mm Hg

108

14. A nurse is preparing to assess the uterine fundus of a client in the immediate
postpartum period. When the nurse locates the fundus, she notes that the uterus feels soft
and boggy. Which of the following nursing interventions would be most appropriate
initially?
1. Massage the fundus until it is firm
2. Elevate the mothers legs
3. Push on the uterus to assist in expressing clots
4. Encourage the mother to void
15. A PP nurse is assessing a mother who delivered a healthy newborn infant by C-section.
The nurse is assessing for signs and symptoms of superficial venous thrombosis. Which of
the following signs or symptoms would the nurse note if superficial venous thrombosis
were present?
1. Paleness of the calf area
2. Enlarged, hardened veins
3. Coolness of the calf area
4. Palpable dorsalis pedis pulses
16. A nurse is providing instructions to a mother who has been diagnosed with mastitis.
Which of the following statements if made by the mother indicates a need for further
teaching?
1. I need to take antibiotics, and I should begin to feel better in 24-48 hours.
2. I can use analgesics to assist in alleviating some of the discomfort.
3. I need to wear a supportive bra to relieve the discomfort.
4. I need to stop breastfeeding until this condition resolves.
17. A PP client is being treated for DVT. The nurse understands that the clients response
to treatment will be evaluated by regularly assessing the client for:
1. Dysuria, ecchymosis, and vertigo
2. Epistaxis, hematuria, and dysuria
3. Hematuria, ecchymosis, and epistaxis
4. Hematuria, ecchymosis, and vertigo
18. A nurse performs an assessment on a client who is 4 hours PP. The nurse notes that the
client has cool, clammy skin and is restless and excessively thirsty. The nurse prepares
immediately to:
1. Assess for hypovolemia and notify the health care provider
2. Begin hourly pad counts and reassure the client
3. Begin fundal massage and start oxygen by mask
4. Elevate the head of the bed and assess vital signs
19. A nurse is assessing a client in the 4th stage if labor and notes that the fundus is firm
but that bleeding is excessive. The initial nursing action would be which of the following?
1. Massage the fundus
2. Place the mother in the Trendelenburgs position
3. Notify the physician
4. Record the findings
20. A nurse is caring for a PP client with a diagnosis of DVT who is receiving a continuous
intravenous infusion of heparin sodium. Which of the following laboratory results will the
nurse specifically review to determine if an effective and appropriate dose of the heparin is
being delivered?

109

1. Prothrombin time
2. International normalized ratio
3. Activated partial thromboplastin time
4. Platelet count
21. A nurse is preparing a list of self-care instructions for a PP client who was diagnosed
with mastitis. Select all instructions that would be included on the list.
1. Take the prescribed antibiotics until the soreness subsides.
2. Wear supportive bra
3. Avoid decompression of the breasts by breastfeeding or breast pump
4. Rest during the acute phase
5. Continue to breastfeed if the breasts are not too sore.
22. Methergine or pitocin is prescribed for a woman to treat PP hemorrhage. Before
administration of these medications, the priority nursing assessment is to check the:
1. Amount of lochia
2. Blood pressure
3. Deep tendon reflexes
4. Uterine tone
23. Methergine or pitocin are prescribed for a client with PP hemorrhage. Before
administering the medication(s), the nurse contacts the health provider who prescribed
the medication(s) in which of the following conditions is documented in the clients medical
history?
1. Peripheral vascular disease
2. Hypothyroidism
3. Hypotension
4. Type 1 diabetes
24. Which of the following factors might result in a decreased supply of breastmilk in a PP
mother?
1. Supplemental feedings with formula
2. Maternal diet high in vitamin C
3. An alcoholic drink
4. Frequent feedings
25. Which of the following interventions would be helpful to a breastfeeding mother who is
experiencing engorged breasts?
1. Applying ice
2. Applying a breast binder
3. Teaching how to express her breasts in a warm shower
4. Administering bromocriptine (Parlodel)
26. On completing a fundal assessment, the nurse notes the fundus is situated on the
clients left abdomen. Which of the following actions is appropriate?
1. Ask the client to empty her bladder
2. Straight catheterize the client immediately
3. Call the clients health provider for direction
4. Straight catheterize the client for half of her uterine volume
27. The nurse is about the give a Type 2 diabetic her insulin before breakfast on her first
day postpartum. Which of the following answers best describes insulin requirements
immediately postpartum?

110

1. Lower than during her pregnancy


2. Higher than during her pregnancy
3. Lower than before she became pregnant
4. Higher than before she became pregnant
28. Which of the following findings would be expected when assessing the postpartum
client?
1. Fundus 1 cm above the umbilicus 1 hour postpartum
2. Fundus 1 cm above the umbilicus on postpartum day 3
3. Fundus palpable in the abdomen at 2 weeks postpartum
4. Fundus slightly to the right; 2 cm above umbilicus on postpartum day 2
29. A client is complaining of painful contractions, or afterpains, on postpartum day 2.
Which of the following conditions could increase the severity of afterpains?
1. Bottle-feeding
2. Diabetes
3. Multiple gestation
4. Primiparity
30. On which of the postpartum days can the client expect lochia serosa?
1. Days 3 and 4 PP
2. Days 3 to 10 PP
3. Days 10-14 PP
4. Days 14 to 42 PP
31. Which of the following behaviors characterizes the PP mother in the taking inphase?
1. Passive and dependant
2. Striving for independence and autonomy
3. Curious and interested in care of the baby
4. Exhibiting maximum readiness for new learning
32. Which of the following complications may be indicated by continuous seepage of blood
from the vagina of a PP client, when palpation of the uterus reveals a firm uterus 1 cm
below the umbilicus?
1. Retained placental fragments
2. Urinary tract infection
3. Cervical laceration
4. Uterine atony
33. What type of milk is present in the breasts 7 to 10 days PP?
1. Colostrum
2. Hind milk
3. Mature milk
4. Transitional milk
34. Which of the following complications is most likely responsible for a delayed
postpartum hemorrhage?
1. Cervical laceration
2. Clotting deficiency
3. Perineal laceration
4. Uterine subinvolution
35. Before giving a PP client the rubella vaccine, which of the following facts should the
nurse include in client teaching?

111

1. The vaccine is safe in clients with egg allergies


2. Breast-feeding isnt compatible with the vaccine
3. Transient arthralgia and rash are common adverse effects
4. The client should avoid getting pregnant for 3 months after the vaccine because the vaccine has
teratogenic effects
36. Which of the following changes best described the insulin needs of a client with type 1
diabetes who has just delivered an infant vaginally without complications?
1. Increase
2. Decrease
3. Remain the same as before pregnancy
4. Remain the same as during pregnancy
37. Which of the following responses is most appropriate for a mother with diabetes who
wants to breastfeed her infant but is concerned about the effects of breastfeeding on her
health?
1. Mothers with diabetes who breastfeed have a hard time controlling their insulin needs
2. Mothers with diabetes shouldnt breastfeed because of potential complications
3. Mothers with diabetes shouldnt breastfeed; insulin requirements are doubled.
4. Mothers with diabetes may breastfeed; insulin requirements may decrease from breastfeeding.
38. On the first PP night, a client requests that her baby be sent back to the nursery so she
can get some sleep. The client is most likely in which of the following phases?
1. Depression phase
2. Letting-go phase
3. Taking-hold phase
4. Taking-in phase
39. Which of the following physiological responses is considered normal in the early
postpartum period?
1. Urinary urgency and dysuria
2. Rapid diuresis
3. Decrease in blood pressure
4. Increase motility of the GI system
40. During the 3rd PP day, which of the following observations about the client would the
nurse be most likely to make?
1. The client appears interested in learning about neonatal care
2. The client talks a lot about her birth experience
3. The client sleeps whenever the neonate isnt present
4. The client requests help in choosing a name for the neonate.
41. Which of the following circumstances is most likely to cause uterine atony and lead to
PP hemorrhage?
1. Hypertension
2. Cervical and vaginal tears
3. Urine retention
4. Endometritis
42. Which type of lochia should the nurse expect to find in a client 2 days PP?
1. Foul-smelling
2. Lochia serosa
3. Lochia alba
4. Lochia rubra

112

43. After expulsion of the placenta in a client who has six living children, an infusion of
lactated ringers solution with 10 units of pitocin is ordered. The nurse understands that
this is indicated for this client because:
1. She had a precipitate birth
2. This was an extramural birth
3. Retained placental fragments must be expelled
4. Multigravidas are at increased risk for uterine atony.
44. As part of the postpartum assessment, the nurse examines the breasts of a
primiparous breastfeeding woman who is one day postpartum. An expected finding would
be:
1. Soft, non-tender; colostrum is present
2. Leakage of milk at let down
3. Swollen, warm, and tender upon palpation
4. A few blisters and a bruise on each areola
45. Following the birth of her baby, a woman expresses concern about the weight she
gained during pregnancy and how quickly she can lose it now that the baby is born. The
nurse, in describing the expected pattern of weight loss, should begin by telling this
woman that:
1. Return to pre pregnant weight is usually achieved by the end of the postpartum period
2. Fluid loss from diuresis, diaphoresis, and bleeding accounts for about a 3 pound weight loss
3. The expected weight loss immediately after birth averages about 11 to 13 pounds
4. Lactation will inhibit weight loss since caloric intake must increase to support milk production
46. Which of the following findings would be a source of concern if noted during the
assessment of a woman who is 12 hours postpartum?
1. Postural hypotension
2. Temperature of 100.4F
3. Bradycardia pulse rate of 55 BPM
4. Pain in left calf with dorsiflexion of left foot
47. The nurse examines a woman one hour after birth. The womans fundus is boggy,
midline, and 1 cm below the umbilicus. Her lochial flow is profuse, with two plum-sized
clots. The nurses initial action would be to:
1. Place her on a bedpan to empty her bladder
2. Massage her fundus
3. Call the physician
4. Administer Methergine 0.2 mg IM which has been ordered prn
48. When performing a postpartum check, the nurse should:
1. Assist the woman into a lateral position with upper leg flexed forward to facilitate the examination of
her perineum
2. Assist the woman into a supine position with her arms above her head and her legs extended for the
examination of her abdomen
3. Instruct the woman to avoid urinating just before the examination since a full bladder will facilitate
fundal palpation
4. Wash hands and put on sterile gloves before beginning the check
49. Perineal care is an important infection control measure. When evaluating a
postpartum womans perineal care technique, the nurse would recognize the need for
further instruction if the woman:

113

1. Uses soap and warm water to wash the vulva and perineum
2. Washes from symphysis pubis back to episiotomy
3. Changes her perineal pad every 2 3 hours
4. Uses the peribottle to rinse upward into her vagina
50. Which measure would be least effective in preventing postpartum hemorrhage?
1. Administer Methergine 0.2 mg every 6 hours for 4 doses as ordered
2. Encourage the woman to void every 2 hours
3. Massage the fundus every hour for the first 24 hours following birth
4. Teach the woman the importance of rest and nutrition to enhance healing
51. When making a visit to the home of a postpartum woman one week after birth, the
nurse should recognize that the woman would characteristically:
1. Express a strong need to review events and her behavior during the process of labor and birth
2. Exhibit a reduced attention span, limiting readiness to learn
3. Vacillate between the desire to have her own nurturing needs met and the need to take charge of
her own care and that of her newborn
4. Have reestablished her role as a spouse/partner
52. Four hours after a difficult labor and birth, a primiparous woman refuses to feed her
baby, stating that she is too tired and just wants to sleep. The nurse should:
1. Tell the woman she can rest after she feeds her baby
2. Recognize this as a behavior of the taking-hold stage
3. Record the behavior as ineffective maternal-newborn attachment
4. Take the baby back to the nursery, reassuring the woman that her rest is a priority at this time
53. Parents can facilitate the adjustment of their other children to a new baby by:
1. Having the children choose or make a gift to give to the new baby upon its arrival home
2. Emphasizing activities that keep the new baby and other children together
3. Having the mother carry the new baby into the home so she can show the other children the new
baby
4. Reducing stress on other children by limiting their involvement in the care of the new baby
54. A primiparous woman is in the taking-in stage of psychosocial recovery and adjustment
following birth. The nurse, recognizing the needs of women during this stage, should:
1. Foster an active role in the babys care
2. Provide time for the mother to reflect on the events of and her behavior during childbirth
3. Recognize the womans limited attention span by giving her written materials to read when she gets
home rather than doing a teaching session now
4. Promote maternal independence by encouraging her to meet her own hygiene and comfort needs
55. All of the following are important in the immediate care of the premature neonate.
Which nursing activity should have the greatest priority?
1. Instillation of antibiotic in the eyes
2. Identification by bracelet and foot prints
3. Placement in a warm environment
4. Neurological assessment to determine gestational age

Answers and Rationale


Gauge your performance by counter checking your answers to the answers below. Learn more about
the question by reading the rationale. If you have any disputes or questions, please direct them to the
comments section.

114

1. Answer: 2. Every 15 minutes during the first hour and then every 30 minutes for the
next two hours.
2. Answer: 4. Increase hydration by encouraging oral fluids. The mothers temperature may be
taken every 4 hours while she is awake. Temperatures up to 100.4 (38 C) in the first 24 hours after
birth are often related to the dehydrating effects of labor. The most appropriate action is to increase
hydration by encouraging oral fluids, which should bring the temperature to a normal reading.
Although the nurse would document the findings, the most appropriate action would be to increase the
hydration.
3. Answer: 2. Instruct the mother to request help when getting out of bed. Orthostatic
hypotension may be evident during the first 8 hours after birth. Feelings of faintness or dizziness are
signs that should caution the nurse to be aware of the clients safety. The nurse should advise the
mother to get help the first few times the mother gets out of bed. Obtaining an H/H requires a
physicians order.
4. Answer: 3. Ask the mother to urinate and empty her bladder. Before starting the fundal
assessment, the nurse should ask the mother to empty her bladder so that an accurate assessment
can be done. When the nurse is performing fundal assessment, the nurse asks the woman to lie flat on
her back with the knees flexed. Massaging the fundus is not appropriate unless the fundus is boggy
and soft, and then it should be massaged gently until firm.
5. Answer: 2. Indicates the presence of infection. Lochia, the discharge present after birth, is red
for the first 1 to 3 days and gradually decreases in amount. Normal lochia has a fleshy odor. Foul
smelling or purulent lochia usually indicates infection, and these findings are not normal. Encouraging
the woman to drink fluids or increase ambulation is not an accurate nursing intervention.
6. Answer: 2. Notify the physician. Normally, one may find a few small clots in the first 1 to 2 days
after birth from pooling of blood in the vagina. Clots larger than 1 cm are considered abnormal. The
cause of these clots, such as uterine atony or retained placental fragments, needs to be determined
and treated to prevent further blood loss. Although the findings would be documented, the most
appropriate action is to notify the physician.
7. Answer: 4. Eight peripads per day. The normal amount of lochia may vary with the individual
but should never exceed 4 to 8 peripads per day. The average number of peripads is 6 per day.
8. Answer: 2. 3 days PP. After birth, the nurse should auscultate the womans abdomen in all four
quadrants to determine the return of bowel sounds. Normal bowel elimination usually returns 2 to 3
days PP. Surgery, anesthesia, and the use of narcotics and pain control agents also contribute to the
longer period of altered bowel function.
9. Answer: 1 and 3. In the PP period, cervical healing occurs rapidly and cervical involution
occurs. After 1 week the muscle begins to regenerate and the cervix feels firm and the external os is
the width of a pencil. Although the vaginal mucosa heals and vaginal distention decreases, it takes the
entire PP period for complete involution to occur and muscle tone is never restored to the pregravid
state. The fundus begins to descent into the pelvic cavity after 24 hours, a process known as
involution. Despite blood loss that occurs during delivery of the baby, a transient increase in cardiac
output occurs. The increase in cardiac output, which persists about 48 hours after childbirth, is
probably caused by an increase in stroke volume because Bradycardia is often noted during the PP
period. Soon after childbirth, digestion begins to begin to be active and the new mother is usually
hungry because of the energy expended during labor.
10. Answer: 3. Changes in vital signs. Because the woman has had epidural anesthesia and is
anesthetized, she cannot feel pain, pressure, or a tearing sensation. Changes in vitals indicate
hypovolemia in the anesthetized PP woman with vulvar hematoma. Heavy bruising may be visualized,
but vital sign changes indicate hematoma caused by blood collection in the perineal tissues.

115

11. Answer: 4. Prepare an ice pack for application to the area. Application of ice will reduce
swelling caused by hematoma formation in the vulvar area. The other options are not interventions
that are specific to the plan of care for a client with a small vulvar hematoma.
12. Answer: 3. Prepare the client for surgery. The use of an epidural, prolonged second stage
labor and forceps delivery are predisposing factors for hematoma formation, and a collection of up to
500 ml of blood can occur in the vaginal area. Although the other options may be implemented, the
immediate action would be to prepare the client for surgery to stop the bleeding.
13. Answer: 2. An increase in the pulse from 88 to 102 BPM. During the 4th stage of labor, the
maternal blood pressure, pulse, and respiration should be checked every 15 minutes during the first
hour. A rising pulse is an early sign of excessive blood loss because the heart pumps faster to
compensate for reduced blood volume. The blood pressure will fall as the blood volume diminishes, but
a decreased blood pressure would not be the earliest sign of hemorrhage. A slight rise in temperature
is normal. The respiratory rate is increased slightly.
14. Answer: 1. Massage the fundus until it is firm. If the uterus is not contracted firmly, the first
intervention is to massage the fundus until it is firm and to express clots that may have accumulated
in the uterus. Pushing on an uncontracted uterus can invert the uterus and cause massive
hemorrhage. Elevating the clients legs and encouraging the client to void will not assist in managing
uterine atony. If the uterus does not remain contracted as a result of the uterine massage, the problem
may be distended bladder and the nurse should assist the mother to urinate, but this would not be the
initial action.
15. Answer: 2. Enlarged, hardened veins. Thrombosis of the superficial veins is usually
accompanied by signs and symptoms of inflammation. These include swelling of the involved
extremity and redness, tenderness, and warmth.
16. Answer: 4. I need to stop breastfeeding until this condition resolves. In most cases, the
mother can continue to breastfeed with both breasts. If the affected breast is too sore, the mother can
pump the breast gently. Regular emptying of the breast is important to prevent abscess formation.
Antibiotic therapy assists in resolving the mastitis within 24-48 hours. Additional supportive measures
include ice packs, breast supports, and analgesics.
17. Answer: 3. Hematuria, ecchymosis, and epistaxis. The treatment for DVT is anticoagulant
therapy. The nurse assesses for bleeding, which is an adverse effect of anticoagulants. This includes
hematuria, ecchymosis, and epistaxis. Dysuria and vertigo are not associated specifically with
bleeding.
18. Answer: 1. Assess for hypovolemia and notify the health care provider. Symptoms of
hypovolemia include cool, clammy, pale skin, sensations of anxiety or impending doom, restlessness,
and thirst. When these symptoms are present, the nurse should further assess for hypovolemia and
notify the health care provider.
19. Answer: 3. Notify the physician. If the bleeding is excessive, the cause may be laceration of
the cervix or birth canal. Massaging the fundus if it is firm will not assist in controlling the bleeding.
Trendelenburgs position is to be avoided because it may interfere with cardiac function.
20. 3. Activated partial thromboplastin time. Anticoagulation therapy may be used to prevent the
extension of thrombus by delaying the clotting time of the blood. Activated partial thromboplastin time
should be monitored, and a heparin dose should be adjusted to maintain a therapeutic level of 1.5 to
2.5 times the control. The prothrombin time and the INR are used to monitor coagulation time when
warfarin (Coumadin) is used.
21. Answer: 2, 4, and 5. Mastitis are an infection of the lactating breast. Client instructions include
resting during the acute phase, maintaining a fluid intake of at least 3 L a day, and taking analgesics to
relieve discomfort. Antibiotics may be prescribed and are taken until the complete prescribed course is

116

finished. They are not stopped when the soreness subsides. Additional supportive measures include
the use of moist heat or ice packs and wearing a supportive bra. Continued decompression of the
breast by breastfeeding or pumping is important to empty the breast and prevent formation of an
abscess.
22. Answer: 2. Blood pressure. Methergine and pitocin are agents that are used to prevent or
control postpartum hemorrhage by contracting the uterus. They cause continuous uterine contractions
and may elevate blood pressure. A priority nursing intervention is to check blood pressure. The
physician should be notified if hypertension is present.
23. Answer: 1. Peripheral vascular disease. These medications are avoided in clients with
significant cardiovascular disease, peripheral disease, hypertension, eclampsia, or preeclampsia.
These conditions are worsened by the vasoconstriction effects of these medications.
24. Answer: 1. Supplemental feedings with formula. Routine formula supplementation may
interfere with establishing an adequate milk volume because decreased stimulation to the mothers
nipples affects hormonal levels and milk production.
25. Answer: 3. Teaching how to express her breasts in a warm shower. Teaching the client how
to express her breasts in a warm shower aids with let-down and will give temporary relief. Ice can
promote comfort by vasoconstriction, numbing, and discouraging further letdown of milk.
26. Answer: 1. Ask the client to empty her bladder. A full bladder may displace the uterine
fundus to the left or right side of the abdomen. Catheterization is unnecessary invasive if the woman
can void on her own.
27. Answer: 3. Lower than before she became pregnant. PP insulin requirements are usually
significantly lower than pre pregnancy requirements. Occasionally, clients may require little to no
insulin during the first 24 to 48 hours postpartum.
28. Answer: 1. Fundus 1 cm above the umbilicus 1 hour postpartum. Within the first 12 hours
postpartum, the fundus usually is approximately 1 cm above the umbilicus. The fundus should be
below the umbilicus by PP day 3. The fundus shouldnt be palpated in the abdomen after day 10.
29. Answer: 3. Multiple gestation. Multiple gestation, breastfeeding, multiparity, and conditions
that cause overdistention of the uterus will increase the intensity of after-pains. Bottle-feeding and
diabetes arent directly associated with increasing severity of afterpains unless the client has delivered
a macrosomic infant.
30. Answer: 2. Days 3 to 10 PP. On the third and fourth PP days, the lochia becomes a pale pink or
brown and contains old blood, serum, leukocytes, and tissue debris. This type of lochia usually lasts
until PP day 10. Lochia rubra usually last for the first 3 to 4 days PP. Lochia alba, which contain
leukocytes, decidua, epithelial cells, mucus, and bacteria, may continue for 2 to 6 weeks PP.
31. Answer: 1. Passive and dependant. During the taking in phase, which usually lasts 1-3 days,
the mother is passive and dependent and expresses her own needs rather than the neonates needs.
The taking hold phase usually lasts from days 3-10 PP. During this stage, the mother strives for
independence and autonomy; she also becomes curious and interested in the care of the baby and is
most ready to learn.
32. Answer: 3. Cervical laceration. Continuous seepage of blood may be due to cervical or vaginal
lacerations if the uterus is firm and contracting. Retained placental fragments and uterine atony may
cause subinvolution of the uterus, making it soft, boggy, and larger than expected. UTI wont cause
vaginal bleeding, although hematuria may be present.
33. Answer: 4. Transitional milk. Transitional milk comes after colostrum and usually lasts until 2
weeks PP.
34. Answer: 4. Uterine subinvolution. Late postpartum bleeding is often the result of subinvolution
of the uterus. Retained products of conception or infection often cause subinvolution. Cervical or

117

perineal lacerations can cause an immediate postpartum hemorrhage. A client with a clotting
deficiency may also have an immediate PP hemorrhage if the deficiency isnt corrected at the time of
delivery.
35. Answer: 4. The client should avoid getting pregnant for 3 months after the vaccine
because the vaccine has teratogenic effects. The client must understand that she must not
become pregnant for 3 months after the vaccination because of its potential teratogenic effects. The
rubella vaccine is made from duck eggs so an allergic reaction may occur in clients with egg allergies.
The virus is not transmitted into the breast milk, so clients may continue to breastfeed after the
vaccination. Transient arthralgia and rash are common adverse effects of the vaccine.
36. Answer: 2. Decrease. The placenta produces the hormone human placental lactogen, an insulin
antagonist. After birth, the placenta, the major source of insulin resistance, is gone. Insulin needs
decrease and women with type 1 diabetes may only need one-half to two-thirds of the prenatal insulin
during the first few PP days.
37. Answer: 4. Mothers with diabetes may breastfeed; insulin requirements may decrease
from breastfeeding. Breastfeeding has an antidiabetogenic effect. Insulin needs are decreased
because carbohydrates are used in milk production. Breastfeeding mothers are at a higher risk of
hypoglycemia in the first PP days after birth because the glucose levels are lower. Mothers with
diabetes should be encouraged to breastfeed.
38. Answer: 4. Taking-in phase. The taking-in phase occurs in the first 24 hours after birth. The
mother is concerned with her own needs and requires support from staff and relatives. The taking-hold
phase occurs when the mother is ready to take responsibility for her care as well as the infants care.
The letting-go phase begins several weeks later, when the mother incorporates the new infant into the
family unit.
39. Answer: 2. Rapid diuresis. In the early PP period, theres an increase in the glomerular filtration
rate and a drop in the progesterone levels, which result in rapid diuresis. There should be no urinary
urgency, though a woman may feel anxious about voiding. Theres a minimal change in blood pressure
following childbirth, and a residual decrease in GI motility.
40. Answer: 1. The client appears interested in learning about neonatal care. The third to
tenth days of PP care are the taking-hold phase, in which the new mother strives for independence
and is eager for her neonate. The other options describe the phase in which the mother relives her
birth experience.
41. Answer: 3. Urine retention. Urine retention causes a distended bladder to displace the uterus
above the umbilicus and to the side, which prevents the uterus from contracting. The uterus needs to
remain contracted if bleeding is to stay within normal limits. Cervical and vaginal tears can cause PP
hemorrhage but are less common occurrences in the PP period.
42. Answer: 4. Lochia rubra
43. Answer: 4. Multigravidas are at increased risk for uterine atony. Multiple full-term
pregnancies and deliveries result in overstretched uterine muscles that do not contract efficiently and
bleeding may ensue.
44. Answer: 1. Soft, non-tender; colostrum is present. Breasts are essentially unchanged for the
first two to three days after birth. Colostrum is present and may leak from the nipples.
45. Answer: 3. The expected weight loss immediately after birth averages about 11 to 13
pounds. Prepregnant weight is usually achieved by 2 to 3 months after birth, not within the 6-week
postpartum period. Weight loss from diuresis, diaphoresis, and bleeding is about 9 pounds. Weight loss
continues during breastfeeding since fat stores developed during pregnancy and extra calories
consumed are used as part of the lactation process.

118

46. Answer: 4. Pain in left calf with dorsiflexion of left foot. Responses 1 and 3 are expected
related to circulatory changes after birth. A temperature of 100.4F in the first 24 hours is most likely
indicative of dehydration which is easily corrected by increasing oral fluid intake. The findings in
response 4 indicate a positive Homan sign and are suggestive of thrombophlebitis and should be
investigated further.
47. Answer: 2. Massage her fundus. A boggy or soft fundus indicates that uterine atony is present.
This is confirmed by the profuse lochia and passage of clots. The first action would be to massage the
fundus until firm, followed by 3 and 4, especially if the fundus does not become or remain firm with
massage. There is no indication of a distended bladder since the fundus is midline and below the
umbilicus.
48. Answer: 1. Assist the woman into a lateral position with upper leg flexed forward to
facilitate the examination of her perineum. While the supine position is best for examining the
abdomen, the woman should keep her arms at her sides and slightly flex her knees in order to relax
abdominal muscles and facilitate palpation of the fundus. The bladder should be emptied before the
check. A full bladder alters the position of the fundus and makes the findings inaccurate. Although
hands are washed before starting the check, clean (not sterile) gloves are put on just before the
perineum and pad are assessed to protect from contact with blood and secretions.
49. Answer: 4. Uses the peribottle to rinse upward into her vagina. Responses 1, 2, and 3 are
all appropriate measures. The peribottle should be used in a backward direction over the perineum.
The flow should never be directed upward into the vagina since debris would be forced upward into the
uterus through the still-open cervix.
50. Answer: 3. Massage the fundus every hour for the first 24 hours following birth. The
fundus should be massaged only when boggy or soft. Massaging a firm fundus could cause it to relax.
Responses 1, 2, and 4 are all effective measures to enhance and maintain contraction of the uterus
and to facilitate healing.
51. Answer: 3. Express a strong need to review events and her behavior during the process
of labor and birth. One week after birth the woman should exhibit behaviors characteristic of the
taking-hold stage as described in response 3. This stage lasts for as long as 4 to 5 weeks after
birth. Responses 1 and 2 are characteristic of the taking-in stage, which lasts for the first few days
after birth. Response 4 reflects the letting-go stage, which indicates that psychosocial recovery is
complete.
52. Answer: 4. Recognize this as a behavior of the taking-hold stage. Response 1 does not
take into consideration the need for the new mother to be nurtured and have her needs met during the
taking-in stage. The behavior described is typical of this stage and not a reflection of ineffective
attachment unless the behavior persists. Mothers need to reestablish their own well-being in order to
effectively care for their baby.
53. Answer: 1. Having the children choose or make a gift to give to the new baby upon its
arrival home. Special time should be set aside just for the other children without interruption from
the newborn. Someone other than the mother should carry the baby into the home so she can give full
attention to greeting her other children. Children should be actively involved in the care of the baby
according to their ability without overwhelming them.
54. Answer: 2. Provide time for the mother to reflect on the events of and her behavior
during childbirth. The focus of the taking-in stage is nurturing the new mother by meeting her
dependency needs for rest, comfort, hygiene, and nutrition. Once they are met, she is more able to
take an active role, not only in her own care but also the care of her newborn. Women express a need
to review their childbirth experience and evaluate their performance. Short teaching sessions, using
written materials to reinforce the content presented, are a more effective approach.

119

55. Answer: 3. Placement in a warm environment

Introduction
A 50-item NCLEX style questionnaire that challenges your knowledge about Obstetrical Nursing. This
includes nursing care for pregnant women during the antepartum period.

Topics

Obstetrical Nursing

Antepartal Care

GTPAL Questions

Preeclampsia

Guidelines

Read each question carefully and choose the best answer.

You are given one minute per question. Spend your time wisely!

Answers and rationales are given below. Be sure to read them.

If you need more clarifications, please direct them to the comments section.

Questions
1. A nursing instructor is conducting lecture and is reviewing the functions of the female
reproductive system. She asks Mark to describe the follicle-stimulating hormone (FSH) and
the luteinizing hormone (LH). Mark accurately responds by stating that:
1. FSH and LH are released from the anterior pituitary gland.
2. FSH and LH are secreted by the corpus luteum of the ovary
3. FSH and LH are secreted by the adrenal glands
4. FSH and LH stimulate the formation of milk during pregnancy.
2. A nurse is describing the process of fetal circulation to a client during a prenatal visit.
The nurse accurately tells the client that fetal circulation consists of:
1. Two umbilical veins and one umbilical artery
2. Two umbilical arteries and one umbilical vein
3. Arteries carrying oxygenated blood to the fetus
4. Veins carrying deoxygenated blood to the fetus
3. During a prenatal visit at 38 weeks, a nurse assesses the fetal heart rate. The nurse
determines that the fetal heart rate is normal if which of the following is noted?
1. 80 BPM
2. 100 BPM
3. 150 BPM
4. 180 BPM
4. A client arrives at a prenatal clinic for the first prenatal assessment. The client tells a
nurse that the first day of her last menstrual period was September 19th, 2005. Using
Naegeles rule, the nurse determines the estimated date of confinement as:
1. July 26, 2006
2. June 12, 2007

120

3. June 26, 2006


4. July 12, 2007
5. A nurse is collecting data during an admission assessment of a client who is pregnant
with twins. The client has a healthy 5-year old child that was delivered at 37 weeks and
tells the nurse that she doesnt have any history of abortion or fetal demise. The nurse
would document the GTPAL for this client as:
1. G = 3, T = 2, P = 0, A = 0, L =1
2. G = 2, T = 0, P = 1, A = 0, L =1
3. G = 1, T = 1. P = 1, A = 0, L = 1
4. G = 2, T = 0, P = 0, A = 0, L = 1
6. A nurse is performing an assessment of a primipara who is being evaluated in a clinic
during her second trimester of pregnancy. Which of the following indicates an abnormal
physical finding necessitating further testing?
1. Consistent increase in fundal height
2. Fetal heart rate of 180 BPM
3. Braxton hicks contractions
4. Quickening
7. A nurse is reviewing the record of a client who has just been told that a pregnancy test
is positive. The physician has documented the presence of a Goodells sign. The nurse
determines this sign indicates:
1. A softening of the cervix
2. A soft blowing sound that corresponds to the maternal pulse during auscultation of the uterus.
3. The presence of hCG in the urine
4. The presence of fetal movement
8. A nursing instructor asks a nursing student who is preparing to assist with the
assessment of a pregnant client to describe the process of quickening. Which of the
following statements if made by the student indicates an understanding of this term?
1. It is the irregular, painless contractions that occur throughout pregnancy.
2. It is the soft blowing sound that can be heard when the uterus is auscultated.
3. It is the fetal movement that is felt by the mother.
4. It is the thinning of the lower uterine segment.
9. A nurse midwife is performing an assessment of a pregnant client and is assessing the
client for the presence of ballottement. Which of the following would the nurse implement
to test for the presence of ballottement?
1. Auscultating for fetal heart sounds
2. Palpating the abdomen for fetal movement
3. Assessing the cervix for thinning
4. Initiating a gentle upward tap on the cervix
10. A nurse is assisting in performing an assessment on a client who suspects that she is
pregnant and is checking the client for probable signs of
pregnancy. Select all probable signs of pregnancy.
1. Uterine enlargement
2. Fetal heart rate detected by nonelectric device
3. Outline of the fetus via radiography or ultrasound
4. Chadwicks sign
5. Braxton Hicks contractions
6. Ballottement

121

11. A pregnant client calls the clinic and tells a nurse that she is experiencing leg cramps
and is awakened by the cramps at night. To provide relief from the leg cramps, the nurse
tells the client to:
1. Dorsiflex the foot while extending the knee when the cramps occur
2. Dorsiflex the foot while flexing the knee when the cramps occur
3. Plantar flex the foot while flexing the knee when the cramps occur
4. Plantar flex the foot while extending the knee when the cramps occur.
12. A nurse is providing instructions to a client in the first trimester of pregnancy
regarding measures to assist in reducing breast tenderness. The nurse tells the client to:
1. Avoid wearing a bra
2. Wash the nipples and areola area daily with soap, and massage the breasts with lotion.
3. Wear tight-fitting blouses or dresses to provide support
4. Wash the breasts with warm water and keep them dry
13. A pregnant client in the last trimester has been admitted to the hospital with a
diagnosis of severe preeclampsia. A nurse monitors for complications associated with the
diagnosis and assesses the client for:
1. Any bleeding, such as in the gums, petechiae, and purpura.
2. Enlargement of the breasts
3. Periods of fetal movement followed by quiet periods
4. Complaints of feeling hot when the room is cool
14. A client in the first trimester of pregnancy arrives at a health care clinic and reports
that she has been experiencing vaginal bleeding. A threatened abortion is suspected, and
the nurse instructs the client regarding management of care. Which statement, if made by
the client, indicates a need for further education?
1. I will maintain strict bedrest throughout the remainder of pregnancy.
2. I will avoid sexual intercourse until the bleeding has stopped, and for 2 weeks following the last
evidence of bleeding.
3. I will count the number of perineal pads used on a daily basis and note the amount and color of
blood on the pad.
4. I will watch for the evidence of the passage of tissue.
15. A prenatal nurse is providing instructions to a group of pregnant client regarding
measures to prevent toxoplasmosis. Which statement if made by one of the clients
indicates a need for further instructions?
1. I need to cook meat thoroughly.
2. I need to avoid touching mucous membranes of the mouth or eyes while handling raw meat.
3. I need to drink unpasteurized milk only.
4. I need to avoid contact with materials that are possibly contaminated with cat feces.
16. A homecare nurse visits a pregnant client who has a diagnosis of mild Preeclampsia
and who is being monitored for pregnancy induced hypertension (PIH). Which assessment
finding indicates a worsening of the Preeclampsia and the need to notify the physician?
1. Blood pressure reading is at the prenatal baseline
2. Urinary output has increased
3. The client complains of a headache and blurred vision
4. Dependent edema has resolved
17. A nurse implements a teaching plan for a pregnant client who is newly diagnosed with
gestational diabetes. Which statement if made by the client indicates a need for further
education?

122

1. I need to stay on the diabetic diet.


2. I will perform glucose monitoring at home.
3. I need to avoid exercise because of the negative effects of insulin production.
4. I need to be aware of any infections and report signs of infection immediately to my health care
provider.
18. A primigravida is receiving magnesium sulfate for the treatment of pregnancy induced
hypertension (PIH). The nurse who is caring for the client is performing assessments every
30 minutes. Which assessment finding would be of most concern to the nurse?
1. Urinary output of 20 ml since the previous assessment
2. Deep tendon reflexes of 2+
3. Respiratory rate of 10 BPM
4. Fetal heart rate of 120 BPM
19. A nurse is caring for a pregnant client with Preeclampsia. The nurse prepares a plan of
care for the client and documents in the plan that if the client progresses from
Preeclampsia to eclampsia, the nurses first action is to:
1. Administer magnesium sulfate intravenously
2. Assess the blood pressure and fetal heart rate
3. Clean and maintain an open airway
4. Administer oxygen by face mask
20. A nurse is monitoring a pregnant client with pregnancy induced hypertension who is at
risk for Preeclampsia. The nurse checks the client for which specific signs of Preeclampsia
(select all that apply)?
1. Elevated blood pressure
2. Negative urinary protein
3. Facial edema
4. Increased respirations
21. Rho (D) immune globulin (RhoGAM) is prescribed for a woman following delivery of a
newborn infant and the nurse provides information to the woman about the purpose of the
medication. The nurse determines that the woman understands the purpose of the
medication if the woman states that it will protect her next baby from which of the
following?
1. Being affected by Rh incompatibility
2. Having Rh positive blood
3. Developing a rubella infection
4. Developing physiological jaundice
22. A pregnant client is receiving magnesium sulfate for the management of preeclampsia.
A nurse determines the client is experiencing toxicity from the medication if which of the
following is noted on assessment?
1. Presence of deep tendon reflexes
2. Serum magnesium level of 6 mEq/L
3. Proteinuria of +3
4. Respirations of 10 per minute
23. A woman with preeclampsia is receiving magnesium sulfate. The nurse assigned to care
for the client determines that the magnesium therapy is effective if:
1. Ankle clonus in noted
2. The blood pressure decreases

123

3. Seizures do not occur


4. Scotomas are present
24. A nurse is caring for a pregnant client with severe preeclampsia who is receiving IV
magnesium sulfate. Select all nursing interventions that apply in the care for the client.
1. Monitor maternal vital signs every 2 hours
2. Notify the physician if respirations are less than 18 per minute.
3. Monitor renal function and cardiac function closely
4. Keep calcium gluconate on hand in case of a magnesium sulfate overdose
5. Monitor deep tendon reflexes hourly
6. Monitor I and Os hourly
7. Notify the physician if urinary output is less than 30 ml per hour.
25. In the 12th week of gestation, a client completely expels the products of conception.
Because the client is Rh negative, the nurse must:
1. Administer RhoGAM within 72 hours
2. Make certain she receives RhoGAM on her first clinic visit
3. Not give RhoGAM, since it is not used with the birth of a stillborn
4. Make certain the client does not receive RhoGAM, since the gestation only lasted 12 weeks.
26. In a lecture on sexual functioning, the nurse plans to include the fact that ovulation
occurs when the:
1. Oxytocin is too high
2. Blood level of LH is too high
3. Progesterone level is high
4. Endometrial wall is sloughed off.
27. The chief function of progesterone is the:
1. Development of the female reproductive system
2. Stimulation of the follicles for ovulation to occur
3. Preparation of the uterus to receive a fertilized egg
4. Establishment of secondary male sex characteristics
28. The developing cells are called a fetus from the:
1. Time the fetal heart is heard
2. Eighth week to the time of birth
3. Implantation of the fertilized ovum
4. End of the send week to the onset of labor
29. After the first four months of pregnancy, the chief source of estrogen and progesterone
is the:
1. Placenta
2. Adrenal cortex
3. Corpus luteum
4. Anterior hypophysis
30. The nurse recognizes that an expected change in the hematologic system that occurs
during the 2nd trimester of pregnancy is:
1. A decrease in WBCs
2. In increase in hematocrit
3. An increase in blood volume
4. A decrease in sedimentation rate

124

31. The nurse is aware than an adaptation of pregnancy is an increased blood supply to the
pelvic region that results in a purplish discoloration of the vaginal mucosa, which is known
as:
1. Ladins sign
2. Hegars sign
3. Goodells sign
4. Chadwicks sign
32. A pregnant client is making her first Antepartum visit. She has a two year old son born
at 40 weeks, a 5 year old daughter born at 38 weeks, and 7 year old twin daughters born at
35 weeks. She had a spontaneous abortion 3 years ago at 10 weeks. Using the GTPAL
format, the nurse should identify that the client is:
1. G4 T3 P2 A1 L4
2. G5 T2 P2 A1 L4
3. G5 T2 P1 A1 L4
4. G4 T3 P1 A1 L4
33. An expected cardiopulmonary adaptation experienced by most pregnant women is:
1. Tachycardia
2. Dyspnea at rest
3. Progression of dependent edema
4. Shortness of breath on exertion
34. Nutritional planning for a newly pregnant woman of average height and weighing 145
pounds should include:
1. A decrease of 200 calories a day
2. An increase of 300 calories a day
3. An increase of 500 calories a day
4. A maintenance of her present caloric intake per day
35. During a prenatal examination, the nurse draws blood from a young Rh negative client
and explain that an indirect Coombs test will be performed to predict whether the fetus is
at risk for:
1. Acute hemolytic disease
2. Respiratory distress syndrome
3. Protein metabolic deficiency
4. Physiologic hyperbilirubinemia
36. When involved in prenatal teaching, the nurse should advise the clients that an
increase in vaginal secretions during pregnancy is called leukorrhea and is caused by
increased:
1. Metabolic rates
2. Production of estrogen
3. Functioning of the Bartholin glands
4. Supply of sodium chloride to the cells of the vagina
37. A 26-year old multigravida is 14 weeks pregnant and is scheduled for an alphafetoprotein test. She asks the nurse, What does the alpha-fetoprotein test indicate? The
nurse bases a response on the knowledge that this test can detect:
1. Kidney defects
2. Cardiac defects
3. Neural tube defects
4. Urinary tract defects

125

38. At a prenatal visit at 36 weeks gestation, a client complains of discomfort with


irregularly occurring contractions. The nurse instructs the client to:
1. Lie down until they stop
2. Walk around until they subside
3. Time contraction for 30 minutes
4. Take 10 grains of aspirin for the discomfort
39. The nurse teaches a pregnant woman to avoid lying on her back. The nurse has based
this statement on the knowledge that the supine position can:
1. Unduly prolong labor
2. Cause decreased placental perfusion
3. Lead to transient episodes of hypotension
4. Interfere with free movement of the coccyx
40. The pituitary hormone that stimulates the secretion of milk from the mammary glands
is:
1. Prolactin
2. Oxytocin
3. Estrogen
4. Progesterone
41. Which of the following symptoms occurs with a hydatidiform mole?
1. Heavy, bright red bleeding every 21 days
2. Fetal cardiac motion after 6 weeks gestation
3. Benign tumors found in the smooth muscle of the uterus
4. Snowstorm pattern on ultrasound with no fetus or gestational sac
42. Which of the following terms applies to the tiny, blanched, slightly raised end
arterioles found on the face, neck, arms, and chest during pregnancy?
1. Epulis
2. Linea nigra
3. Striae gravidarum
4. Telangiectasias
43. Which of the following conditions is common in pregnant women in the 2nd trimester of
pregnancy?
1. Mastitis
2. Metabolic alkalosis
3. Physiologic anemia
4. Respiratory acidosis
44. A 21-year old client, 6 weeks pregnant is diagnosed with hyperemesis gravidarum.
This excessive vomiting during pregnancy will often result in which of the following
conditions?
1. Bowel perforation
2. Electrolyte imbalance
3. Miscarriage
4. Pregnancy induced hypertension (PIH)
45. Clients with gestational diabetes are usually managed by which of the following
therapies?
1. Diet
2. NPH insulin (long-acting)

126

3. Oral hypoglycemic drugs


4. Oral hypoglycemic drugs and insulin
46. The antagonist for magnesium sulfate should be readily available to any client
receiving IV magnesium. Which of the following drugs is the antidote for magnesium
toxicity?
1. Calcium gluconate
2. Hydralazine (Apresoline)
3. Narcan
4. RhoGAM
47. Which of the following answers best describes the stage of pregnancy in which
maternal and fetal blood are exchanged?
1. Conception
2. 9 weeks gestation, when the fetal heart is well developed
3. 32-34 weeks gestation
4. maternal and fetal blood are never exchanged
48. Gravida refers to which of the following descriptions?
1. A serious pregnancy
2. Number of times a female has been pregnant
3. Number of children a female has delivered
4. Number of term pregnancies a female has had.
49. A pregnant woman at 32 weeks gestation complains of feeling dizzy and lightheaded
while her fundal height is being measured. Her skin is pale and moist. The nurses initial
response would be to:
1. Assess the womans blood pressure and pulse
2. Have the woman breathe into a paper bag
3. Raise the womans legs
4. Turn the woman on her side.
50. A pregnant womans last menstrual period began on April 8, 2005, and ended on April
13. Using Naegeles rule her estimated date of birth would be:
1. January 15, 2006
2. January 20, 2006
3. July 1, 2006
4. November 5, 2005

Answers and Rationale


Gauge your performance by counter checking your answers to the answers below. Learn more about
the question by reading the rationale. If you have any disputes or questions, please direct them to the
comments section.
1. Answer: 1. FSH and LH are released from the anterior pituitary gland. FSH and LH, when
stimulated by gonadotropin-releasing hormone from the hypothalamus, are released from the anterior
pituitary gland to stimulate follicular growth and development, growth of the graafian follicle, and
production of progesterone.
2. Answer: 2. Two umbilical arteries and one umbilical vein. Blood pumped by the embryos
heart leaves the embryo through two umbilical arteries. Once oxygenated, the blood then is returned
by one umbilical vein. Arteries carry deoxygenated blood and waste products from the fetus, and veins
carry oxygenated blood and provide oxygen and nutrients to the fetus.

127

3. Answer: 3. 150 BPM. The fetal heart rate depends in gestational age and ranges from 160-170
BPM in the first trimester but slows with fetal growth to 120-160 BPM near or at term. At or near term,
if the fetal heart rate is less than 120 or more than 160 BPM with the uterus at rest, the fetus may be
in distress.
4. Answer: 3. June 26, 2006. Accurate use of Naegeles rule requires that the woman have a regular
28-day menstrual cycle. Add 7 days to the first day of the last menstrual period, subtract three
months, and then add one year to that date.
5. Answer: 2. G = 2, T = 0, P = 1, A = 0, L =1. Pregnancy outcomes can be described with the

acronym GTPAL.
G is Gravidity, the number of pregnancies.

T is term births, the number of born at term (38 to 41 weeks).

P is preterm births, the number born before 38 weeks gestation.

A is abortions or miscarriages, included in G if before 20 weeks gestation, included in parity


if past 20 weeks AOE.

L is live births, the number of births of living children.


Therefore, a woman who is pregnant with twins and has a child has a gravida of 2. Because the child
was delivered at 37 weeks, the number of preterm births is 1, and the number of term births is 0. The
number of abortions is 0, and the number of live births is 1.
6. Answer: 2. Fetal heart rate of 180 BPM. The normal range of the fetal heart rate depends on
gestational age. The heart rate is usually 160-170 BPM in the first trimester and slows with fetal
growth, near and at term, the fetal heart rate ranges from 120-160 BPM. The other options are
expected.
7. Answer: 1. A softening of the cervix. In the early weeks of pregnancy the cervix becomes softer
as a result of increased vascularity and hyperplasia, which causes the Goodells sign.
8. Answer: 3. It is the fetal movement that is felt by the mother. Quickening is fetal
movement and may occur as early as the 16th and 18th week of gestation, and the mother first
notices subtle fetal movements that gradually increase in intensity. Braxton Hicks contractions are
irregular, painless contractions that may occur throughout the pregnancy. A thinning of the lower
uterine segment occurs about the 6th week of pregnancy and is called Hegars sign.
9. Answer: 4. Initiating a gentle upward tap on the cervix. Ballottement is a technique of
palpating a floating structure by bouncing it gently and feeling it rebound. In the technique used to
palpate the fetus, the examiner places a finger in the vagina and taps gently upward, causing the fetus
to rise. The fetus then sinks, and the examiner feels a gentle tap on the finger.
10. Answers: 1, 4, 5, and 6.

The probable signs of pregnancy include:


Uterine Enlargement

Hegars sign or softening and thinning of the uterine segment that occurs at week 6.

Goodells sign or softening of the cervix that occurs at the beginning of the 2nd month

Chadwicks sign or bluish coloration of the mucous membranes of the cervix, vagina and vulva.
Occurs at week 6.

Ballottement or rebounding of the fetus against the examiners fingers of palpation

Braxton-Hicks contractions

Positive pregnancy test measuring for hCG.


Positive signs of pregnancy include:

128

Fetal Heart Rate detected by electronic device (doppler) at 10-12 weeks

Fetal Heart rate detected by nonelectronic device (fetoscope) at 20 weeks AOG

Active fetal movement palpable by the examiners

Outline of the fetus via radiography or ultrasound


11. Answer: 1. Dorsiflex the foot while extending the knee when the cramps occur. Legs
cramps occur when the pregnant woman stretches the leg and plantar flexes the foot. Dorsiflexion of
the foot while extending the knee stretches the affected muscle, prevents the muscle from contracting,
and stops the cramping.
12. Answer: 4. Wash the breasts with warm water and keep them dry. The pregnant woman
should be instructed to wash the breasts with warm water and keep them dry. The woman should be
instructed to avoid using soap on the nipples and areola area to prevent the drying of tissues. Wearing
a supportive bra with wide adjustable straps can decrease breast tenderness. Tight-fitting blouses or
dresses will cause discomfort.
13. Answer: 1. Any bleeding, such as in the gums, petechiae, and purpura. Severe
Preeclampsia can trigger disseminated intravascular coagulation because of the widespread damage
to vascular integrity. Bleeding is an early sign of DIC and should be reported to the M.D.
14. Answer: 1. I will maintain strict bedrest throughout the remainder of pregnancy. Strict
bed rest throughout the remainder of pregnancy is not required. The woman is advised to curtail
sexual activities until the bleeding has ceased, and for 2 weeks following the last evidence of bleeding
or as recommended by the physician. The woman is instructed to count the number of perineal pads
used daily and to note the quantity and color of blood on the pad. The woman also should watch for
the evidence of the passage of tissue.
15. Answer: 3. I need to drink unpasteurized milk only. All pregnant women should be
advised to do the following to prevent the development of toxoplasmosis. Women should be instructed
to cook meats thoroughly, avoid touching mucous membranes and eyes while handling raw meat;
thoroughly wash all kitchen surfaces that come into contact with uncooked meat, wash the hands
thoroughly after handling raw meat; avoid uncooked eggs and unpasteurized milk; wash fruits and
vegetables before consumption, and avoid contact with materials that possibly are contaminated with
cat feces, such as cat litter boxes, sandboxes, and garden soil.
16. Answer: 3. The client complains of a headache and blurred vision. If the client complains
of a headache and blurred vision, the physician should be notified because these are signs of
worsening Preeclampsia.
17. Answer: 3. I need to avoid exercise because of the negative effects of insulin
production. Exercise is safe for the client with gestational diabetes and is helpful in lowering the
blood glucose level.
18. Answer: 3. Respiratory rate of 10 BPM. Magnesium sulfate depresses the respiratory rate. If
the respiratory rate is less than 12 breaths per minute, the physician or other health care provider
needs to be notified, and continuation of the medication needs to be reassessed. A urinary output of
20 ml in a 30 minute period is adequate; less than 30 ml in one hour needs to be reported. Deep
tendon reflexes of 2+ are normal. The fetal heart rate is WNL for a resting fetus.
19. Answer: 3. Clean and maintain an open airway. The immediate care during a seizure
(eclampsia) is to ensure a patent airway. The other options are actions that follow or will be
implemented after the seizure has ceased.
20. Answers: 1 Elevated blood pressure and 3 Facial edema. The three classic signs of
preeclampsia are hypertension, generalized edema, and proteinuria. Increased respirations are not a
sign of preeclampsia.

129

21. Answer: 1. Being affected by Rh incompatibility. Rh incompatibility can occur when an Rhnegative mom becomes sensitized to the Rh antigen. Sensitization may develop when an Rh-negative
woman becomes pregnant with a fetus who is Rh positive. During pregnancy and at delivery, some of
the babys Rh positive blood can enter the maternal circulation, causing the womans immune system
to form antibodies against Rh positive blood. Administration of Rho(D) immune globulin prevents the
woman from developing antibodies against Rh positive blood by providing passive antibody protection
against the Rh antigen.
22. Answer: 4. Respirations of 10 per minute. Magnesium toxicity can occur from magnesium
sulfate therapy. Signs of toxicity relate to the central nervous system depressant effects of the
medication and include respiratory depression, loss of deep tendon reflexes, and a sudden drop in the
fetal heart rate and maternal heart rate and blood pressure. Therapeutic levels of magnesium are 4-7
mEq/L. Proteinuria of +3 would be noted in a client with preeclampsia.
23. Answer: 3. Seizures do not occur. For a client with preeclampsia, the goal of care is directed at
preventing eclampsia (seizures). Magnesium sulfate is an anticonvulsant, not an antihypertensive
agent. Although a decrease in blood pressure may be noted initially, this effect is usually transient.
Ankle clonus indicated hyperreflexia and may precede the onset of eclampsia. Scotomas are areas of
complete or partial blindness. Visual disturbances, such as scotomas, often precede an eclamptic
seizure.
24. Answers: 3, 4, 5, 6, and 7. When caring for a client receiving magnesium sulfate therapy, the
nurse would monitor maternal vital signs, especially respirations, every 30-60 minutes and notify the
physician if respirations are less than 12, because this would indicate respiratory depression. Calcium
gluconate is kept on hand in case of magnesium sulfate overdose, because calcium gluconate is the
antidote for magnesium sulfate toxicity. Deep tendon reflexes are assessed hourly. Cardiac and renal
function is monitored closely. The urine output should be maintained at 30 ml per hour because the
medication is eliminated through the kidneys.
25. Answer: 1. Administer RhoGAM within 72 hours. RhoGAM is given within 72 hours
postpartum if the client has not been sensitized already.
26. Answer: 2. Blood level of LH is too high. It is the surge of LH secretion in mid cycle that is
responsible for ovulation.
27. Answer: 3. Preparation of the uterus to receive a fertilized egg. Progesterone stimulates
differentiation of the endometrium into a secretory type of tissue.
28. Answer: 2. Eighth week to the time of birth. In the first 7-14 days the ovum is known as a
blastocyst; it is called an embryo until the eighth week; the developing cells are then called a fetus
until birth.
29. Answer: 1. Placenta. When placental formation is complete, around the 16th week of
pregnancy; it produces estrogen and progesterone.
30. Answer: 3. An increase in blood volume. The blood volume increases by approximately 4050% during pregnancy. The peak blood volume occurs between 30 and 34 weeks of gestation. The
hematocrit decreases as a result of the increased blood volume.
31. Answer: 4. Chadwicks sign. A purplish color results from the increased vascularity and blood
vessel engorgement of the vagina.
32. Answer: 3. G5 T2 P1 A1 L4. 5 pregnancies; 2 term births; twins count as 1; one abortion; 4
living children.
33. Answer: 4. Shortness of breath on exertion. This is an expected cardiopulmonary adaptation
during pregnancy; it is caused by an increased ventricular rate and elevated diaphragm.
34. Answer: 2. An increase of 300 calories a day. This is the recommended caloric increase for
adult women to meet the increased metabolic demands of pregnancy.

130

35. Answer: 1. Acute hemolytic disease. When an Rh negative mother carries an Rh positive fetus
there is a risk for maternal antibodies against Rh positive blood; antibodies cross the placenta and
destroy the fetal RBCs.
36. Answer: 2. Production of estrogen. The increase of estrogen during pregnancy causes
hyperplasia of the vaginal mucosa, which leads to increased production of mucus by the endocervical
glands. The mucus contains exfoliated epithelial cells.
37. Answer: 3. Neural tube defects. The alpha-fetoprotein test detects neural tube defects and
Down syndrome.
38. Answer: 2. Walk around until they subside. Ambulation relieves Braxton Hicks.
39. Answer: 2. Cause decreased placental perfusion. This is because impedance of venous
return by the gravid uterus, which causes hypotension and decreased systemic perfusion.
40. Answer: 1. Prolactin. Prolactin is the hormone from the anterior pituitary gland that stimulates
mammary gland secretion. Oxytocin, a posterior pituitary hormone, stimulates the uterine musculature
to contract and causes the let down reflex.
41. Answer: 4. Snowstorm pattern on ultrasound with no fetus or gestational sac. The
chorionic villi of a molar pregnancy resemble a snowstorm pattern on ultrasound. Bleeding with a
hydatidiform mole is often dark brown and may occur erratically for weeks or months.
42. Answer: 4. Telangiectasias. The dilated arterioles that occur during pregnancy are due to the
elevated level of circulating estrogen. The linea nigra is a pigmented line extending from the
symphysis pubis to the top of the fundus during pregnancy.
43. Answer: 3. Physiologic anemia. Hemoglobin and hematocrit levels decrease during pregnancy
as the increase in plasma volume exceeds the increase in red blood cell production.
44. Answer: 2. Electrolyte imbalance. Excessive vomiting in clients with hyperemesis gravidarum
often causes weight loss and fluid, electrolyte, and acid-base imbalances.
45. Answer: 1. Diet. Clients with gestational diabetes are usually managed by diet alone to control
their glucose intolerance. Oral hypoglycemic agents are contraindicated in pregnancy. NPH isnt
usually needed for blood glucose control for GDM.
46. Answer: 1. Calcium gluconate. Calcium gluconate is the antidote for magnesium toxicity. Ten
ml of 10% calcium gluconate is given IV push over 3-5 minutes. Hydralazine is given for sustained
elevated blood pressures in preeclamptic clients.
47. Answer: 4. maternal and fetal blood are never exchanged. Only nutrients and waste
products are transferred across the placenta. Blood exchange only occurs in complications and some
medical procedures accidentally.
48. Answer: 2. Number of times a female has been pregnant. Gravida refers to the number of
times a female has been pregnant, regardless of pregnancy outcome or the number of neonates
delivered.
49. Answer: 4. Turn the woman on her side. During a fundal height measurement the woman is
placed in a supine position. This woman is experiencing supine hypotension as a result of uterine
compression of the vena cava and abdominal aorta. Turning her on her side will remove the
compression and restore cardiac output and blood pressure. Then vital signs can be assessed. Raising
her legs will not solve the problem since pressure will still remain on the major abdominal blood
vessels, thereby continuing to impede cardiac output. Breathing into a paper bag is the solution for
dizziness related to respiratory alkalosis associated with hyperventilation.
50. Answer: 1. January 15, 2006. Naegeles rule requires subtracting 3 months and adding 7 days
and 1 year if appropriate to the first day of a pregnant womans last menstrual period. When this rule,
is used with April 8, 2005, the estimated date of birth is January 15, 2006.

131

Introduction
A 60-item examination that questions your wits about Kidney Stones, Bladder Cancer and Diseases of
the Prostate.

Topics

Kidney Stones

Bladder Cancer

Diseases of the Prostate

Guidelines

Read each question carefully and choose the best answer.

You are given one minute per question. Spend your time wisely!

Answers and rationales are given below. Be sure to read them.

If you need more clarifications, please direct them to the comments section.

Questions
1. A client is complaining of severe flank and abdominal pain. A flat plate of the abdomen
shows urolithiasis. Which of the following interventions is important?
1. Strain all urine
2. Limit fluid intake
3. Enforce strict bed rest
4. Encourage a high calcium diet
2. A client is receiving a radiation implant for the treatment of bladder cancer. Which of the
following interventions is appropriate?
1. Flush all urine down the toilet
2. Restrict the clients fluid intake
3. Place the client in a semi-private room
4. Monitor the client for signs and symptoms of cystitis
3. A client has just received a renal transplant and has started cyclosporine therapy to
prevent graft rejection. Which of the following conditions is a major complication of this
drug therapy?
1. Depression
2. Hemorrhage
3. Infection
4. Peptic ulcer disease
4. A client received a kidney transplant 2 months ago. Hes admitted to the hospital with
the diagnosis of acute rejection. Which of the following assessment findings would be
expected?
1. Hypotension
2. Normal body temperature
3. Decreased WBC count
4. Elevated BUN and creatinine levels
5. The client is to undergo kidney transplantation with a living donor. Which of the
following preoperative assessments is important?
1. Urine output
2. Signs of graft rejection

132

3. Signs and symptoms of rejection


4. Clients support system and understanding of lifestyle changes.
6. A client had a transurethral prostatectomy for benign prostatic hypertrophy. Hes
currently being treated with a continuous bladder irrigation and is complaining of an
increase in severity of bladder spasms. Which of the interventions should be done first?
1. Administer an oral analgesic
2. Stop the irrigation and call the physician
3. Administer a belladonna and opium suppository as ordered by the physician.
4. Check for the presence of clots, and make sure the catheter is draining properly.
7. A client is admitted with a diagnosis of hydronephrosis secondary to calculi. The calculi
have been removed and postobstructive diuresis is occurring. Which of the following
interventions should be done?
1. Take vital signs every 8 hours
2. Weigh the client every other day
3. Assess for urine output every shift
4. Monitor the clients electrolyte levels.
8. A client has passed a renal calculus. The nurse sends the specimen to the laboratory so
it can be analyzed for which of the following factors?
1. Antibodies
2. Type of infection
3. Composition of calculus
4. Size and number of calculi
9. Which of the following symptoms indicate acute rejection of a transplanted kidney?
1. Edema, nausea
2. Fever, anorexia
3. Weight gain, pain at graft site
4. Increased WBC count, pain with voiding
10. Adverse reactions of prednisone therapy include which of the following conditions?
1. Acne and bleeding gums
2. Sodium retention and constipation
3. Mood swings and increased temperature
4. Increased blood glucose levels and decreased wound healing.
11. The nurse suspects that a client with polyuria is experiencing water diuresis. Which
laboratory value suggests water diuresis?
1. High urine specific gravity
2. High urine osmolarity
3. Normal to low urine specific gravity
4. Elevated urine pH
12. A client is diagnosed with prostate cancer. Which test is used to monitor progression
of this disease?
1. Serum creatinine
2. Complete blood cell count (CBC)
3. Prostate specific antigen (PSA)
4. Serum potassium
13. a 27-year old client, who became paraplegic after a swimming accident, is
experiencing autonomic dysreflexia. Which condition is the most common cause of
autonomic dysreflexia?

133

1. Upper respiratory infection


2. Incontinence
3. Bladder distention
4. Diarrhea
14. When providing discharge teaching for a client with uric acid calculi, the nurse should
an instruction to avoid which type of diet?
1. Low-calcium
2. Low-oxalate
3. High-oxalate
4. High-purine
15. The client with urolithiasis has a history of chronic urinary tract infections. The nurse
concludes that this client most likely has which of the following types of urinary stones?
1. Calcium oxalate
2. Uric acid
3. Struvite
4. Cystine
16. The nurse is receiving in transfer from the postanesthesia care unit a client who has
had a percutaneous ultrasonic lithotripsy for calculuses in the renal pelvis. The nurse
anticipates that the clients care will involve monitoring which of the following?
1. Suprapubic tube
2. Urethral stent
3. Nephrostomy tube
4. Jackson-Pratt drain
17. The client is admitted to the ER following a MVA. The client was wearing a lap seat belt
when the accident occurred. The client has hematuria and lower abdominal pain. To
determine further whether the pain is due to bladder trauma, the nurse asks the client if
the pain is referred to which of the following areas?
1. Shoulder
2. Umbilicus
3. Costovertebral angle
4. Hip
18. The client complains of fever, perineal pain, and urinary urgency, frequency, and
dysuria. To assess whether the clients problem is related to bacterial prostatitis, the nurse
would look at the results of the prostate examination, which should reveal that the
prostate gland is:
1. Tender, indurated, and warm to the touch
2. Soft and swollen
3. Tender and edematous with ecchymosis
4. Reddened, swollen, and boggy.
19. The nurse is taking the history of a client who has had benign prostatic hyperplasia in
the past. To determine whether the client currently is experiencing difficulty, the nurse
asks the client about the presence of which of the following early symptoms?
1. Urge incontinence
2. Nocturia
3. Decreased force in the stream of urine
4. Urinary retention

134

20. The client who has a cold is seen in the emergency room with inability to void. Because
the client has a history of BPH, the nurse determines that the client should be questioned
about the use of which of the following medications?
1. Diuretics
2. Antibiotics
3. Antitussives
4. Decongestants
21. The nurse is preparing to care for the client following a renal scan. Which of the
following would the nurse include in the plan of care?
1. Place the client on radiation precautions for 18 hours
2. Save all urine in a radiation safe container for 18 hours
3. Limit contact with the client to 20 minutes per hour.
4. No special precautions except to wear gloves if in contact with the clients urine.
22. The client passes a urinary stone, and lab analysis of the stone indicates that it is
composed of calcium oxalate. Based on this analysis, which of the following would the
nurse specifically include in the dietary instructions?
1. Increase intake of meat, fish, plums, and cranberries
2. Avoid citrus fruits and citrus juices
3. Avoid green, leafy vegetables such as spinach.
4. Increase intake of dairy products.
23. The client returns to the nursing unit following a pyelolithotomy for removal of a
kidney stone. A Penrose drain is in place. Which of the following would the nurse include
on the clients postoperative care?
1. Sterile irrigation of the Penrose drain
2. Frequent dressing changes around the Penrose drain
3. Weighing the dressings
4. Maintaining the clients position on the affected side
24. The nurse is caring for a client following a kidney transplant. The client develops
oliguria. Which of the following would the nurse anticipate to be prescribed as the
treatment of oliguria?
1. Encourage fluid intake
2. Administration of diuretics
3. Irrigation of foley catheter
4. Restricting fluids
25. A week after kidney transplantation the client develops a temperature of 101, the
blood pressure is elevated, and the kidney is tender. The x-ray results the transplanted
kidney is enlarged. Based on these assessment findings, the nurse would suspect which of
the following?
1. Acute rejection
2. Chronic rejection
3. Kidney infection
4. Kidney obstruction
26. The client with BPH undergoes a transurethral resection of the prostate.
Postoperatively, the client is receiving continuous bladder irrigations. The nurse assesses
the client for signs of transurethral resection syndrome. Which of the following assessment
data would indicate the onset of this syndrome?

135

1. Bradycardia and confusion


2. Tachycardia and diarrhea
3. Decreased urinary output and bladder spasms
4. Increased urinary output and anemia
27. The client is admitted to the hospital with BPH, and a transurethral resection of the
prostate is performed. Four hours after surgery the nurse takes the clients VS and empties
the urinary drainage bag. Which of the following assessment findings would indicate the
need to notify the physician?
1. Red bloody urine
2. Urinary output of 200 ml greater than intake
3. Blood pressure of 100/50 and pulse 130.
4. Pain related to bladder spasms.
28. Which of the following symptoms is the most common clinical finding associated with
bladder cancer?
1. Suprapubic pain
2. Dysuria
3. Painless hematuria
4. Urinary retention
29. A client who has been diagnosed with bladder cancer is scheduled for an ileal conduit.
Preoperatively, the nurse reinforces the clients understanding of the surgical procedure
by explaining that an ileal conduit:
1. Is a temporary procedure that can be reversed later.
2. Diverts urine into the sigmoid colon, where it is expelled through the rectum.
3. Conveys urine from the ureters to a stoma opening in the abdomen.
4. Creates an opening in the bladder that allows urine to drain into an external pouch.
30. After surgery for an ileal conduit, the nurse should closely evaluate the client for the
occurrence of which of the following complications related to pelvic surgery?
1. Peritonitis
2. Thrombophlebitis
3. Ascites
4. Inguinal hernia
31. The nurse is assessing the urine of a client who has had an ileal conduit and notes that
the urine is yellow with a moderate amount of mucus. Based on the assessment data,
which of the following nursing interventions would be most appropriate at this time?
1. Change the appliance bag
2. Notify the physician
3. Obtain a urine specimen for culture
4. Encourage a high fluid intake
32. When teaching the client to care for an ileal conduit, the nurse instructs the client to
empty the appliance frequently, primarily to prevent which of the following problems?
1. Rupture of the ileal conduit
2. Interruption of urine production
3. Development of odor
4. Separation of the appliance from the skin
33. The client with an ileal conduit will be using a reusable appliance at home. The nurse
should teach the client to clean the appliance routinely with what product?

136

1. Baking soda
2. Soap
3. Hydrogen peroxide
4. Alcohol
34. The nurse is evaluating the discharge teaching for a client who has an ileal conduit.
Which of the following statements indicates that the client has correctly understood the
teaching? Select all that apply.
1. If I limit my fluid intake I will not have to empty my ostomy pouch as often.
2. I can place an aspirin tablet in my pouch to decrease odor.
3. I can usually keep my ostomy pouch on for 3 to 7 days before changing it.
4. I must use a skin barrier to protect my skin from urine.
5. I should empty my ostomy pouch of urine when it is full.
35. A female client with a urinary diversion tells the nurse, This urinary pouch is
embarrassing. Everyone will know that Im not normal. I dont see how I can go out in
public anymore. The most appropriate nursing diagnosis for this patient is:
1. Anxiety related to the presence of urinary diversion.
2. Deficient Knowledge about how to care for the urinary diversion.
3. Low Self-Esteem related to feelings of worthlessness
4. Disturbed Body Image related to creation of a urinary diversion.
36. The nurse teaches the client with a urinary diversion to attach the appliance to a
standard urine collection bag at night. The most important reason for doing this is to
prevent:
1. Urine reflux into the stoma
2. Appliance separation
3. Urine leakage
4. The need to restrict fluids
37. The nurse teaches the client with an ileal conduit measures to prevent a UTI. Which of
the following measures would be most effective?
1. Avoid people with respiratory tract infections
2. Maintain a daily fluid intake of 2,000 to 3,000 ml
3. Use sterile technique to change the appliance
4. Irrigate the stoma daily.
38. A client who has been diagnosed with calculi reports that the pain is intermittent and
less colicky. Which of the following nursing actions is most important at this time?
1. Report hematuria to the physician
2. Strain the urine carefully
3. Administer meperidine (Demerol) every 3 hours
4. Apply warm compresses to the flank area
39. A client has a ureteral catheter in place after renal surgery. A priority nursing action for
care of the ureteral catheter would be to:
1. Irrigate the catheter with 30 ml of normal saline every 8 hours
2. Ensure that the catheter is draining freely
3. Clamp the catheter every 2 hours for 30 minutes.
4. Ensure that the catheter drains at least 30 ml an hour
40. Which of the following interventions would be most appropriate for preventing the
development of a paralytic ileus in a client who has undergone renal surgery?

137

1. Encourage the client to ambulate every 2 to 4 hours


2. Offer 3 to 4 ounces of a carbonated beverage periodically.
3. Encourage use of a stool softener
4. Continue intravenous fluid therapy
41. The nurse is conducting a postoperative assessment of a client on the first day after
renal surgery. Which of the following findings would be most important for the nurse to
report to the physician?
1. Temperature, 99.8
2. Urine output, 20 ml/hour
3. Absence of bowel sounds
4. A 22 inch area of serous sanguineous drainage on the flank dressing.
42. Because a clients renal stone was found to be composed to uric acid, a low-purine,
alkaline-ash diet was ordered. Incorporation of which of the following food items into the
home diet would indicate that the client understands the necessary diet modifications?
1. Milk, apples, tomatoes, and corn
2. Eggs, spinach, dried peas, and gravy.
3. Salmon, chicken, caviar, and asparagus
4. Grapes, corn, cereals, and liver.
43. Allopurinol (Zyloprim), 200 mg/day, is prescribed for the client with renal calculi to take
home. The nurse should teach the client about which of the following side effects of this
medication?
1. Retinopathy
2. Maculopapular rash
3. Nasal congestion
4. Dizziness
44. The client has a clinic appointment scheduled 10 days after discharge. Which
laboratory finding at that time would indicate that allopurinol (Zyloprim) has had a
therapeutic effect?
1. Decreased urinary alkaline phosphatase level
2. Increased urinary calcium excretion
3. Increased serum calcium level
4. Decreased serum uric acid level
45. When developing a plan of care for the client with stress incontinence, the nurse
should take into consideration that stress incontinence is best defined as the involuntary
loss of urine associated with:
1. A strong urge to urinate
2. Overdistention of the bladder
3. Activities that increase abdominal pressure
4. Obstruction of the urethra
46. Which of the following assessment data would most likely be related to a clients
current complaint of stress incontinence?
1. The clients intake of 2 to 3 L of fluid per day.
2. The clients history of three full-term pregnancies
3. The clients age of 45 years
4. The clients history of competitive swimming
47. The nurse is developing a teaching plan for a client with stress incontinence. Which of
the following instructions should be included?

138

1. Avoid activities that are stressful and upsetting


2. Avoid caffeine and alcohol
3. Do not wear a girdle
4. Limit physical exertion
48. A client has urge incontinence. Which of the following signs and symptoms would the
nurse expect to find in this client?
1. Inability to empty the bladder
2. Loss of urine when coughing
3. Involuntary urination with minimal warning
4. Frequent dribbling of urine
49. A 72-year old male client is brought to the emergency room by his son. The client is
extremely uncomfortable and has been unable to void for the past 12 hours. He has known
for some time that he has an enlarged prostate but has wanted to avoid surgery. The best
method for the nurse to use when assessing for bladder distention in a male client is to
check for:
1. A rounded swelling above the pubis.
2. Dullness in the lower left quadrant
3. Rebound tenderness below the symphysis
4. Urine discharge from the urethral meatus
50. During a clients urinary bladder catheterization, the bladder is emptied gradually. The
best rationale for the nurses action is that completely emptying an overdistended bladder
at one time tends to cause:
1. Renal failure
2. Abdominal cramping
3. Possible shock
4. Atrophy of bladder musculature
51. The primary reason for taping an indwelling catheter laterally to the thigh of a male
client is to:
1. Eliminate pressure at the penoscrotal angle
2. Prevent the catheter from kinking in the urethra
3. Prevent accidental catheter removal
4. Allow the client to turn without kinking the catheter
52. The primary function of the prostate gland is:
1. To store underdeveloped sperm before ejaculation
2. To regulate the acidity and alkalinity of the environment for proper sperm development.
3. To produce a secretion that aids in the nourishment and passage of sperm
4. To secrete a hormone that stimulates the production and maturation of sperm
53. The nurse is reviewing a medication history of a client with BPH. Which medication
should be recognized as likely to aggravate BPH?
1. Metformin (Glucophage)
2. Buspirone (BuSpar)
3. Inhaled ipratropium (Atrovent)
4. Ophthalmic timolol (Timoptic)
54. A client is scheduled to undergo a transurethral resection of the prostate gland (TURP).
The procedure is to be done under spinal anesthesia. Postoperatively, the nurse should be
particularly alert for early signs of:

139

1. Convulsions
2. Cardiac arrest
3. Renal shutdown
4. Respiratory paralysis
55. A client with BPH is being treated with terazosin (Hytrin) 2 mg at bedtime. The nurse
should monitor the clients:
1. Urinary nitrites
2. White blood cell count
3. Blood pressure
4. Pulse
56. A client underwent a TURP, and a large three way catheter was inserted in the bladder
with continuous bladder irrigation. In which of the following circumstances would the nurse
increase the flow rate of the continuous bladder irrigation?
1. When the drainage is continuous but slow
2. When the drainage appears cloudy and dark yellow
3. When the drainage becomes bright red
4. When there is no drainage of urine and irrigating solution
57. A priority nursing diagnosis for the client who is being discharged t home 3 days after a
TURP would be:
1. Deficient fluid volume
2. Imbalanced Nutrition: Less than Body Requirements
3. Impaired Tissue Integrity
4. Ineffective Airway Clearance
58. If a clients prostate enlargement is caused by a malignancy, which of the following
blood examinations should the nurse anticipate to assess whether metastasis has
occurred?
1. Serum creatinine level
2. Serum acid phosphatase level
3. Total nonprotein nitrogen level
4. Endogenous creatinine clearance time
59. Steroids, if used following kidney transplantation would cause which of the following
side effects?
1. Alopecia
2. Increase Cholesterol Level
3. Orthostatic Hypotension
4. Increase Blood Glucose Level
60. Mr. Roberto was readmitted to the hospital with acute graft rejection. Which of the
following assessment finding would be expected?
1. Hypotension
2. Normal Body Temperature
3. Decreased WBC
4. Elevated BUN and Creatinine

Answers and Rationale


1. Answer: 1. Urine should be strained for calculi and sent to the lab for analysis. Fluid intake of 3 to 4
L is encouraged to flush the urinary tract and prevent further calculi formation. A low-calcium diet is

140

recommended to help prevent the formation of calcium calculi. Ambulation is encouraged to help pass
the calculi through gravity.
2. Answer: 4. Cystitis is the most common adverse reaction of clients undergoing radiation therapy;
symptoms include dysuria, frequency, urgency, and nocturia. Clients with radiation implants require a
private room. Urine of clients with radiation implants for bladder cancer should be sent to the
radioisotopes lab for monitoring. It is recommended that fluid intake be increased.
3. Answer: 3. Infections is the major complication to watch for in clients on cyclosporine therapy
because its an immunosuppressive drug. Depression may occur posttransplantation but not because
of cyclosporine. Hemorrhage is a complication associated with anticoagulant therapy. Peptic ulcer
disease is a complication of steroid therapy.
4. Answer: 4. In a client with acute renal graft rejection, evidence of deteriorating renal function is
expected. The nurse would see elevated WBC counts and fever because the body is recognizing the
graft as foreign and is attempting to fight it. The client would most likely have acute hypertension.
5. Answer: 4. The client undergoing a renal transplantation will need vigilant follow-up care and must
adhere to the medical regimen. The client is most likely anuric or oliguric preoperatively, but
postoperatively will require close monitoring of urine output to make sure the transplanted kidney is
functioning optimally. While the client will always need to be monitored for signs and symptoms of
infection, its most important post-op will require close monitoring of urine output to make sure the
transplanted kidney is functioning optimally. While the client will always need to be monitored for signs
and symptoms of infection, its most important postoperatively due to the immunosuppressant
therapy. Rejection can occur postoperatively.
6. Answer: 4. Blood clots and blocked outflow if the urine can increase spasms. The irrigation shouldnt
be stopped as long as the catheter is draining because clots will form. A belladonna and opium
suppository should be given to relieve spasms but only after assessment of the drainage. Oral
analgesics should be given if the spasms are unrelieved by the belladonna and opium suppository.
7. Answer: 4. Postobstructive diuresis seen in hydronephrosis can cause electrolyte imbalances; lab
values must be checked so electrolytes can be replaced as needed. VS should initially be taken every
30 minutes for the first 4 hours and then every 2 hours. Urine output needs to be assessed hourly. The
clients weight should be taken daily to assess fluid status more closely.
8. Answer: 3. The calculus should be analyzed for composition to determine appropriate interventions
such as dietary restrictions. Calculi dont result in infections. The size and number of calculi arent
relevant, and they dont contain antibodies.
9. Answer: 3. Pain at the graft site and weight gain indicates the transplanted kidney isnt functioning
and possibly is being rejected. Transplant clients usually have edema, anorexia, fever, and nausea
before transplantation, so those symptoms may not indicate rejection.
10. Answer: 4. Steroid use tends to increase blood glucose levels, particularly in clients with diabetes
and borderline diabetes. Steroids also contribute to poor wound healing and may cause acne, mood
swings, and sodium and water retention. Steroids dont affect thermoregulation, bleeding tendencies,
or constipation.
11. Answer: 3. Water diuresis causes low urine specific gravity, low urine osmolarity, and a normal to
elevated serum sodium level. High specific gravity indicates dehydration. Hypernatremia signals
acidosis and shock. Elevated urine pH can result from potassium deficiency, a high-protein diet, or
uncontrolled diabetes.
12. Answer: 3. The PSA test is used to monitor prostate cancer progression; higher PSA levels indicate
a greater tumor burden. Serum creatinine levels may suggest blockage from an enlarged prostate. CBC
is used to diagnose anemia and polycythemia. Serum potassium levels identify hypokalemia and
hyperkalemia.

141

13. Answer: 3. Autonomic dysreflexia is a potentially life-threatening complication of spinal cord injury,
occurring from obstruction of the urinary system or bowel. Incontinence and diarrhea dont result in
obstruction of the urinary system or bowel, respectively. An URI could obstruct the respiratory system,
but not the urinary or bowel system.
14. Answer: 4. To control uric acid calculi, the client should follow a low-purine diet, which excludes
high-purine foods such as organ meats. A low-calcium diet decreases the risk for oxalate renal calculi.
Oxalate is an essential amino acid and must be included in the diet. A low-oxalate diet is used to
control calcium or oxalate calculi.
15. Answer: 3. Struvite stones commonly are referred to as infection stones because they form in urine
that is alkaline and rich in ammonia, such as with a urinary tract infection. Calcium oxalate stones
result from increased calcium intake or conditions that raise serum calcium concentrations. Uric acid
stones occur in clients with gout. Cystine stones are rare and occur in clients with a genetic defect that
results in decreased renal absorption of the amino acid cystine.
16. Answer: 3. A nephrostomy tube is put in place after a percutaneous ultrasonic lithotripsy to treat
calculuses in the renal pelvis. The client may also have a foley catheter to drain urine produced by the
other kidney. The nurse monitors the drainage from each of these tubes and strains the urine to detect
elimination of the calculus fragments.
17. Answer: 1. Bladder trauma or injury is characterized by lower abdominal pain that may radiate to
one of the shoulders. Bladder injury pain does not radiate to the umbilicus, CV angle, or hip.
18. Answer: 1. The client with prostatitis has a prostate gland that is swollen and tender but that is also
warm to the touch, firm, and indurated. Systemic symptoms include fever with chills, perineal and low
back pain, and signs of urinary tract infection (which often accompany the disorder).
19. Answer: 3. Decreased force in the stream of urine is an early sign of BPH. The stream later
becomes weak and dribbling. The client then may develop hematuria, frequency, urgency, urge
incontinence, and nocturia. If untreated, complete obstruction and urinary retention can occur.
20. Answer: 4. In the client with BPH, episodes of urinary retention can be triggered by certain
medications, such as decongestants, anticholinergics, and antidepressants. The client should be
questioned about the use of these medications if the client has urinary retention. Retention can also be
precipitated by other factors, such as alcoholic beverages, infection, bedrest, and becoming chilled.
21. Answer: 4. No specific precautions are necessary following a renal scan. Urination into a commode
is acceptable without risk from the small amount of radioactive material to be excreted. The nurse
wears gloves to maintain body secretion precautions.
22. Answer: 3. Oxalate is found in dark green foods such as spinach. Other foods that raise urinary
oxalate are rhubarb, strawberries, chocolate, wheat bran, nuts, beets, and tea.
23. Answer: 2. Frequent dressing changes around the Penrose drain is required to protect the skin
against breakdown from urinary drainage. If urinary drainage is excessive, an ostomy pouch may be
placed over the drain to protect the skin. A Penrose drain is not irrigated. Weighing the dressings is not
necessary. Placing the client on the affected side will prevent a free flow of urine through the drain.
24. Answer: 2. To increase urinary output, diuretics and osmotic agents are considered. The client
should be monitored closely because fluid overload can cause hypertension, congestive heart failure,
and pulmonary edema. Fluid intake would not be encouraged or restricted. Irrigation of the foley
catheter will not assist in alleviating this oliguria.
25. Answer: 1. Acute rejection most often occurs in the first 2 weeks after transplant. Clinical
manifestations include fever, malaise, elevated WBC count, acute hypertension, graft tenderness, and
manifestations of deteriorating renal function. Chronic rejection occurs gradually during a period of
months to years. Although kidney infection or obstruction can occur, the symptoms presented in the
question do not relate specifically to these disorders.

142

26. Answer: 1. Transurethral resection syndrome is caused by increased absorption of nonelectrolyte


irrigating fluid used during surgery. The client may show signs of cerebral edema and increased
intracranial pressure such as increased blood pressure, bradycardia, confusion, disorientation, muscle
twitching, visual disturbances, and nausea and vomiting.
27. Answer: 3. Frank bleeding (arterial or venous) may occur during the first few days after surgery.
Some hematuria is usual for several days after surgery. A urinary output of 200 ml of greater than
intake is adequate. Bladder spasms are expected to occur after surgery. A rapid pulse with a low blood
pressure is a potential sign of excessive blood loss. The physician should be notified.
28. Answer: 3. Painless hematuria is the most common clinical finding in bladder cancer. Other
symptoms include frequency, dysuria, and urgency, but these are not as common as the hematuria.
Suprapubic pain and urinary retention do not occur in bladder cancer.
29. Answer: 3. An ileal conduit is a permanent urinary diversion in which a portion of the ileum is
surgically resected and one end of the segment is closed. The ureters are surgically attached to this
segment of the ileum, and the open end of the ileum is brought to the skin surface on the abdomen to
form the stoma. The client must wear a pouch to collect the urine that continually flows through the
conduit. The bladder is removed during the surgical procedure and the ileal conduit is not reversible.
Diversion of the urine to the sigmoid colon is called a ureter ileosigmoidostomy. An opening in the
bladder that allows urine to drain externally is called a cystostomy.
30. Answer: 2. After pelvic surgery, there is an increased chance of thrombophlebitis owing to the
pelvic manipulation that can interfere with circulation and promote venous stasis. Peritonitis is a
potential complication of any abdominal surgery, not just pelvic surgery. Ascites is most frequently an
indication of liver disease. Inguinal hernia may be caused by an increase in abdominal pressure or a
congenital weakness of the abdominal wall; ventral hernia occurs at the site of a previous abdominal
surgery.
31. Answer: 4. Mucus is secreted by the intestinal segment used to create the conduit and is a normal
occurrence. The client should be encouraged to maintain a large fluid intake to help flush the mucus
out of the conduit. Because mucus in the urine is expected, it is not necessary to change the appliance
bag or notify the physician. The mucus is not an indication of an infection, so a urine culture is not
necessary.
32. Answer: 4. If the appliance becomes too full, it is likely to pull away from the skin completely or to
leak urine onto the skin. A full appliance will not rupture the ileal conduit or interrupt urine production.
Odor formation has numerous causes.
33. Answer: 2. A reusable appliance should be routinely cleaned with soap and water.
34. Answer: 3, 4. The client with an ileal conduit must learn self-care activities related to care of the
stoma and ostomy appliances. The client should be taught to increase fluid intake to about 3,000 ml
per day and should not limit intake. Adequate fluid intake helps to flush mucus from the ileal conduit.
The ostomy appliance should be changed approximately every 3 to 7 days and whenever a leak
develops. A skin barrier is essential to protecting the skin from the irritation of the urine. An aspirin
should not be used as a method of odor control because it can be an irritant to the stoma and lead to
ulceration. The ostomy pouch should be emptied when it is one-third to one-half full to prevent the
weight from pulling the appliance away from the skin.
35. Answer: 4. It is normal for clients to express fears and concerns about the body changes associated
with a urinary diversion. Allowing the client time to verbalize concerns in a supportive environment
and suggest that she discuss these concerns with people who have successfully adjusted to ostomy
surgery can help her begin coping with these changes in a positive manner. Although the client may be
anxious about this situation and self-esteem may be diminished, the underlying problem is disturbance
in body image. There are no data to support a diagnosis of Deficient Knowledge.

143

36. Answer: 1. The most important reason for attaching the appliance to a standard urine collection
bag at night is to prevent reflux into the stoma and ureters, which can result in infection. Use of a
standard collection bag also keeps the appliance from separating from the skin and helps prevent urine
leakage from an overly full bag, but the primary purpose is to prevent reflux of urine. A client with a
urinary diversion should drink 2000-3000 ml of fluid each day; it would be inappropriate to suggest
decreasing fluid intake.
37. Answer: 2. Maintaining a fluid intake of 2,000 to 3,000 ml/day is likely to be effective in preventing
UTI. A high fluid intake results in high urine output, which prevents urinary stasis and bacterial growth.
Avoiding people with respiratory tract infections will not prevent urinary tract infections. Clean, not
sterile, technique is used to change the appliance. An ileal conduit stoma is not irrigated.
38. Answer: 2. Intermittent pain that is less colicky indicates that the calculi may be moving along the
urinary tract. Fluids should be encouraged to promote movement, and the urine should be strained to
detect passage of the stone. Hematuria is to be expected from the irritation of the stone. Analgesics
should be administered when the client needs them, not routinely. Moist heat to the flank area is
helpful when renal colic occurs, but it is less necessary as pain is lessened.
39. Answer: 2. The ureteral catheter should drain freely without bleeding at the site. The catheter is
rarely irrigated, and any irrigation would be done by the physician. The catheter is never clamped. The
clients total urine output (ureteral catheter plus voiding or foley catheter output) should be 30
ml/hour.
40. Answer: 1. Ambulation stimulates peristalsis. A client with paralytic ileus is kept NPO until
peristalsis returns. Intravenous fluid infusion is a routine postoperative order that does not have any
effect on preventing paralytic ileus. A stool softener will not stimulate peristalsis.
41. Answer: 2. The decrease in urinary output may indicate inadequate renal perfusion and should be
reported immediately. Urine output of 30 ml/hour or greater is considered acceptable. A slight
elevation in temperature is expected after surgery. Peristalsis returns gradually, usually the second or
third day after surgery. Bowel sounds will be absent until then. A small amount of serous sanguineous
drainage is to be expected.
42. Answer: 1. Because a high-purine diet contributes to the formation of uric acid, a low-purine diet is
advocated. An alkaline-ash diet is also advocated, because uric acid crystals are more likely to develop
in acid urine. Foods that may be eaten as desired in a low-purine diet include milk, all fruits, tomatoes,
cereals, and corn. Food allowed on an alkaline-ash diet include milk, fruits (except cranberries, plums,
and prunes), and vegetables (especially legumes and green vegetables). Gravy, chicken, and liver are
high in purine.
43. Answer: 2. Allopurinol is used to treat renal calculi composed of uric acid. Side effects of allopurinol
include drowsiness, maculopapular rash, anemia, abdominal pain, nausea, vomiting, and bone marrow
depression. Clients should be instructed to report skin rashes and any unusual bleeding or bruising.
Retinopathy, nasal congestion, and dizziness are not side effects of allopurinol.
44. Answer: 4. By inhibiting uric acid synthesis, allopurinol decreases its excretion. The drugs
effectiveness is assessed by evaluating for a decreased serum uric acid concentration. Allopurinol does
not alter the level of alkaline phosphatase, not does it affect urinary calcium excretion or the serum
calcium level.
45. Answer: 3. Stress incontinence is the involuntary loss of urine during such activities as coughing,
sneezing, laughing, or physical exertion. These activities increase abdominal and detrusor pressure. A
strong urge to urinate is associated with urge incontinence. Overdistention of the bladder can lead to
overflow incontinence. Obstruction of the urethra can lead to urinary retention.

144

46. Answer: 2. The history of three pregnancies is most likely the cause of the clients current episodes
of stress incontinence. The clients fluid intake, age, or history of swimming would not create an
increase in intra-abdominal pressure.
47. Answer: 2. Clients with stress incontinence are encouraged to avoid substances such as caffeine
and alcohol which are bladder irritants. Emotional stressors do not cause stress incontinence. It is
caused most commonly be relaxed pelvic musculature. Wearing girdles is not contraindicated.
Although clients may be inclined to limit physical exertion to avoid incontinence episodes, they should
be encouraged to seek treatment instead of limiting their activities.
48. Answer: 3. A characteristic of urge incontinence is involuntary urination with little or no warning.
The inability to empty the bladder is urinary retention. Loss of urine when coughing occurs with stress
incontinence. Frequent dribbling of urine is common in male clients after some types of prostate
surgery or may occur in women after the development of vesicovaginal or urethrovaginal fistula.
49. Answer: 1. The best way to assess for a distended bladder in either a male or female client is to
check for a rounded swelling above the pubis. The swelling represents the distended bladder rising
above the pubis into the abdominal cavity. Dullness does not indicate a distended bladder. The client
might experience tenderness or pressure above the symphysis. No urine discharge is expected; the
urine flow is blocked by the enlarged prostate.
50. Answer: 3. Rapid emptying of an overdistended bladder may cause hypotension and shock due to
the sudden change of pressure within the abdominal viscera. Previously, removing no more than 1,000
ml at one time was the standard of practice, but this is no longer thought to be necessary as long as
the overdistended bladder is emptied slowly.
51. Answer: 1. The primary reason for taping an indwelling catheter to a male client soothe penis is
held in a lateral position to prevent pressure at the penoscrotal angle. Prolonged pressure at the
penoscrotal angle can cause a ureterocutaneous fistula.
52. Answer: 3. The prostate gland is located below the bladder and surrounds the urethra. It serves
one primary purpose: to produce a secretion that aids in the nourishment and passage of sperm.
53. Answer: 3. Atrovent is a bronchodilator, and its anticholinergic effects can aggravate urinary
retention. Glucophage and BuSpar do not affect the urinary system; timolol does not have a systemic
effect.
54. Answer: 4. If paralysis of vasomotor nerves in the upper spinal cord occurs when spinal anesthesia
is used, the client is likely to develop respiratory paralysis. Artificial ventilation is required until the
effects of the anesthesia subside. Convulsions, cardiac arrest, and renal shutdown are not likely results
of spinal anesthesia.
55. Answer: 3. Terazosin (Hytrin) is an antihypertensive drug that is also used in the treatment of BPH.
Blood pressure must be monitored to ensure that the client does not develop hypotension, syncope, or
postural hypotension. The client should be instructed to change positions slowly. Urinary nitrites, white
blood cell count, and pulse rate are not affected by terazosin.
56. Answer: 3. The decision made by the surgeon to insert a catheter after a TURP or prostatectomy
depends on the amount of bleeding that is expected after the procedure. During continuous bladder
irrigation after a TURP or prostatectomy, the rate at which the solution enters the bladder should be
increased when the drainage becomes brighter red. The color indicates the presence of blood.
Increasing the flow of irrigating solution helps flush the catheter well so clots do not plug it. There
would be no reason to increase the flow rate when the return is continuous or when the return appears
cloudy and dark yellow. Increasing the flow would be contraindicated when there is no return of urine
and irrigating solution.
57. Answer: 1. Deficient Fluid Volume is a priority diagnosis, because the client needs to drink a large
amount of fluid to keep the urine clear. The urine should be almost without color. About 2 weeks after a

145

TURP, when desiccated tissue is sloughed out, a secondary hemorrhage could occur. The client should
be instructed to call the surgeon or go to the ED if at any time the urine turns bright red. The client is
not specifically at risk for nutritional problems after a TURP. The client is not specifically at risk for
nutritional problems after a TURP. The client is not specifically at risk for impaired tissue integrity
because there is no external incision, and the client is not specifically at risk for airway problems
because the procedure is done after spinal anesthesia.
58. Answer: 2. The most specific examination to determine whether a malignancy extends outside of
the prostatic capsule is a study of the serum acid phosphatase level. The level increases when a
malignancy has metastasized. The prostate specific antigen (PSA) determination and a digital rectal
examination are done when screening for prostate cancer. Serum creatinine level, total nonprotein
nitrogen level, and endogenous creatinine clearance time give information about kidney function, not
prostate malignancy.
59. Answer: 4. Increased Blood Glucose Level.
60. Answer: 4. Elevated BUN and Creatinine.

Introduction
This is a 30-item NCLEX style questionnaire about the concept of Sleep.

Topics

All about Sleep

REM Sleep, Non-REM Sleep

Hypnotics and Sedatives

Guidelines

Read each question carefully and choose the best answer.

You are given one minute per question. Spend your time wisely!

Answers and rationales are given below. Be sure to read them.

If you need more clarifications, please direct them to the comments section.

Questions
1. To validate the suspicion that a married male client has sleep apnea the nurse first:
1. Asks the client if he experiences apnea in the middle of the night
2. Questions the spouse if she is awakened by her husbands snoring
3. Places the client on a continuous positive airway pressure (CPAP) device
4. Schedules the client for a sleep test
2. When analgesics are ordered for a client with obstructive sleep apnea (OSA) following
surgery, the nurse is most concerned about:
1. Nonsteroidal antiinflammatory drugs (NSAIDs)
2. Opioids
3. Anticonvulsants
4. Antidepressants
5. Adjuvants
3. The nurse finds a client sleep walking down the unit hallway. An appropriate
intervention the nurse implements is:

146

1. Asking the client what he or she is doing and call for help
2. Quietly approaching the client and then loudly calling his or her name
3. Lightly tapping the client on the shoulder and leading him or her back to bed
4. Blocking the hallway with chairs and seating the client
4. The nurse is sure to implement strategies to reduce noise on the unit particularly on the
______ night of admission, when the client is especially sensitive to hospital noises.
1. 1st
2. 2nd
3. 3rd
4. 4th
5. Which of the following medications are the safest to administer to adults needing
assistance in falling asleep?
1. Sedatives
2. Hypnotics
3. Benzodiazepines
4. Anti-anxiety agents
6. To assist an adult client to sleep better the nurse recommends which of the following?
(Select all that apply.)
1. Drinking a glass of wine just before retiring to bed
2. Eating a large meal 1 hour before bedtime
3. Consuming a small glass of warm milk at bedtime
4. Performing mild exercises 30 minutes before going to bed
7. The nurse recognizes that a client is experiencing insomnia when the client reports
(select all that apply):
1. Extended time to fall asleep
2. Falling asleep at inappropriate times
3. Difficulty staying asleep
4. Feeling tired after a nights sleep
8. The nurse teaches the mother of a newborn that in order to prevent sudden infant death
syndrome (SIDS) the best position to place the baby after nursing is (select all that apply):
1. Prone
2. Side-lying
3. Supine
4. Fowlers
9. When assessing a client for obstructive sleep apnea (OSA), the nurse understands the
most common symptom is:
1. Headache
2. Early awakening
3. Impaired reasoning
4. Excessive daytime sleepiness
10. The nurse understands that the most vivid dreaming occurs during:
1. REM sleep
2. Stage 1 NREM
3. Stage 4 NREM
4. Transition period from NREM to REM sleep
11. A client taking a beta adrenergic blockers for HTN can experience interference with
sleep patterns such as:

147

1. Nocturia
2. Increased daytime sleepiness
3. Increased awakening from sleep
4. Increased difficulty falling asleep
12. Narcolepsy can be best explained as:
1. A sudden muscle weakness during exercise
2. Stopping breathing for short intervals during sleep
3. Frequent awakenings during the night
4. An overwhelming wave of sleepiness and falling asleep
13. A nursing measure to promote sleep in school-age children is to:
1. Make sure the room is dark and quiet
2. Encourage evening exercise
3. Encourage television watching
4. Encourage quiet activities prior to bed time.
14. A female client verbalizes that she has been having trouble sleeping and feels wide
awake as soon as getting into bed. The nurse recognizes that there are many interventions
the promote sleep. Check all that apply.
1. Eat a heavy snack before bedtime
2. Read in bed before shutting out the light
3. Leave the bedroom if you are unable to sleep
4. Drink a cup of warm tea with milk at bedtime
5. Exercise in the afternoon rather than the evening
6. Count backwards from 100 to 0 when your mind is racing.
15. A client has a diagnosis of primary insomnia. Before assessing this client, the nurse
recalls the numerous causes of this disorder. Select all that apply:
1. Chronic stress
2. Severe anxiety
3. Generalized pain
4. Excessive caffeine
5. Chronic depression
6. Environmental noise
16. A hospitalized client is prescribed chloral hydrate (Noctec). The nurse includes which
action in the plan of care?
1. Monitor apical heart rate every 2 hours
2. Monitor blood pressure every 4 hours
3. Instruct the client to call for ambulation assistance
4. Clear a path to the bathroom at bedtime.
17. Select all that apply to the use of barbiturates in treating insomnia:
1. Barbiturates deprive people of NREM sleep
2. Barbiturates deprive people of REM sleep
3. When the barbiturates are discontinued, the NREM sleep increases.
4. When the barbiturates are discontinued, the REM sleep increases.
5. Nightmares are often an adverse effect when discontinuing barbiturates.
18. Select all that apply that is appropriate when there is a benzodiazepine overdose:
1. Administration of syrup of ipecac
2. Gastric lavage
3. Activated charcoal and a saline cathartic

148

4. Hemodialysis
5. Administration of Flumazenil
19. A patient is admitted to the emergency department with an overdose of a
benzodiazepine. The nurse immediately prepares to administer which of the following
antidotes from the emergency drug cart?
1. naloxone (Narcan)
2. naltrexone (ReVia)
3. nalmefene (Revex)
4. flumazenil (Romazicon)
20. Older adults who take long-acting sedatives or hypnotics are likely to experience:
1. Hallucinations
2. Ataxia
3. Alertness
4. Dyspnea
21. Which nursing diagnosis is appropriate for a patient who has received a sedativehypnotic agent?
1. Alteration in tissue perfusion
2. Fluid volume excess
3. Risk for injury
4. Risk for infection
22. A patient is admitted to the emergency department with an overdose of a barbiturate.
The nurse immediately prepares to administer which of the following from the emergency
drug cart?
1. naloxone HCl (Narcan
2. activated charcoal
3. flumazenil (Romazicon)
4. ipecac syrup
23. During patient teaching, the nurse explains the difference between a sedative and
hypnotic by stating:
1. Sedatives are much stronger than hypnotic drugs and should only be used for short periods of
time.
2. Sedative drugs induce sleep, whereas hypnotic drugs induce a state of hypnosis.
3. Most drugs produce sedation at low doses and sleep (the hypnotic effect) at higher doses.
4. There really is no difference; the terms are used interchangeably.
24. The patients chart notes the administration of dantrolene (Dantrium) immediately
postoperatively. The nurse suspects that the patient experienced:
1. Delirium tremens
2. Malignant hyperthermia
3. A tonic-clonic seizure
4. Respiratory arrest
25. Which of the following is an important nursing action for the administration of a
benzodiazepine as a sedative-hypnotic agent?
1. Use IM dosage forms for longer duration
2. Administer safely with other CNS depressants for insomnia
3. Monitor geriatric patients for the common occurrence of paradoxical reactions.
4. Evaluate for physical dependence that occurs within 48 hours of beginning the drug.

149

26. Pediatric and geriatric patients often react with more sensitivity to CNS depressants.
This type of sensitivity manifests itself in the development of which type of reaction?
1. Idiopathic
2. Teratogenic
3. Paradoxical
4. Psychogenic
27. Which of the following is an appropriate nursing intervention for patients who are
receiving CNS depressants?
1. Prevent any activity within the hospital setting while on oral muscle relaxants
2. Make sure that the patient knows that sedation should be minimal with these agents.
3. Cardiovascular stimulation, a common side effect, would lead to hypertension
4. Make sure the patients call light is close by in case of the need for assistance with activities.
28. Which of the following conditions characterizes rapid eye movement (REM) sleep?
1. Disorientation and disorganized thinking
2. Jerky limb movements and position changes
3. Pulse rate slowed by 5 to 10 beats/minute
4. Highly active brain and physiological activity levels.
29. Which of the following sleep disorders is the most prevalent?
1. Hypersomnia
2. Insomnia
3. Parasomnia
4. Sleep-awake schedule disturbance.
30. Which of the following substances is a natural hormone produced by the pineal gland
that induces sleep?
1. Amphetamine
2. Melatonin
3. Methylphenidate
4. Pemoline

Answers and Rationale


1. Answer: 4. (2- Although this is a diagnostic tool, the first thing the nurse would do is question the
spouse. This may lead to determining whether more tests are needed).
2. Answer: 2. Clients with obstructive sleep apnea are particularly sensitive to opioids. Thus the risk of
respiratory depression is increased. The nurse must recognize that clients with OSA should start out
receiving very low doses of opioids.
3. Answer: 3. The nurse should not startle the client but should gently awaken the client and lead him
or her back to bed.
4. Answer: 1. The client is most sensitive to noise in the hospital setting the first night because
everything is new. This represents sensory overload, which interferes with sleep and decreases rapid
eye movement (REM) as well as total sleep time.
5. Answer: 3. The group of drugs that are the safest are the benzodiazepines. They facilitate the action
of the neurons in the central nervous system (CNS) that suppress responsiveness to stimulation,
therefore decreasing levels of arousal.
6. Answer: 3. A small glass of milk relaxes the body and promotes sleep.
7. Answer: 1, 3, and 4. These symptoms are often reported by clients with insomnia. Clients report
nonrestorative sleep. Arising once at night to urinate (nocturia) is not in and of itself insomnia.

150

8. Answer: 2 and 3. Research demonstrate that the occurrence of SIDS is reduced with these two
positions.
9. Answer: 4. Excessive daytime sleepiness is the most common complaint of people with OSA. Persons
with severe OSA may report taking daytime naps and experiencing a disruption in their daily activities
because of sleepiness.
10. Answer: 1. Although dreams occur during both NREM and REM sleep, the dreams of REM sleep are
more vivid and elaborate and are believed to be functionally important to learning, memory
processing, and adaptation to stress.
11. Answer: 2. Beta Blockers can cause nightmares, insomnia, and awakenings from sleep.
12. Answer: 4. Narcolepsy is a dysfunction of mechanisms that regulate the sleep and wake states.
Excessive daytime sleepiness is the most common complaint associated with this disorder. During the
day a person may suddenly feel an overwhelming wave of sleepiness and fall asleep; REM sleep can
occur within 15 minutes of falling asleep.
13. Answer: 4. The amount of sleep needed during the school years is individualized because of
varying states of activities and levels of health. A 6-year old averages 11-12 hours of sleep nightly,
whereas an 11-year old sleeps about 9-10 hours. The 6- or 7-year old can usually be persuaded to go
to bed by encouraging quiet activities.
14. Answer: 3, 5, and 6. Lying in bed when one is unable to sleep increases frustration and anxiety
which further impede sleep; other activities, such as reading or watching television, should not be
conducted in bed. Counting backwards requires minimal concentration but it is enough to interfere
with thoughts that distract a person from falling asleep.
15. Answer: 1, 4, and 6. Acute or primary insomnia is caused by emotional or physical discomfort not
caused by the direct physiologic effects of a substance or a medical condition. Excessive caffeine
intake is an example of disruptive sleep hygiene; caffeine is a stimulant that inhibits sleep.
Environmental noise causes physical and/or emotional and therefore is related to primary insomnia.
16. Answer: 3. Chloral hydrate is a sedative. This medication does not affect cardiac function. Blood
pressure changes are not significant with the use of this medication. A client should call for assistance
to the bathroom at night. Additionally, the client may experience residual daytime sedation; therefore,
the nurse should instruct the client to call for ambulation assistance during the daytime hours.
17. Answer: 2, 4, and 5. Barbiturates deprive people of REM sleep. When the barbiturate is stopped
and REM sleep once again occurs, a rebound phenomenon occurs. During this phenomenon, the
persons dream time constitutes a larger percentage of the total sleep pattern, and the dreams are
often nightmares.
18. Answer: 2, 3, and 5. If ingestion is recent, decontamination of the GI system is indicated. The
administration of syrup of ipecac is contraindicated because of aspiration risks related to sedation.
Gastric lavage is generally the best and most effective means of gastric decontamination. Activated
charcoal and a saline cathartic may be administered to remove any remaining drug. Hemodialysis is
not useful in the treatment of benzodiazepine overdose. Flumazenil can be used to acutely reverse the
sedative effects of benzodiazepines, though this is normally done only in cases of extreme overdose or
sedation.
19. Answer: 4. Flumazenil is the antidote for benzodiazepine overdoses.
20. Answer: 2. If longer-acting barbiturates are used in older adults, these clients may experience
daytime sedation, ataxia, and memory deficits.
21. Answer: 3. Sedative-hypnotics cause CNS depression, putting the patient at risk for injury.
22. Answer: 2. There is no antidote for barbiturates. The use of activated charcoal absorbs any drug in
the GI tract, preventing absorption.

151

23. Answer: 3. Many drugs have both sedative and hypnotic properties, with the sedative properties
evident at low doses and the hypnotic properties demonstrated at larger doses.
24. Answer: 2. Dantrolene is a direct-acting musculoskeletal muscle relaxant and is the drug of choice
to treat malignant hyperthermia, a complication of generalized anesthesia (remember intraoperative
nursing???)
25. Answer: 3
26. Answer: 3
27. Answer: 4
28. Answer: 4.Highly active brain and physiological activity levels characterize REM stage. Stages 3
and 4 of NREM sleep are characterized by disorientation and disorganization, During REM sleep, the
body movement ceases except for the eyes. The pulse rate slows by 5-10 beats/minute during NREM
sleep, not REM sleep.
29. Answer: 2. Approximately 1/3 of American adults have some type of sleep disorder, and insomnia is
the most common.
30. Answer: 2. Melatonin is a natural hormone that induces sleep. All the others are medications
classified as stimulants.

Introduction
Take the last part of our Nursing Pharmacology exam series with this 30-item exam. This exam focuses
more on providing interventions when adverse effects occurs.

Topics

Drug administration

Interventions for adverse effects

Guidelines

Read each question carefully.

Choose the best answer.

You are given 1 minute and 20 seconds for each question.

Answers & Rationales are given below. Be sure to read them!

Questions
1. Walter, a teenage patient is admitted to the hospital because of acetaminophen (Tylenol) overdose.
Overdoses of acetaminophen can precipitate life-threatening abnormalities in which of the following
organs?
a. Lungs
b. Liver
c. Kidney
d. Adrenal Glands
2. A contraindication for topical corticosteroid usage in a male patient with atopic dermatitis (eczema)
is:
a. Parasite infection.
b. Viral infection.
c. Bacterial infection.
d. Spirochete infection.

152

3. In infants and children, the side effects of first generation over-the-counter (OTC) antihistamines,
such as diphenhydramine (Benadryl) and hydroxyzine (Atarax) include:
a. Reyes syndrome.
b. Cholinergic effects.
c. Paradoxical CNS stimulation.
d. Nausea and diarrhea.
4. Reyes syndrome, a potentially fatal illness associated with liver failure and encephalopathy is
associated with the administration of which over-the-counter (OTC) medication?
a. acetaminophen (Tylenol)
b. ibuprofen (Motrin)
c. aspirin
d. brompheniramine/psudoephedrine (Dimetapp)
5. The nurse is aware that the patients who are allergic to intravenous contrast media are usually also
allergic to which of the following products?
a. Eggs
b. Shellfish
c. Soy
d. acidic fruits
6. A 13-month-old child recently arrived in the United States from a foreign country with his parents
and needs childhood immunizations. His mother reports that he is allergic to eggs. Upon further
questioning, you determine that the allergy to eggs is anaphylaxis. Which of the following vaccines
should he not receive?
a. Hepatitis B
b. inactivated polio
c. diphtheria, acellular pertussis, tetanus (DTaP)
d. mumps, measles, rubella (MMR)
7. The cell and Coombs classification system categorizes allergic reactions and is useful in describing
and classifying patient reactions to drugs. Type I reactions are immediate hypersensitivity reactions
and are mediated by:
a. immunoglobulin E (IgE).
b. immunoglobulin G (IgG).
c. immunoglobulin A (IgA).
d. immunoglobulin M (IgM).
8. Drugs can cause adverse events in a patient. Bone marrow toxicity is one of the most frequent types
of drug-induced toxicity. The most serious form of bone marrow toxicity is:
a. aplastic anemia.
b. thrombocytosis.
c. leukocytosis.
d. granulocytosis.
9. Serious adverse effects of oral contraceptives include:
a. Increase in skin oil followed by acne.
b. Headache and dizziness.
c. Early or mid-cycle bleeding.
d. Thromboembolic complications.
10. The most serious adverse effect of Alprostadil (Prostin VR pediatric injection) administration in
neonates is:

153

a. Apnea.
b. Bleeding tendencies.
c. Hypotension.
d. Pyrexia.
11. Mandy, a patient calls the clinic today because he is taking atrovastatin (Lipitor) to treat his high
cholesterol and is having pain in both of his legs. You instruct him to:
a. Stop taking the drug and make an appointment to be seen next week.
b. Continue taking the drug and make an appointment to be seen next week.
c. Stop taking the drug and come to the clinic to be seen today.
d. Walk for at least 30 minutes and call if symptoms continue.
12. Which of the following adverse effects is associated with levothyroxine (Synthroid) therapy?
a. Tachycardia
b. Bradycardia
c. Hypotension
d. Constipation
13. Which of the following adverse effects is specific to the biguanide diabetic drug metformin
(Glucophage) therapy?
a. Hypoglycemia
b. GI distress
c. Lactic acidosis
d. Somulence
14. The most serious adverse effect of tricyclic antidepressant (TCA) overdose is:
a. Seizures.
b. Hyperpyrexia.
c. Metabolic acidosis.
d. Cardiac arrhythmias.
15. The nurse is aware that the following solutions is routinely used to flush an IV device before and
after the administration of blood to a patient is:
a. 0.9 percent sodium chloride
b. 5 percent dextrose in water solution
c. Sterile water
d. Heparin sodium
16. Cris asks the nurse whether all donor blood products are cross-matched with the recipient to
prevent a transfusion reaction. Which of the following always require cross-matching?
a. packed red blood cells
b. platelets
c. plasma
d. granulocytes
17. A month after receiving a blood transfusion an immunocompromised male patient develops fever,
liver abnormalities, a rash, and diarrhea. The nurse would suspect this patient has:
a. Nothing related to the blood transfusion.
b. Graft-versus-host disease (GVHD).
c. Myelosuppression.
d. An allergic response to a recent medication.
18. Jonas comes into the local blood donation center. He says he is here to donate platelets only today.
The nurse knows this process is called:

154

a. Directed donation.
b. Autologous donation.
c. Allogenic donation.
d. Apheresis.
19. Nurse Bryan knows that the age group that uses the most units of blood and blood products is:
a. Premature infants.
b. Children ages 1-20 years.
c. Adults ages 21-64 years.
d. The elderly above age 65 years.
20. A child is admitted with a serious infection. After two days of antibiotics, he is severely neutropenic.
The physician orders granulocyte transfusions for the next four days. The mother asks the nurse why?
The nurse responds:
a. This is the only treatment left to offer the child.
b. This therapy is fast and reliable in treating infections in children.
c. The physician will have to explain his rationale to you.
d. Granulocyte transfusions replenish the low white blood cells until the body can produce its own.
21. A neighbor tells nurse Maureen he has to have surgery and is reluctant to have any blood product
transfusions because of a fear of contracting an infection. He asks the nurse what are his options. The
nurse teaches the person that the safest blood product is:
a. An allogenic product.
b. A directed donation product.
c. An autologous product.
d. A cross-matched product.
22. A severely immunocompromised female patient requires a blood transfusion. To prevent GVHD, the
physician will order:
a. Diphenhydramine hydrochloride (Benadryl).
b. The transfusion to be administered slowly over several hours.
c. Irradiation of the donor blood.
d. Acetaminophen (Tylenol).
23. Louie who is to receive a blood transfusion asks the nurse what is the most common type of
infection he could receive from the transfusion. The nurse teaches him that approximately 1 in
250,000 patients contract:
a. Human immunodeficiency disease (HIV).
b. Hepatitis C infection.
c. Hepatitis B infection.
d. West Nile viral disease.
24. A male patient with blood type AB, Rh factor positive needs a blood transfusion. The Transfusion
Service (blood bank) sends type O, Rh factor negative blood to the unit for the nurse to infuse into this
patient. The nurse knows that:
a. This donor blood is incompatible with the patients blood.
b. Premedicating the patient with diphenhydramine hydrochloride (Benadryl) and acetaminophen
(Tylenol) will prevent any transfusion reactions or side effects.
c. This is a compatible match.
d. The patient is at minimal risk receiving this product since it is the first time he has been transfused
with type O, Rh negative blood.
25. Dr. Rodriguez orders 250 milliliters of packed red blood cells (RBC) for a patient. This therapy is
administered for treatment of:

155

a. Thrombocytopenia.
b. Anemia.
c. Leukopenia.
d. Hypoalbuminemia.
26. A female patient needs a whole blood transfusion. In order for transfusion services (the blood bank)
to prepare the correct product a sample of the patients blood must be obtained for:
a. A complete blood count and differential.
b. A blood type and cross-match.
c. A blood culture and sensitivity.
d. A blood type and antibody screen.
27. A male patient needs to receive a unit of whole blood. What type of intravenous (IV) device should
the nurse consider starting?
a. A small catheter to decrease patient discomfort
b. The type of IV device the patient has had in the past, which worked well
c. A large bore catheter
d. The type of device the physician prefers
28. Dr. Smith orders a gram of human salt poor albumin product for a patient. The product is available
in a 50 milliliter vial with a concentration of 25 percent. What dosage will the nurse administer?
a. The nurse should use the entire 50 milliliter vial.
b. The nurse should determine the volume to administer from the physician.
c. This concentration of product should not be used.
d. The nurse will administer 4 milliliters.
29. Central venous access devices (CVADs) are frequently utilized to administer chemotherapy. What is
a distinct advantage of using the CVAD for chemotherapeutic agent administration?
a. CVADs are less expensive than a peripheral IV.
b. Once a week administration is possible.
c. Caustic agents in small veins can be avoided.
d. The patient or his family can administer the drug at home.
30. A female patients central venous access device (CVAD) becomes infected. Why would the
physician order antibiotics to be given through the line rather than through a peripheral IV line?
a. To prevent infiltration of the peripheral line
b. To reduce the pain and discomfort associated with antibiotic administration in a small vein
c. To lessen the chance of an allergic reaction to the antibiotic
d. To attempt to sterilize the catheter and prevent having to remove it

Answers & Rationale


Here are the answers and rationale for this exam. Counter check your answers to those below and tell
us your scores. If you have any disputes or need more clarification to a certain question, please direct
them to the comments section.
1.

Answer B. Acetaminophen is extensively metabolized by pathways in the liver. Toxic doses


of acetaminophen deplete hepatic glutathione, resulting in accumulation of the intermediate agent,
quinine, which leads to hepatic necrosis. Prolonged use of acetaminophen may result in an
increased risk of renal dysfunction, but a single overdose does not precipitate life-threatening
problems in the respiratory system, renal system, or adrenal glands.

2.

Answer B. Topical agents produce a localized, rather than systemic effect. When treating
atopic dermatitis with a steroidal preparation, the site is vulnerable to invasion by organisms.
Viruses, such as herpes simplex or varicella-zoster, present a risk of disseminated infection. Educate

156

the patient using topical corticosteroids to avoid crowds or people known to have infections and to
report even minor signs of an infection. Topical corticosteroid usage results in little danger of
concurrent infection with these agents.
3.

Answer C. Typically, first generation OTC antihistamines have a sedating effect because of
passage into the CNS. However, in some individuals, especially infants and children, paradoxical
CNS stimulation occurs and is manifested by excitement, euphoria, restlessness, and confusion. For
this reason, use of first generation OTC antihistamines has declined, and second generation product
usage has increased. Reyes syndrome is a systemic response to a virus. First generation OTC
antihistamines do not exhibit a cholinergic effect. Nausea and diarrhea are uncommon when first
generation OTC antihistamines are taken.

4.

Answer C. Virus-infected children who are given aspirin to manage pain, fever, and
inflammation are at an increased risk of developing Reyes syndrome. Use of acetaminophen has
not been associated with Reyes syndrome and can be safely given to patients with fever due to
viral illnesses. Ibuprofen adverse effects include GI irritation and bleeding, and in toxic doses, both
renal and hepatic failure are reported. However, ibuprofen has not been associated with the onset of
Reyes disease. Brompheniramine/psudoephedrine contains a first generation OTC antihistamine
and a decongestant. Neither agent has been associated with the development of Reyes syndrome.

5.

Answer B. Some types of contrast media contain iodine as an ingredient. Shellfish also
contain significant amounts of iodine. Therefore, a patient who is allergic to iodine will exhibit an
allergic response to both iodine containing contrast media and shellfish. These products do not
contain iodine.

6.

Answer D. The measles portion of the MMR vaccine is grown in chick embryo cells. The
current MMR vaccine does not contain a significant amount of egg proteins, and even children with
dramatic egg allergies are extremely unlikely to have an anaphylactic reaction. However, patients
that do respond to egg contact with anaphylaxis should be in a medically controlled setting where
full resuscitation efforts can be administered if anaphylaxis results. The vaccines in options a,b and
c do not contain egg protein.

7.

Answer A. IgE, the least common serum immunoglobulin (Ig) binds very tightly to receptors
on basophils and mast cells and is involved in allergic reactions. Binding of the allergen to the IgE on
the cells results in the release of various pharmacological mediators that result in allergic
symptoms. IgG is the major Ig (75 percent of serum Ig is IgG). Most versatile Ig because it is capable
of carrying out all of the functions of Ig molecules. IgG is the only class of Ig that crosses the
placenta. It is an opsonin, a substance that enhances phagocytosis. IgA, the second most common
serum Ig is found in secretions (tears, saliva, colostrum, and mucus). It is important in local
(mucosal) immunity. IgM, the third most common serum Ig, is the first Ig to be made by the fetus
and the first Ig to be made by a virgin B cell when it is stimulated by antigen. IgM antibodies are
very efficient in leading to the lysis of microorganisms.

8.

Answer A. Aplastic anemia is the result of a hypersensitivity reaction and is often irreversible.
It leads to pancytopenia, a severe decrease in all cell types: red blood cells, white blood cells, and
platelets. A reduced number of red blood cells causes hemoglobin to drop. A reduced number of
white blood cells make the patient susceptible to infection. And, a reduced number of platelets
cause the blood not to clot as easily. Treatment for mild cases is supportive. Transfusions may be
necessary. Severe cases require a bone marrow transplant. Option 2 is an elevated platelet count.
Option 3 is an elevated white count. Option 4 is an elevated granulocyte count. A granulocyte is a
type of white blood cell.

9.

Answer D. Oral contraceptives have been associated with an increased risk of stroke,
myocardial infarction, and deep vein thrombosis. These risks are increased in women who smoke.

157

Increased skin oil and acne are effects of progestin excess. Headache and dizziness are effects of
estrogen excess. Early or mid-cycle bleeding are effects of estrogen deficiency.
10.

Answer A. All items are adverse reactions of the drug. However, apnea appearing during the
first hour of drug infusion occurs in 10-12 percent of neonates with congenital heart defects.
Clinicians deciding to utilize alprostadil must be prepared to intubate and mechanically ventilate the
infant. Careful monitoring for apnea or respiratory depression is mandatory. In some institutions,
elective intubation occurs prior to initiation of the medication.

11.

Answer C. Muscle aches, soreness, and weakness may be early signs of myopathy such as
rhabdomyolysis associated with the HMG-CoA reducatase class of antilipemic agents. This patient
will need an immediate evaluation to rule out myopathy. Additional doses may exacerbate the
problem. Exercise will not reverse myopathy and delays diagnosis.

12.

Answer A. Levothyroxine, especially in higher doses, can induce hyperthyroid-like symptoms


including tachycardia. An agent that increases the basal metabolic rate would not be expected to
induce a slow heart rate. Hypotension would be a side effect of bradycardia. Constipation is a
symptom of hypothyroid disease.

13.

Answer C. Lactic acidosis is the most dangerous adverse effect of metformin administration
with death resulting in approximately 50 percent of individuals who develop lactic acidosis while on
this drug. Metformin does not induce insulin production; thus, administration does not result in
hypoglycemic events. Some nausea, vomiting, and diarrhea may develop but is usually not severe.
NVD is not specific for metformin. Metformin does not induce sleepiness.

14.

Answer D. Excessive ingestion of TCAs result in life-threatening wide QRS complex


tachycardia. TCA overdose can induce seizures, but they are typically not life-threatening. TCAs do
not cause an elevation in body temperature. TCAs do not cause metabolic acidosis.

15.

Answer A. 0.9 percent sodium chloride is normal saline. This solution has the same osmolarity
as blood. Its use prevents red cell lysis. The solutions given in options 2 and 3 are hypotonic
solutions and can cause red cell lysis. The solution in option 4 may anticoagulate the patient and
result in bleeding.

16.

Answer A. Red blood cells contain antigens and antibodies that must be matched between
donor and recipient. The blood products in options 2-4 do not contain red cells. Thus, they require
no cross-match.

17.

Answer B. GVHD occurs when white blood cells in donor blood attack the tissues of an
immunocompromised recipient. This process can occur within a month of the transfusion. Options 1
and 4 may be a thought, but the nurse must remember that immunocompromised transfusion
recipients are at risk for GVHD.

18.

Answer D. The process of apheresis involves removal of whole blood from a donor. Within an
instrument that is essentially designed as a centrifuge, the components of whole blood are
separated. One of the separated portions is then withdrawn, and the remaining components are
retransfused into the donor. Directed donation is collected from a blood donor other than the
recipient, but the donor is known to the recipient and is usually a family member or friend.
Autologous donation is the collection and reinfusion of the patients own blood. Allogenic donation is
collected from a blood donor other than the recipient.

19.

Answer D. People older than 65 years use 43 percent of donated blood. This number is
expected to increase as the population ages.

20.

Answer D. Granulocyte (neutrophil) replacement therapy is given until the patients blood
values are normal and he is able to fight the infection himself. Options 1 and 3 are not therapeutic
responses. The treatment in option 2 takes days and is not always able to prevent morbidity and
mortality.

158

21.

Answer C. This process is the collection and reinfusion of the patients own blood. It is
recommended by the American Medical Associations Council on Scientific Affairs as the safest
product since it eliminates recipient incompatibility and infection. The product in option 1 is
collected from a blood donor other than the recipient. The process in option 2 is also collected from
a blood donor other than the recipient, but the donor is known to the recipient and is usually a
family member or friend. Cross-matching significantly enhances compatibility. It does not detect
infection.

22.

Answer C. This process eliminates white blood cell functioning, thus, preventing GVHD.
Diphenhydramine HCl is an antihistamine. Its use prior to a blood transfusion decreases the
likelihood of a transfusion reaction. Option 2 will not prevent GVHD. Use of acetaminophen prevents
and treats the common side effects of blood administration caused by the presence of white blood
cells in the transfusion product: fever, headache, and chills.

23.

Answer C. Hepatitis B is the most common infection spread via blood transfusion. Donors are
screened by a questionnaire that includes symptoms. The donated blood is also tested for infection.
The risk of infection with the agents in options 2 and 3 has decreased to approximately 1 in 2 million
secondary to donor questioning and donor blood testing. The incidence of West Nile viral
transmission is unknown, but donor infection is still relatively rare.

24.

Answer C. Type O, Rh negative blood has none of the major antigens and is safely
administered to patients of all blood types. It is also known as the universal donor. Premedicating
with these agents will not prevent a major transfusion reaction if the blood type and Rh factors of
the donor blood are incompatible with the recipients blood.

25.

Answer B. A red blood cell transfusion is used to correct anemia in patients in which the low
red blood cell count must be rapidly corrected. RBC transfusion will not correct a low platelet count.
RBC transfusion will not correct a low white blood cell count. Packed RBCs contain very little plasma
and, thus, only a small amount of albumin. This amount will not correct low albumin levels.

26.

Answer B. This is needed to utilize the correct type of donor blood and to match the donor
product with the patient. Incompatible matches would result in severe adverse events and possible
death. The tests in options 1 and 3 are unnecessary. The test in option 4 is utilized to determine the
patients blood type and presence of antibodies to blood antigens. It does not determine donor
blood compatibility with the patient.

27.

Answer C. Large bore catheters prevent damage to blood components and are less likely to
develop clotting problems than a small bore catheter. The nurse should determine the correct
device without asking the patient what type has been used before or asking the physician which
type he prefers and start the IV.

28.

Answer D. A 25 percent solution contains one quarter of a gram per milliliter. Thus, the nurse
will administer 4 milliliters to provide a complete gram of albumin. The volume in option 1 would
provide 12.5 grams of albumin. The nurse should determine the volume. It is unnecessary to seek
the answer from the physician. A 25 percent solution is an acceptable product and can safely be
used.

29.

Answer C. Many chemotherapeutic drugs are vesicants (highly active corrosive materials that
can produce tissue damage even in low concentrations). Extravasations of a vesicant can result in
significant tissue necrosis. Administration into a large vein is optimal. CVADs are more expensive
than a peripheral IV. Dosing depends on the drug. IV chemotherapeutic agents are not administered
at home. They are given in an outpatient or clinic setting if not given during hospitalization.

30.

Answer D. Microorganisms that infect CVADs are often coagulase-negative staphylococci,


which can be eliminated by antibiotic administration through the catheter. If unsuccessful in
eliminating the microorganism, the CVAD must be removed. CVAD use lessens the need for

159

peripheral IV lines and, thus, the risk of infiltration. In this case however, the antibiotics are given to
eradicate microorganisms from the CVAD. CVAD use has this effect, but in this case, the antibiotics
are given through the CVAD to eliminate the infective agent. The third option would not occur.

Introduction
This is the third part of our Nursing Pharmacology exam series which focuses more on administration
of various drugs. Take our 30-item exam that will review you on the different concepts of drug
administration.

Topics

Drug administration

Interventions for adverse effects

Guidelines

Read each question carefully.

Choose the best answer.

You are given 1 minute and 20 seconds for each question.

Answers & Rationales are given below. Be sure to read them!

Questions
1. An infection in a central venous access device is not eliminated by giving antibiotics through the
catheter. How would bacterial glycocalyx contribute to this?
a. It protects the bacteria from antibiotic and immunologic destruction.
b. Glycocalyx neutralizes the antibiotic rendering it ineffective.
c. It competes with the antibiotic for binding sites on the microbe.
d. Glycocalyx provides nutrients for microbial growth.
2. Central venous access devices are beneficial in pediatric therapy because:
a. They dont frighten children.
b. Use of the arms is not restricted.
c. They cannot be dislodged.
d. They are difficult to see.
3. How can central venous access devices (CVADs) be of value in a patient receiving chemotherapy
who has stomatitis and severe diarrhea?
a. The chemotherapy can be rapidly completed allowing the stomatitis and diarrhea to resolve.
b. Crystalloid can be administered to prevent dehydration.
c. Concentrated hyperalimentation fluid can be administered through the CVAD.
d. The chemotherapy dose can be reduced.
4. Some central venous access devices (CVAD) have more than one lumen. These multi lumen
catheters:
a. Have an increased risk of infiltration.
b. Only work a short while because the small bore clots off.
c. Are beneficial to patient care but are prohibitively expensive.
d. Allow different medications or solutions to be administered simultaneously.
5. Some institutions will not infuse a fat emulsion, such as Intralipid, into central venous access
devices (CVAD) because:

160

a. Lipid residue may accumulate in the CVAD and occlude the catheter.
b. If the catheter clogs, there is no treatment other than removal and replacement.
c. Lipids are necessary only in the most extreme cases to prevent essential fatty acid (EFA) deficiency.
d. Fat emulsions are very caustic.
6. A male patient needs a percutaneously inserted central catheter (PICC) for prolonged IV therapy. He
knows it can be inserted without going to the operating room. He mentions that, at least the doctor
wont be wearing surgical garb, will he? How will the nurse answer the patient?
a. You are correct. It is a minor procedure performed on the unit and does not necessitate surgical
attire.
b. To decrease the risk of infection, the doctor inserting the PICC will wear a cap, mask, and sterile
gown and gloves.
c. It depends on the doctors preference.
d. Most doctors only wear sterile gloves, not a cap, mask, or sterile gown.
7. A male patient is to receive a percutaneously inserted central catheter (PICC). He asks the nurse
whether the insertion will hurt. How will the nurse reply?
a. You will have general anesthesia so you wont feel anything.
b. It will be inserted rapidly, and any discomfort is fleeting.
c. The insertion site will be anesthetized. Threading the catheter through the vein is not painful.
d. You will receive sedation prior to the procedure.
8. What volume of air can safely be infused into a patient with a central venous access device (CVAD)?
a. It is dependent on the patients weight and height.
b. Air entering the patient through a CVAD will follow circulation to the lungs where it will be absorbed
and cause no problems.
c. It is dependent on comorbidities such as asthma or chronic obstructive lung disease.
d. None.
9. Kent a new staff nurse asks her preceptor nurse how to obtain a blood sample from a patient with a
portacath device. The preceptor nurse teaches the new staff nurse:
a. The sample will be withdrawn into a syringe attached to the portacath needle and then placed into a
vacutainer.
b. Portacath devices are not used to obtain blood samples because of the risk of clot formation.
c. The vacutainer will be attached to the portacath needle to obtain a direct sample.
d. Any needle and syringe may be utilized to obtain the sample.
10. What is the purpose of tunneling (inserting the catheter 2-4 inches under the skin) when the
surgeon inserts a Hickman central catheter device? Tunneling:
a. Increases the patients comfort level.
b. Decreases the risk of infection.
c. Prevents the patients clothes from having contact with the catheter
d. Makes the catheter less visible to other people.
11. The primary complication of a central venous access device (CVAD) is:
a. Thrombus formation in the vein.
b. Pain and discomfort.
c. Infection.
d. Occlusion of the catheter as the result of an intra-lumen clot.
12. Nurse Blessy is doing some patient education related to a patients central venous access device.
Which of the following statements will the nurse make to the patient?
a. These type of devices are essentially risk free.
b. These devices seldom work for more than a week or two necessitating replacement.

161

c. The dressing should only the changed by your doctor.


d. Heparin in instilled into the lumen of the catheter to decrease the risk of clotting.
13. The chemotherapeutic DNA alkylating agents such as nitrogen mustards are effective because
they:
a. Cross-link DNA strands with covalent bonds between alkyl groups on the drug and guanine bases on
DNA.
b. Have few, if any, side effects.
c. Are used to treat multiple types of cancer.
d. Are cell cycle-specific agents.
14. Hormonal agents are used to treat some cancers. An example would be:
a. Thyroxine to treat thyroid cancer.
b. ACTH to treat adrenal carcinoma.
c. Estrogen antagonists to treat breast cancer.
d. Glucagon to treat pancreatic carcinoma.
15. Chemotherapeutic agents often produce a certain degree of myelosuppression including
leukopenia. Leukopenia does not present immediately but is delayed several days to weeks because:
a. The patients hemoglobin and hematocrit are normal.
b. Red blood cells are affected first.
c. Folic acid levels are normal.
d. The current white cell count is not affected by chemotherapy.
16. Currently, there is no way to prevent myelosuppression. However, there are medications available
to elicit a more rapid bone marrow recovery. An example is:
a. Epoetin alfa (Epogen, Procrit).
b. Glucagon.
c. Fenofibrate (Tricor).
d. Lamotrigine (Lamictal).
17. Estrogen antagonists are used to treat estrogen hormone-dependent cancer, such as breast
carcinoma. Androgen antagonists block testosterone stimulation of androgen-dependent cancers. An
example of an androgen-dependent cancer would be:
a. Prostate cancer.
b. Thyroid cancer.
c. Renal carcinoma.
d. neuroblastoma.
18. Serotonin release stimulates vomiting following chemotherapy. Therefore, serotonin antagonists
are effective in preventing and treating nausea and vomiting related to chemotherapy. An example of
an effective serotonin antagonist antiemetic is:
a. ondansetron (Zofran).
b. fluoxetine (Prozac).
c. paroxetine (Paxil).
d. sertraline (Zoloft).
19. Methotrexate, the most widely used antimetabolite in cancer chemotherapy does not penetrate the
central nervous system (CNS). To treat CNS disease this drug must be administered:
a. Intravenously.
b. Subcutaneously.
c. Intrathecally.
d. By inhalation.

162

20. Methotrexate is a folate antagonist. It inhibits enzymes required for DNA base synthesis. To prevent
harm to normal cells, a fully activated form of folic acid known as leucovorin (folinic acid; citrovorum
factor) can be administered. Administration of leucovorin is known as:
a. Induction therapy.
b. Consolidation therapy.
c. Pulse therapy.
d. Rescue therapy.
21. A male Patient is undergoing chemotherapy may also be given the drug allopurinol (Zyloprim,
Aloprim). Allopurinol inhibits the synthesis of uric acid. Concomitant administration of allopurinol
prevents:
a. Myelosuppression.
b. Gout and hyperuricemia.
c. Pancytopenia.
d. Cancer cell growth and replication
22. Superficial bladder cancer can be treated by direct instillation of the antineoplastic antibiotic agent
mitomycin (Mutamycin). This process is termed:
a. Intraventricular administration.
b. Intravesical administration.
c. Intravascular administration.
d. Intrathecal administration.
23. The most common dose-limiting toxicity of chemotherapy is:
a. Nausea and vomiting.
b. Bloody stools.
c. Myelosuppression.
d. Inability to ingest food orally due to stomatitis and mucositis.
24. Chemotherapy induces vomiting by:
a. Stimulating neuroreceptors in the medulla.
b. Inhibiting the release of catecholamines.
c. Autonomic instability.
d. Irritating the gastric mucosa.
25. Myeloablation using chemotherapeutic agents is useful in cancer treatment because:
a. It destroys the myelocytes (muscle cells).
b. It reduces the size of the cancer tumor.
c. After surgery, it reduces the amount of chemotherapy needed.
d. It destroys the bone marrow prior to transplant.
26. Anticipatory nausea and vomiting associated with chemotherapy occurs:
a. Within the first 24 hours after chemotherapy.
b. 1-5 days after chemotherapy.
c. Before chemotherapy administration.
d. While chemotherapy is being administered.
27. Medications bound to protein have the following effect:
a. Enhancement of drug availability.
b. Rapid distribution of the drug to receptor sites.
c. The more drug bound to protein, the less available for desired effect.
d. Increased metabolism of the drug by the liver.
28. Some drugs are excreted into bile and delivered to the intestines. Prior to elimination from the
body, the drug may be absorbed. This process is known as:

163

a. Hepatic clearance.
b. Total clearance.
c. Enterohepatic cycling.
d. First-pass effect.
29. An adult patient has been taking a drug (Drug A) that is highly metabolized by the cytochrome p450 system. He has been on this medication for 6 months. At this time, he is started on a second
medication (Drug B) that is an inducer of the cytochrome p-450 system. You should monitor this
patient for:
a. Increased therapeutic effects of Drug A.
b. Increased adverse effects of Drug B.
c. Decreased therapeutic effects of Drug A.
d. Decreased therapeutic effects of Drug B.
30. Epinephrine is administered to a female patient. The nurse should expect this agent to rapidly
affect:
a. Adrenergic receptors.
b. Muscarinic receptors.
c. Cholinergic receptors.
d. Nicotinic receptors.

Answers & Rationale


Here are the answers and rationale for this exam. Counter check your answers to those below and tell
us your scores. If you have any disputes or need more clarification to a certain question, please direct
them to the comments section.
1.

Answer C. Glycocalyx is a viscous polysaccharide or polypeptide slime that covers microbes.


It enhances adherence to surfaces, resists phagocytic engulfment by the white blood cells, and
prevents antibiotics from contacting the microbe. Glycocalyx does not have the effects in options BD.

2.

Answer B. The child can move his extremities and function in a normal fashion. This lessens
stress associated with position restriction and promotes normal activity. Fear may not be eliminated.
All lines can be dislodged. Even small catheters can be readily seen.

3.

Answer C. In patients unable to take oral nutrition, parenteral hyperalimentation is an option


for providing nutritional support. High concentrations of dextrose, protein, minerals, vitamins, and
trace elements can be provided. Dosing is not affected with options a and d. Crystalloid can provide
free water but has very little nutritional benefits. Hyperalimentation can provide free water and
considerable nutritional benefits.

4.

Answer D. A multilumen catheter contains separate ports and means to administer agents.
An agent infusing in one port cannot mix with an agent infusing into another port. Thus, agents that
would be incompatible if given together can be given in separate ports simultaneously.

5.

Answer A. Occlusion occurs with slow infusion rates and concurrent administration of some
medications. Lipid occlusions may be treated with 70 percent ethanol or with 0.1 mmol/mL NaOH.
Lipids provide essential fatty acids. It is recommended that approximately 4 percent of daily calories
be EFAs. A deficiency can quickly develop. Daily essential fatty acids are necessary for constant
prostaglandin production. Lipids are almost isotonic with blood.

6.

Answer B. Strict aseptic technique including the use of cap, mask, and sterile gown and
gloves is require when placing a central venous line including a PICC. Options A, C, and D are
incorrect statements. They increase the risk of infection.

164

7.

Answer C. Pain related to PICC insertion occurs with puncture of the skin. When inserting PICC
lines, the insertion site is anesthetized so no pain is felt. The patient will not receive general
anesthesia or sedation. Statement 2 is false. Unnecessary pain should be prevented.

8.

Answer D. Any air entering the right heart can lead to a pulmonary embolus. All air should be
purged from central venous lines; none should enter the patient.

9.

Answer A. A special portacath needle is used to access the portacath device. A syringe is
attached and the sample is obtained. One of the primary reasons for insertion of a portacath device
is the need for frequent or long-term blood sampling. A vacutainer will exert too much suction on
the central line resulting in collapse of the line. Only special portacath needles should be used to
access the portacath device.

10.

Answer B. The actual access to the subclavian vein is still just under the clavicle, but by
tunneling the distal portion of the catheter several inches under the skin the risk of migratory
infection is reduces compared to a catheter that enters the subclavian vein directly and is not
tunneled. The catheter is tunneled to prevent infection.

11.

Answer C. A foreign body in a blood vessel increases the risk of infection. Catheters that come
outside the body have an even higher risk of infection. Most infections are caused by skin bacteria.
Other infective organisms include yeasts and fungi. Options 1 and 4 are complications of a CVAD but
are not the primary problem. Once placed, these lines do not cause pain and discomfort.

12.

Answer D. A solution containing heparin is used to reduce catheter clotting and maintain
patency. The concentration of heparin used depends on the patients age, comorbidities, and the
frequency of catheter access/flushing. Although patients have few complications, the device is not
risk free. Patients may develop infection, catheter clots, vascular obstruction, pneumothorax,
hemothorax, or mechanical problems (catheter breakage). Strict adherence to protocol enhances
the longevity of central access devices. They routinely last weeks to months and sometimes years.
The patient will be taught how to perform dressing changes at home.

13.

Answer A. Alkylating agents are highly reactive chemicals that introduce alkyl radicals into
biologically active molecules and thereby prevent their proper functioning, replication, and
transcription. Alkylating agents have numerous side effects including alopecia, nausea, vomiting,
and myelosuppression. Nitrogen mustards have a broad spectrum of activity against chronic
lymphocytic leukemia, non-Hodgkins lymphoma, and breast and ovarian cancer, but they are
effective chemotherapeutic agents because of DNA cross-linkage. Alkylating agents are noncell
cycle-specific agents.

14.

Answer C. Estrogen antagonists are used to treat estrogen hormone-dependent cancer, such
as breast carcinoma. A well-known estrogen antagonist used in breast cancer therapy is tamoxifen
(Nolvadex). This drug, in combination with surgery and other chemotherapeutic drugs reduces
breast cancer recurrence by 30 percent. Estrogen antagonists can also be administered to prevent
breast cancer in women who have a strong family history of the disease. Thyroxine is a natural
thyroid hormone. It does not treat thyroid cancer. ACTH is an anterior pituitary hormone, which
stimulates the adrenal glands to release glucocorticoids. It does not treat adrenal cancer. Glucagon
is a pancreatic alpha cell hormone, which stimulates glycogenolysis and gluconeogenesis. It does
not treat pancreatic cancer.

15.

Answer D. The time required to clear circulating cells before the effect that chemotherapeutic
drugs have on precursor cell maturation in the bone marrow becomes evident. Leukopenia is an
abnormally low white blood cell count. Answers A-C pertain to red blood cells.

16.

Answer A. Epoetin alfa (Epogen, Procrit) is a recombinant form of endogenous erythropoietin,


a hematopoietic growth factor normally produced by the kidney that is used to induce red blood cell
production in the bone marrow and reduce the need for blood transfusion. Glucagon is a pancreatic

165

alpha cell hormone, which cause glycogenolysis and gluconeogenesis. Fenofibrate (Tricor) is an
antihyperlipidemic agent that lowers plasma triglycerides. Lamotrigine (Lamictal) is an
anticonvulsant.
17.

Answer A. Prostate tissue is stimulated by androgens and suppressed by estrogens. Androgen


antagonists will block testosterone stimulation of prostate carcinoma cells. The types of cancer in
options 2-4 are not androgen dependent.

18.

Answer A. Chemotherapy often induces vomiting centrally by stimulating the chemoreceptor


trigger zone (CTZ) and peripherally by stimulating visceral afferent nerves in the GI tract.
Ondansetron (Zofran) is a serotonin antagonist that bocks the effects of serotonin and prevents and
treats nausea and vomiting. It is especially useful in single-day highly emetogenic cancer
chemotherapy (for example, cisplatin). The agents in options 2-4 are selective serotonin reuptake
inhibitors. They increase the available levels of serotonin.

19.

Answer C. With intrathecal administration chemotherapy is injected through the theca of the
spinal cord and into the subarachnoid space entering into the cerebrospinal fluid surrounding the
brain and spinal cord. The methods in options A, B, and D are ineffective because the medication
cannot enter the CNS.

20.

Answer B. Leucovorin is used to save or rescue normal cells from the damaging effects of
chemotherapy allowing them to survive while the cancer cells die. Therapy to rapidly reduce the
number of cancerous cells is the induction phase. Consolidation therapy seeks to complete or
extend the initial remission and often uses a different combination of drugs than that used for
induction. Chemotherapy is often administered in intermittent courses called pulse therapy. Pulse
therapy allows the bone marrow to recover function before another course of chemotherapy is
given.

21.

Answer B. Prevent uric acid nephropathy, uric acid lithiasis, and gout during cancer therapy
since chemotherapy causes the rapid destruction of cancer cells leading to excessive purine
catabolism and uric acid formation. Allopurinol can induce myelosuppression and pancytopenia.
Allopurinol does not have this function.

22.

Answer B. Medications administered intravesically are instilled into the bladder.


Intraventricular administration involves the ventricles of the brain. Intravascular administration
involves blood vessels. Intrathecal administration involves the fluid surrounding the brain and spinal
cord.

23.

Answer C. The overall goal of cancer chemotherapy is to give a dose large enough to be
lethal to the cancer cells, but small enough to be tolerable for normal cells. Unfortunately, some
normal cells are affected including the bone marrow. Myelosuppression limits the bodys ability to
prevent and fight infection, produce platelets for clotting, and manufacture red blood cells for
oxygen portage. Even though the effects in options a, b, and d are uncomfortable and distressing to
the patient, they do not have the potential for lethal outcomes that myelosuppression has.

24.

Answer A. Vomiting (emesis) is initiated by a nucleus of cells located in the medulla called the
vomiting center. This center coordinates a complex series of events involving pharyngeal,
gastrointestinal, and abdominal wall contractions that lead to expulsion of gastric contents.
Catecholamine inhibition does not induce vomiting. Chemotherapy does not induce vomiting from
autonomic instability. Chemotherapy, especially oral agents, may have an irritating effect on the
gastric mucosa, which could result in afferent messages to the solitary tract nucleus, but these
pathways do not project to the vomiting center.

25.

Answer A. Myelo comes from the Greek word myelos, which means marrow. Ablation comes
from the Latin word ablatio, which means removal. Thus, myeloablative chemotherapeurtic agents
destroy the bone marrow. This procedure destroys normal bone marrow as well as the cancerous

166

marrow. The patients bone marrow will be replaced with a bone marrow transplant. Myelocytes are
not muscle cells Tumors are solid masses typically located in organs. Surgery may be performed to
reduce tumor burden and require less chemotherapy afterward.
26.

Answer C. Nausea and vomiting (N&V) are common side effects of chemotherapy. Some
patients are able to trigger these events prior to actually receiving chemotherapy by anticipating, or
expecting, to have these effects. N&V occurring post-chemotherapeutic administration is not an
anticipatory event but rather an effect of the drug. N&V occurring during the administration of
chemotherapy is an effect of the drug.

27.

Answer C. Only an unbound drug can be distributed to active receptor sites. Therefore, the
more of a drug that is bound to protein, the less it is available for the desired drug effect. Less drug
is available if bound to protein. Distribution to receptor sites is irrelevant since the drug bound to
protein cannot bind with a receptor site. Metabolism would not be increased. The liver will first have
to remove the drug from the protein molecule before metabolism can occur. The protein is then free
to return to circulation and be used again.

28.

Answer C. Drugs and drug metabolites with molecular weights higher than 300 may be
excreted via the bile, stored in the gallbladder, delivered to the intestines by the bile duct, and then
reabsorbed into the circulation. This process reduces the elimination of drugs and prolongs their
half-life and duration of action in the body. Hepatic clearance is the amount of drug eliminated by
the liver. Total clearance is the sum of all types of clearance including renal, hepatic, and respiratory.
First-pass effect is the amount of drug absorbed from the GI tract and then metabolized by the liver;
thus, reducing the amount of drug making it into circulation.

29.

Answer C. Drug B will induce the cytochrome p-450 enzyme system of the liver; thus,
increasing the metabolism of Drug A. Therefore, Drug A will be broken down faster and exert
decreased therapeutic effects. Drug A will be metabolized faster, thus reducing, not increasing its
therapeutic effect. Inducing the cytochrome p-450 system will not increase the adverse effects of
Drug B. Drug B induces the cytochrome p-450 system but is not metabolized faster. Thus, the
therapeutic effects of Drug B will not be decreased.

30.

Answer A. Epinephrine (adrenaline) rapidly affects both alpha and beta adrenergic receptors
eliciting a sympathetic (fight or flight) response. Muscarinic receptors are cholinergic receptors and
are primarily located at parasympathetic junctions. Cholinergic receptors respond to acetylcholine
stimulation. Cholinergic receptors include muscarinic and nicotinic receptors. Nicotinic receptors are
cholinergic receptors activated by nicotine and found in autonomic ganglia and somatic
neuromuscular junctions.

As a nurse, we need to have an extensive knowledge about the different drugs and their uses. But
how vast is your knowledge about the concepts covering Pharmacology? Take our 20-item exam
about this topic.

Topics

Pharmacology nursing

Drug computation and dosages


Administration of medicine

167

Guidelines

Read each question carefully.

Choose the best answer.


You are given 1 minute and 20 seconds for each question.
Answers & Rationales are given below. Be sure to read them!

Questions
1. A 2 year-old child is receiving temporary total parental nutrition (TPN) through a
central venous line. This is the first day of TPN therapy. Although all of the following
nursing actions must be included in the plan of care of this child, which one would be a
priority at this time?
a. Use aseptic technique during dressing changes
b. Maintain central line catheter integrity
c. Monitor serum glucose levels
d. Check results of liver function tests
2. Nurse Jamie is administering the initial total parenteral nutrition solution to a client.
Which of the following assessments requires the nurses immediate attention?
a. Temperature of 37.5 degrees Celsius
b. Urine output of 300 cc in 4 hours
c. Poor skin turgor
d. Blood glucose of 350 mg/dl
3. Nurse Susan administered intravenous gamma globulin to an 18 month-old child with
AIDS. The parent asks why this medication is being given. What is the nurses best
response?
a. It will slow down the replication of the virus.
b. This medication will improve your childs overall health status.
c. This medication is used to prevent bacterial infections.
d. It will increase the effectiveness of the other medications your child receives.
4. When caring for a client with total parenteral nutrition (TPN), what is the most
important action on the part of the nurse?
a. Record the number of stools per day
b. Maintain strict intake and output records
168

c. Sterile technique for dressing change at IV site


d. Monitor for cardiac arrhythmias
5. The nurse is administering an intravenous vesicant chemotherapeutic agent to a client.
Which assessment would require the nurses immediate action?
a. Stomatitis lesion in the mouth
b. Severe nausea and vomiting
c. Complaints of pain at site of infusion
d. A rash on the clients extremities
6. Nurse Celine is caring for a client with clinical depression who is receiving a MAO
inhibitor. When providing instructions about precautions with this medication, the nurse
should instruct the client to:
a. Avoid chocolate and cheese
b. Take frequent naps
c. Take the medication with milk
d. Avoid walking without assistance
7. While providing home care to a client with congestive heart failure, the nurse is asked
how long diuretics must be taken. The BEST response to this client should be:
a. As you urinate more, you will need less medication to control fluid.
b. You will have to take this medication for about a year.
c. The medication must be continued so the fluid problem is controlled.
d. Please talk to your physician about medications and treatments.
8. George, age 8, is admitted with rheumatic fever. Which clinical finding indicates to the
nurse that George needs to continue taking the salicylates he had received at home?
a. Chorea.
b. Polyarthritis.
c. Subcutaneous nodules.
d. Erythema marginatum.
9. An order is written to start an IV on a 74-year-old client who is getting ready to go to the
operating room for a total hip replacement. What gauge of catheter would best meet the
needs of this client?
a. 18
b. 20
169

c. 21 butterfly
d. 25
10. A client with an acute exacerbation of rheumatoid arthritis is admitted to the hospital
for treatment. Which drug, used to treat clients with rheumatoid arthritis, has both an
anti-inflammatory and immunosuppressive effect?
a. Gold sodium thiomalate (Myochrysine)
b. Azathioprine (Imuran)
c. Prednisone (Deltasone)
d. Naproxen (Naprosyn)
11. Which of the following is least likely to influence the potential for a client to comply
with lithium therapy after discharge?
a. The impact of lithium on the clients energy level and life-style.
b. The need for consistent blood level monitoring.
c. The potential side effects of lithium.
d. What the clients friends think of his need to take medication
12. Which of the following is least likely to influence the potential for a client to comply
with lithium therapy after discharge?
a. The impact of lithium on the clients energy level and life-style.
b. The need for consistent blood level monitoring.
c. The potential side effects of lithium.
d. What the clients friends think of his need to take medication.
13. The nurse is caring for an elderly client who has been diagnosed as having sundown
syndrome. He is alert and oriented during the day but becomes disoriented and disruptive
around dinnertime. He is hospitalized for evaluation. The nurse asks the client and his
family to list all of the medications, prescription and nonprescription, he is currently
taking. What is the primary reason for this action?
a. Multiple medications can lead to dementia
b. The medications can provide clues regarding his medical background
c. Ability to recall medications is a good assessment of the clients level of orientation.
d. Medications taken by a client are part of every nursing assessment.
14. A 25-year-old woman is in her fifth month of pregnancy. She has been taking 20 units of
NPH insulin for diabetes mellitus daily for six years. Her diabetes has been well controlled
with this dosage. She has been coming for routine prenatal visits, during which diabetic
170

teaching has been implemented. Which of the following statements indicates that the
woman understands the teaching regarding her insulin needs during her pregnancy?
a. Are you sure all this insulin wont hurt my baby?
b. Ill probably need my daily insulin dose raised.
c. I will continue to take my regular dose of insulin.
d. These finger sticks make my hand sore. Can I do them less frequently?
15. Mrs. Johansons physician has prescribed tetracycline 500 mg po q6h. While assessing
Mrs. Johansons nursing history for allergies, the nurse notes that Mrs. Johansons is also
taking oral contraceptives. What is the most appropriate initial nursing intervention?
a. Administer the dose of tetracycline.
b. Notify the physician that Mrs. Johanson is taking oral contraceptives.
c. Tell Mrs. Johanson, she should stop taking oral contraceptives since they are inactivated by
tetracycline.
d. Tell Mrs. Johanson, to use another form of birth control for at least two months.
16. An adult clients insulin dosage is 10 units of regular insulin and 15 units of NPH insulin
in the morning. The client should be taught to expect the first insulin peak:
a. as soon as food is ingested.
b. in two to four hours.
c. in six hours.
d. in ten to twelve hours.
17. An adult is hospitalized for treatment of deep electrical burns. Burn wound sepsis
develops and mafenide acetate 10% (Sulfamylon) is ordered bid. While applying the
Sulfamylon to the wound, it is important for the nurse to prepare the client for expected
responses to the topical application, which include:
a. severe burning pain for a few minutes following application.
b. possible severe metabolic alkalosis with continued use.
c. black discoloration of everything that comes in contact with this drug.
d. chilling due to evaporation of solution from the moistened dressings.
18. Ms.Clark has hyperthyroidism and is scheduled for a thyroidectomy. The physician has
ordered Lugols solution for the client. The nurse understands that the primary reason for
giving Lugols solution preoperatively is to:
a. decrease the risk of agranulocytosis postoperatively.
b. prevent tetany while the client is under general anesthesia.
171

c. reduce the size and vascularity of the thyroid and prevent hemorrhage.
d. potentiate the effect of the other preoperative medication so less medicine can be given while
the client is under anesthesia.
19. A two-year-old child with congestive heart failure has been receiving digoxin for one
week. The nurse needs to recognize that an early sign of digitalis toxicity is:
a. bradypnea.
b. failure to thrive.
c. tachycardia.
d. vomiting.
20. Mr. Bates is admitted to the surgical ICU following a left adrenalectomy. He is sleepy
but easily aroused. An IV containing hydrocortisone is running. The nurse planning care
for Mr. Bates knows it is essential to include which of the following nursing interventions at
this time?
a. Monitor blood glucose levels every shift to detect development of hypo- or hyperglycemia.
b. Keep flat on back with minimal movement to reduce risk of hemorrhage following surgery.
c. Administer hydrocortisone until vital signs stabilize, then discontinue the IV.
d. Teach Mr. Bates how to care for his wound since he is at high risk for developing
postoperative infection.

Answers & Rationale


Here are the answers and rationale for this exam. Counter check your answers to those below and
tell us your scores. If you have any disputes or need more clarification to a certain question,
please direct them to the comments section.
1. Answer C.
Monitor serum glucose levels. Hyperglycemia may occur during the first day or 2 as the child
adapts to the high-glucose load of the TPN solution. Thus, a chief nursing responsibility is blood
glucose testing.
2. Answer D.
Total parenteral nutrition formulas contain dextrose in concentrations of 10% or greater to supply
20% to 50% of the total calories. Blood glucose levels should be checked every 4 to 6 hours. A
sliding scale dose of insulin may be ordered to maintain the blood glucose level below 200mg/dl.

172

3. Answer C.
Intravenous gamma globulin is given to help prevent as well as to fight bacterial infections in
young children with AIDS.
4. Answer C.
Clients receiving TPN are very susceptible to infection. The concentrated glucose solutions are a
good medium for bacterial growth. Strict sterile
technique is crucial in preventing infection at IV infusion site.
5. Answer C.
A vesicant is a chemotherapeutic agent capable of causing blistering of tissues and possible
tissue necrosis if there is extravasation. These agents are irritants which cause pain along the vein
wall, with or without inflammation.
6. Answer A.
Foods high in tryptophan, tyramine and caffeine, such as chocolate and cheese may precipitate
hypertensive crisis.
7. Answer C.
This is the most therapeutic response and gives the client accurate information.
8. Answer B.
Chorea is the restless and sudden aimless and irregular movements of the extremities suddenly
seen in persons with rheumatic fever, especially girls. Polyarthritis is characterized by swollen,
painful, hot joints that respond to salicylates. Subcutaneous nodules are nontender swellings over
bony prominences sometimes seen in persons with rheumatic fever. Erythema marginatum is a
skin condition characterized by nonpruritic rash, affecting trunk and proximal extremities, seen
in persons with rheumatic fever.
9. Answer A.
Clients going to the operating room ideally should have an 18- gauge catheter. This is large
enough to handle blood products safely and to allow rapid administration of large amounts of
fluid if indicated during the perioperative period. An 18-gauge catheter is recommended. A 20gauge catheter is a second choice. A 21-gauge needle is too small and a butterfly too unstable for
a client going to surgery. A 25-gauge needle is too small.
10. Answer C.
Gold sodium thiomalate is usually used in combination with aspirin and nonsteroidal antiinflammatory drugs to relieve pain. Gold has an immunosuppressive affect. Azathioprine is used
for clients with life-threatening rheumatoid arthritis for its immunosuppressive effects.
Prednisone is used to treat persons with acute exacerbations of rheumatoid arthritis. This
173

medication is given for its anti-inflammatory and immunosuppressive effects. Naproxen is a


nonsteroidal anti-inflammatory drug. Immunosuppression does not occur.
11. Answer D.
The impact of lithium on the clients energy level and life style are great determinants to
compliance. The frequent blood level monitoring required is difficult for clients to follow for a
long period of time. Potential side effects such as fine tremor, drowsiness, diarrhea, polyuria,
thirst, weight gain, and fatigue can be disturbing to the client. While the clients social network
can influence the client in terms of compliance, the influence is typically secondary to that of the
other factors listed.
12. Answer D.
The impact of lithium on the clients energy level and life style are great determinants to
compliance. The frequent blood level monitoring required is difficult for clients to follow for a
long period of time. Potential side effects such as fine tremor, drowsiness, diarrhea, polyuria,
thirst, weight gain, and fatigue can be disturbing to the client. While the clients social network
can influence the client in terms of compliance, the influence is typically secondary to that of the
other factors listed.
13. Answer A.
Drugs commonly used by elderly people, especially in combination, can lead to dementia.
Assessment of the medication taken may or may not provide information on the clients medical
background. However, this is not the primary reason for assessing medications in a client who is
exhibiting sundown syndrome. Ability to recall medications may indicate short-term memory
and recall. However, that is not the primary reason for assessing medications in a client with
sundown syndrome. Medication history should be a part of the nursing assessment. In this client
there is an even more important reason for evaluating the medications taken.
14. Answer B.
The client starts to need increased insulin in the second trimester. This statement indicates a lack
of understanding. As a result of placental maturation and placental production of lactogen,
insulin requirements begin increasing in the second trimester and may double or quadruple by
the end of pregnancy. The client starts to need increased insulin in the second trimester. This
statement indicates a lack of understanding. Insulin doses depend on blood glucose levels. Finger
sticks for glucose levels must be continued.
15. Answer B.
The nurse should be aware that tetracyclines decrease the effectiveness of oral contraceptives.
The physician should be notified. The physician should be notified. Tetracycline decreases the
effectiveness of oral contraceptives. There may be an equally effective antibiotic available that
can be prescribed. Note on the clients chart that the physician was notified. The nurse should be
174

aware that tetracyclines decrease the effectiveness of oral contraceptives. The nurse should not
tell the client to stop taking oral contraceptives unless the physician orders this. The nurse should
be aware that tetracyclines decrease the effectiveness of oral contraceptives. If the physician
chooses to keep the client on tetracycline, the client should be encouraged to use another form of
birth control. The first intervention is to notify the physician.
16. Answer B.
The first insulin peak will occur two to four hours after administration of regular insulin. Regular
insulin is classified as rapid acting and will peak two to four hours after administration. The
second peak will be eight to twelve hours after the administration of NPH insulin. This is why a
snack must be eaten mid-morning and also three to four hours after the evening meal. The first
insulin peak will occur two to four hours after administration of regular insulin. The first insulin
peak will occur two to four hours after administration of regular insulin. The second peak will
occur eight to twelve hours after the administration of NPH insulin.
17. Answer A.
Mafenide acetate 10% (Sulfamylon) does cause burning on application. An analgesic may be
required before the ointment is applied. Mafenide acetate 10% (Sulfamylon) is a strong carbonic
anhydrase inhibitor that affects the renal tubular buffering system, resulting in metabolic
acidosis. Mafenide acetate 10% (Sulfamylon) does not cause discoloration. Silver nitrate
solution, another topical antibiotic used to treat burn sepsis, has the disadvantage of turning
everything it touches black. Mafenide acetate 10% (Sulfamylon) is an ointment that is applied
directly to the wound. It has the ability to diffuse rapidly through the eschar. The wound may be
left open or dry dressing may be applied. Silver nitrate solution is applied by soaking the wound
dressings and keeping them constantly wet, which may cause chilling and hypotension.
18. Answer C.
Doses of over 30 mg/day may increase the risk of agranulocytosis. Lugols solution does not act
to prevent tetany. Calcium is used to treat tetany. The client may receive iodine solution (Lugols
solution) for 10 to 14 days before surgery to decrease vascularity of the thyroid and thus prevent
excess bleeding. Lugols solution does not potentiate any other preoperative medication.
19. Answer D.
Bradypnea (slow breathing) is not associated with digitalis toxicity. Bradycardia is associated
with digitalis toxicity. Although children with congestive heart failure often have a related
condition of failure to thrive, it is not directly related to digitalis administration. It is more related
to chronic hypoxia. Tachycardia is not a sign of digitalis toxicity. Bradycardia is a sign of
digitalis toxicity. The earliest sign of digitalis toxicity is vomiting, although one episode does not
warrant discontinuing medication.

175

20. Answer A.
Hydrocortisone promotes gluconeogenesis and elevates blood glucose levels. Following
adrenalectomy the normal supply of hydrocortisone is interrupted and must be replaced to
maintain the blood glucose at normal levels. Care for the client following adrenalectomy is
similar to that for any abdominal operation. The client is encouraged to change position, cough,
and deep breathe to prevent postoperative complications such as pneumonia or thrombophlebitis.
Maintenance doses of hydrocortisone will be administered IV until the client is able to take it by
mouth and will be necessary for six months to two years or until the remaining gland recovers.
The client undergoing an adrenalectomy is at increased risk for infection and delayed wound
healing and will need to learn about wound care, but not at this time while he is in the ICU.

Introduction
As a nurse, we need to have an extensive knowledge about the different drugs and their uses. But how
vast is your knowledge about the concepts covering Pharmacology? Take our part two exam for this
topic.

Topics

Different drugs and their effects

Guidelines

Read each question carefully.

Choose the best answer.

You are given 1 minute and 20 seconds for each question.

Answers & Rationales are given below. Be sure to read them!

Questions
1. The nursery nurse is putting erythromycin ointment in the newborns eyes to prevent infection. She
places it in the following area of the eye:
A. under the eyelid
B. on the cornea.
C. in the lower conjunctival sac
D. by the optic disc.
2. The physician orders penicillin for a patient with streptococcal pharyngitis. The nurse administers
the drug as ordered, and the patient has an allergic reaction. The nurse checks the medication order
sheet and finds that the patient is allergic to penicillin. Legal responsibility for the error is:
A. only the nursesshe should have checked the allergies before administering the medication.
B. only the physiciansshe gave the order, the nurse is obligated to follow it.
C. only the pharmacistshe should alert the floor to possible allergic reactions.
D. the pharmacist, physician, and nurse are all liable for the mistake
3. James Perez, a nurse on a geriatric floor, is administering a dose of digoxin to one of his patients.
The woman asks why she takes a different pill than her niece, who also has heart trouble. James replies
that as people get older, liver and kidney function decline, and if the dose is as high as her nieces, the
drug will tend to:

176

A. have a shorter half-life.


B. accumulate.
C. have decreased distribution.
D. have increased absorption.
4. The nurse is administering augmentin to her patient with a sinus infection. Which is the best way for
her to insure that she is giving it to the right patient?
A. Call the patient by name
B. Read the name of the patient on the patients door
C. Check the patients wristband
D. Check the patients room number on the unit census list
5. The most important instructions a nurse can give a patient regarding the use of the
antibiotic ampicillin prescribed for her are to
A. call the physician if she has any breathing difficulties.
B. take it with meals so it doesnt cause an upset stomach.
C. take all of the medication prescribed even if the symptoms stop sooner.
D. not share the pills with anyone else.
6. Mr. Jessie Ray, a newly admitted patient, has a seizure disorder which is being treated with
medication. Which of the following drugs would the nurse question if ordered for him?
A. Phenobarbitol, 150 mg hs
B. Amitriptylene (Elavil), 10 mg QID.
C. Valproic acid (Depakote), 150 mg BID
D. Phenytoin (Dilantin), 100 mg TID
7. Mrs. Jane Gately has been dealing with uterine cancer for several months. Pain management is the
primary focus of her current admission to your oncology unit. Her vital signs on admission are BP
110/64, pulse 78, respirations 18, and temperature 99.2 F. Morphine sulfate 6mg IV, q 4 hours, prn has
been ordered. During your assessment after lunch, your findings are: BP 92/60, pulse 66, respirations
10, and temperature 98.8. Mrs. Gately is crying and tells you she is still experiencing severe pain. Your
action should be to
A. give her the next ordered dose of MS.
B. give her a back rub, put on some light music, and dim the lights in the room.
C. report your findings to the MD, requesting an alternate medication order
D. be obtained from the physician.
E. call her daughter to come and sit with her.
8. When counseling a patient who is starting to take MAO (monoamine oxidase) inhibitors such as
Nardil for depression, it is essential that they be warned not to eat foods containing tyramine, such as:
A. Roquefort, cheddar, or Camembert cheese.
B. grape juice, orange juice, or raisins.
C. onions, garlic, or scallions.
D. ground beef, turkey, or pork.
9. The physician orders an intramuscular injection of Demerol for the postoperativepatients pain.
When preparing to draw up the medication, the nurse is careful to remove the correct vial from the
narcotics cabinet. It is labeled
A. simethicone.
B. albuterol.
C. meperidine.
D. ibuprofen.

177

10. The nurse is administering an antibiotic to her pediatric patient. She checks the patients armband
and verifies the correct medication by checking the physicians order, medication kardex, and vial.
Which of the following is not considered one of the five rights of drug administration?
A. Right dose
B. Right route
C. Right frequency
D. Right time
11. A nurse is preparing the clients morning NPH insulin dose and notices a clumpy precipitate inside
the insulin vial. The nurse should:
A. draw up and administer the dose
B. shake the vial in an attempt to disperse the clumps
C. draw the dose from a new vial
D. warm the bottle under running water to dissolve the clump
12. A client with histoplasmosis has an order for ketoconazole (Nizoral). The nurse teaches the client to
do which of the following while taking this medication?
A. take the medication on an empty stomach
B. take the medication with an antacid
C. avoid exposure to sunlight
D. limit alcohol to 2 ounces per day
13. A nurse has taught a client taking a xanthine bronchodilator about beverages to avoid. The nurse
determines that the client understands the information if the client chooses which of the following
beverages from the dietary menu?
A. chocolate milk
B. cranberry juice
C. coffee
D. cola
14. A client is taking famotidine (Pepcid) asks the home care nurse what would be the best medication
to take for a headache. The nurse tells the client that it would be best to take:
A. aspirin (acetylsalicylic acid, ASA)
B. ibuprofen (Motrin)
C. acetaminophen (Tylenol)
D. naproxen (Naprosyn)
15. A nurse is planning dietary counseling for the client taking triamterene (Dyrenium). The nurse
plans to include which of the following in a list of foods that are acceptable?
A. baked potato
B. bananas
C. oranges
D. pears canned in water
16. A client with advanced cirrhosis of the liver is not tolerating protein well, as eveidenced by
abnormal laboratory values. The nurse anticipates that which of the following medications will be
prescribed for the client?
A. lactulose (Chronulac)
B. ethacrynic acid (Edecrin)
C. folic acid (Folvite)
D. thiamine (Vitamin B1)
17. A female client tells the clinic nurse that her skin is very dry and irritated. Which product would the
nurse suggest that the client apply to the dry skin?

178

A. glycerin emollient
B. aspercreame
C. myoflex
D. acetic acid solution
18. A nurse is providing instructions to a client regarding quinapril hydrochloride (Accupril). The nurse
tells the client:
A. to take the medication with food only
B. to rise slowly from a lying to a sitting position
C. to discontinue the medication if nausea occurs
D. that a therapeutic effect will be noted immediately
19. Auranofin (Ridaura) is prescribed for a client with rheumatoid arthritis, and the nurse monitors the
client for signs of an adverse effect related to the medication. Which of the following indicates an
adverse effect?
A. nausea
B. diarrhea
C. anorexia
D. proteinuria
20. A client has been taking benzonatate (Tessalon) as ordered. The nurse tells the client that this
medication should do which of the following?
A. take away nausea and vomiting
B. calm the persistent cough
C. decrease anxiety level
D. increase comfort level

Answers & Rationale


Here are the answers and rationale for this exam. Counter check your answers to those below and tell
us your scores. If you have any disputes or need more clarification to a certain question, please direct
them to the comments section.
1.

C. The ointment is placed in the lower conjunctival sac so it will not scratch the eye itself and
will get well distributed.

2.

D. The physician, nurse, and pharmacist all are licensed professionals and share responsibility
for errors.

3.

B. The decreased circulation to the kidney and reduced liver function tend to allow drugs to
accumulate and have toxic effects.

4.

C. The correct way to identify a patient before giving a medication is to check the name on the
medication administration record with the patients identification band. The nurse should also ask
the patient to state their name. The name on the door or the census list are not sufficient proof of
identification. Calling the patient by name is not as effective as having the patient state their name;
patients may not hear well or understand what the nurse is saying, and may respond to a name
which is not their own.

5.

C. Frequently patients do not complete an entire course of antibiotic therapy, and the bacteria
are not destroyed.

6.

B. Elavil is an antidepressant that lowers the seizure threshold, so would not be appropriate for
this patient. The other medications are anti-seizure drugs.

7.

C. Morphine sulfate depresses the respiratory center. When the rate is less than 10, the MD
should be notified.

179

8.

A. Monoamine oxidase inhibitors react with foods high in the amino acid tyramine to cause
dangerously high blood pressure. Aged cheeses are all high in this amino acid; the other foods are
not.

9.
10.

C. The generic name for Demerol is meperidine.


C. The five rights of medication administration are right drug, right dose, right route, right
time, right patient. Frequency is not included.

11.

C. The nurse should always inspect the vial of insulin before use for solution changes that may
signify loss of potency. NPH insulin is normally uniformly cloudy. Clumping, frosting, and precipitates
are signs of insulin damage. In this situation, because potency is questionable, it is safer to discard
the vial and draw up the dose from a new vial.

12.

C. The client should be taught that ketoconazole is an antifungal medication. It should be


taken with food or milk. Antacids should be avoided for 2 hours after it is taken because gastric acid
is needed to activate the medication. The client should avoid concurrent use of alcohol, because the
medication is hepatotoxic. The client should also avoid exposure to sunlight, because the
medication increases photosensitivity.

13.

B. Cola, coffee, and chocolate contain xanthine and should be avoided by the client taking a
xanthine bronchodilator. This could lead to an increased incidence of cardiovascular and central
nervous system side effects that can occur with the use of these types of bronchodilators.

14.

C. The client is taking famotidine, a histamine receptor antagonist. This implies that the client
has a disorder characterized by gastrointestinal (GI) irritation. The only medication of the ones listed
in the options that is not irritating to the GI tract is acetaminophen. The other medications could
aggravate an already existing GI problem.

15.

D. Triamterene is a potassium-sparing diuretic, and clients taking this medication should be


cautioned against eating foods that are high in potassium, including many vegetables, fruits, and
fresh meats. Because potassium is very water-soluble, foods that are prepared in water are often
lower in potassium.

16.

A. The client with cirrhosis has impaired ability to metabolize protein because of liver
dysfunction. Administration of lactulose aids in the clearance of ammonia via the gastrointestinal
(GI) tract. Ethacrynic acid is a diuretic. Folic acid and thiamine are vitamins, which may be used in
clients with liver disease as supplemental therapy.

17.

A. Glycerin is an emollient that is used for dry, cracked, and irritated skin. Aspercreme and
Myoflex are used to treat muscular aches. Acetic acid solution is used for irrigating, cleansing, and
packing wounds infected by Pseudomonas aeruginosa.

18.

B. Accupril is an angiotensin-converting enzyme (ACE) inhibitor. It is used in the treatment of


hypertension. The client should be instructed to rise slowly from a lying to sitting position and to
permit the legs to dangle from the bed momentarily before standing to reduce the hypotensive
effect. The medication does not need to be taken with meals. It may be given without regard to
food. If nausea occurs, the client should be instructed to take a non cola carbonated beverage and
salted crackers or dry toast. A full therapeutic effect may be noted in 1 to 2 weeks.

19.

D. Auranofin (Ridaura) is a gold preparation that is used as an antirheumatic. Gold toxicity is


an adverse effect and is evidenced by decreased hemoglobin, leukopenia, reduced granulocyte
counts, proteinuria, hematuria, stomatitis, glomerulonephritis, nephrotic syndrome, or cholestatic
jaundice. Anorexia, nausea, and diarrhea are frequent side effects of the medication.

20.

B. Benzonatate is a locally acting antitussive. Its effectiveness is measured by the degree to


which it decreases the intensity and frequency of cough, without eliminating the cough reflex.

180

12 Effective Test-Taking Skills & Strategies for


Nurses
Before stepping into the real world of nursing, one must pass their licensure
examinations first! Reviewing board exam questions are just a way of further
enhancing your knowledge and confidence for one of the most important test
in your life. What you must remember is that you are already knowledgeable
about cores of nursing, but this knowledge would be futile if you cannot
practice effective test-taking skills during your exams.
Researches suggest that an examiner who fully understands test
construction and familiar with appropriate test-taking strategies score higher
than those with similar level of knowledge but with inadequate test-taking
skills.
Multiple choice tests are different from tests such as essays, identifications
and true/false. In multiple choice exams, the question is called the stem.
The stem is followed by four alternative answers. One answer is correct, and
the other three are called distracterbecause they distract your attention
from identifying the correct answer. Note also that these distracter are not
necessarily incorrect answers but rather they may not be as correct as as the
one you are required to choose.
Here we have 12 effective test-taking skills and strategies that can help you
during your examinations:
1. Do not read extra meaning into the question

Tests questions are made to be direct and to the point. The question asks for
one particular response and you should not read or add other information into the
question.
Often you will find questions that require common sense answers and that
reading into these questions may give you an other interpretation. You should not
search for subtle meaning about the questions or answers.
Ask yourself What is the question asking?. Look for keywords and phrases
to help you understand. Interpret the question correctly first before reading into
the choices.

2. Understand exactly what the stem is asking before considering


the distracter

181

Make sure you read the stem correctly. Notice particularly the way the
question is phrase. Is it asking for the best response or the initial response?
Understand what the question is asking.

3. Rephrase the question in your own words

This technique requires you to interpret or translate the question into your
own words so that it is very clear in your own mind. Rephrasing the stem of the
question can assist your read the question correctly and in turn choose the
appropriate response.
Placing the question into your own words would help you in removing
extraneous data and get into the core of the stem.

4. When analyzing the distracter, isolate what is important in the


answer alternatives from what is not important relative to the
question.

In a good test construction, all of the distracter should be feasible and


reasonable and should apply directly to the stem. There should be a commonality
in all of the distracter.
Also, all of the distracter may be correct but not the right chose for the
specific question that is being asked. The technique here is to ask yourself
whether each possible alternative is true or false in relation to the stem.

5. After choosing the correct answer alternative and separating it


form the distracter, go back to the stem and make sure your choice
does, in fact, answer the question.

Many test-takers fail to recheck the answer with the stem, and they answer
the question incorrectly.
An effective strategy is to judge all four alternative choices against the stem
and not against one another. Read the stem, then check alternative 1 against the
stem, then check alternative 2 against the stem and so on. This process will
eliminate choosing an alternative that does not fit with the question.

6. When a question contains multiple variables as alternative


choices, use the elimination of variable technique.

Each question may pose different alternatives with several variables. Use the
process of elimination. Study the question first and ask yourself what variable fits
with this condition, or after examining the distracter underline the symptom that
you know is correct. Now ask yourself what variable is not present with this
condition. Again examine the distracter and cross out those variables that are
incorrect. By this process your probably eliminated at least two distracter even
without taking the time to consider the other two.

182

7. When answering a difficult question, utilize your body of


knowledge.

When you come across a difficult question and you cannot immediately
identify the answer, go back to your body of knowledge and draw all the
information that you do know about the condition.
If you are unfamiliar with the disease or disorder and cannot choose the right
nursing action, try to generalize to other situations. For example, if the question
asks about dog bites, and youve never learned the course of the disorder, go
back to an area of k nowledge that you do know, for example, circulation and
body response to toxic substances.
Even though you do not know exactly what to d, you might know what not to
do. Eliminate distracter to increase your chances of arriving at the correct answer.

8. The ability to guess correctly is both a skill and an art.

The board exams is not a right minus wrong type. It is important for you to
answer every question even if you have to guess. Guessing gives you only a 25%
chance of getting the correct answer.
Try to eliminate at least one (or more) distracter as this will increase the
percentage margin of chance for guessing correctly.
Examine the distracter and if one is the exact opposite of another (e.g.
complete bed rest is different from activity as tolerated; both cannot be correct
since they are of opposites), choose the one that seems to be most logical.
Try to identify the underlying principle that supports the question. If you can
answer the question, you might then be able to guess the correct answer. This
strategy is especially true with a psychosocial question.
Look at the way the alternatives are presented. Are there two answers that
are very close? often when this occurs, the ability to discriminate will show
evidence of judgement. Check to see if one, more than the other, is the best
choice for the question.
Are there distracter that are presented not logical (which are correct in
themselves but do not have anything to do with the question)? Eliminate these
and focus on other alternatives.
Use your intuition. If you cannot choose an alternative from a logical point,
allow yourself to feel which one might be right. Often your subconscious mind will
choose correctly (based on all the conscious knowledge you have of course) so
simply let yourself feel which alternative might be right. Remember, its better to
choose one answer than none at all.

9. Choosing answer from a hunch

There comes a time when you are faced with a certain question and you have
a hunch that this particular choice is correct. Do we depend on this hunch?.
Current studies supports that hunches are often correct, for they are based on
rapid subconscious connections in the brain.
183

Your stored knowledge, recall, and experience can combine to assist you in
arriving at the correct answer. So, if you have an initial hunch, go with it! Do not
chance the answer if and only if, upon reflection, it just doesnt seem right. On the
other hand, if later in the test you find relevant information or make new
connection of information and you feel that your answer was incorrect, do go back
and change it.

10. Choosing the best answer from a strategy point-of-view.

Frequently, the most comprehensive answer is the best choice (Longest the
best!). For example, if two alternatives seems reasonable but one answer includes
the other (i.e., it is more detailed, more comprehensive), then this answer would
be the best choice. If an answer focuses on medical knowledge, be wary, for
this alternative might be just a good distractor. Remember, this is a nursing test
and questions are designed to test your nursing competency and safety.
It is unlikely that a question would require a medical action for the correct
answer; it may, however offer these actions as distracter.
Beware of answers that contain specific qualifiers, such as always and
never. they rarely fit within a logical framework.

11. Time Management is important

Time allotment for each question in the NLE is 2 hours/100 questions is equal
to 1.2 minutes. It is important to use your time wisely and program yourself not to
spend too much time on one question. Since your are not penalized for incorrect
answers, it is important to finish each portion of the test.
Prepare yourself to practice time management by timing yourself for each
question during your sample exams.

12. Let the Divine Intervention Intervene

If you cannot answer the question. Have faith in the Lord and thus youll be
confident to answer each question. If you do not know, He knows. Ask for Gods
help! Remember, you are doing this for His glory!

Reference:
Smith S. (1989). Review questions for NCLEX-RN 5th Edition.

Introduction
Another set of the ultimate reviewer for Fundamentals of Nursing. The Nursing Bullets are compilation
of bite-sized information that are extremely useful during your review for the NCLEX or the board
exams.

Topics
184

Topics included are:

Nursing Assessment

Diets

Various topics about Fundamentals of Nursing

Bullets
1. After turning a patient, the nurse should document the position used, the time that the patient was
turned, and the findings of skin assessment.
2. PERRLA is an abbreviation for normal pupil assessment findings: pupils equal, round, and reactive to
light with accommodation.
3. When percussing a patients chest for postural drainage, the nurses hands should be cupped.
4. When measuring a patients pulse, the nurse should assess its rate, rhythm, quality, and strength.
5. Before transferring a patient from a bed to a wheelchair, the nurse should push the wheelchair
footrests to the sides and lock its wheels.
6. When assessing respirations, the nurse should document their rate, rhythm, depth, and quality.
7. For a subcutaneous injection, the nurse should use a 5/8 25G needle.
8. The notation AA & O 3 indicates that the patient is awake, alert, and oriented to person (knows
who he is), place (knows where he is), and time (knows the date and time).
9. Fluid intake includes all fluids taken by mouth, including foods that are liquid at room temperature,
such as gelatin, custard, and ice cream; I.V. fluids; and fluids administered in feeding tubes. Fluid
output includes urine, vomitus, and drainage (such as from a nasogastric tube or from a wound) as
well as blood loss, diarrhea or feces, and perspiration.
10. After administering an intradermal injection, the nurse shouldnt massage the area because
massage can irritate the site and interfere with results.
11. When administering an intradermal injection, the nurse should hold the syringe almost flat against
the patients skin (at about a 15-degree angle), with the bevel up.
12. To obtain an accurate blood pressure, the nurse should inflate the manometer to 20 to 30 mm Hg
above the disappearance of the radial pulse before releasing the cuff pressure.
13. The nurse should count an irregular pulse for 1 full minute.
14. A patient who is vomiting while lying down should be placed in a lateral position to prevent
aspiration of vomitus.
15. Prophylaxis is disease prevention.
16. Body alignment is achieved when body parts are in proper relation to their natural position.
17. Trust is the foundation of a nurse-patient relationship.
18. Blood pressure is the force exerted by the circulating volume of blood on the arterial walls.
19. Malpractice is a professionals wrongful conduct, improper discharge of duties, or failure to meet
standards of care that causes harm to another.
20. As a general rule, nurses cant refuse a patient care assignment; however, in most states, they
may refuse to participate in abortions.
21. A nurse can be found negligent if a patient is injured because the nurse failed to perform a duty
that a reasonable and prudent person would perform or because the nurse performed an act that a
reasonable and prudent person wouldnt perform.
22. States have enacted Good Samaritan laws to encourage professionals to provide medical
assistance at the scene of an accident without fear of a lawsuit arising from the assistance. These laws
dont apply to care provided in a health care facility.
23. A physician should sign verbal and telephone orders within the time established by facility policy,
usually 24 hours.

185

24. A competent adult has the right to refuse lifesaving medical treatment; however, the individual
should be fully informed of the consequences of his refusal.
25. Although a patients health record, or chart, is the health care facilitys physical property, its
contents belong to the patient.
26. Before a patients health record can be released to a third party, the patient or the patients legal
guardian must give written consent.
27. Under the Controlled Substances Act, every dose of a controlled drug thats dispensed by the
pharmacy must be accounted for, whether the dose was administered to a patient or discarded
accidentally.
28. A nurse cant perform duties that violate a rule or regulation established by a state licensing board,
even if they are authorized by a health care facility or physician.
29. To minimize interruptions during a patient interview, the nurse should select a private room,
preferably one with a door that can be closed.
30. In categorizing nursing diagnoses, the nurse addresses life-threatening problems first, followed by
potentially life-threatening concerns.
31. The major components of a nursing care plan are outcome criteria (patient goals) and nursing
interventions.
32. Standing orders, or protocols, establish guidelines for treating a specific disease or set of
symptoms.
33. In assessing a patients heart, the nurse normally finds the point of maximal impulse at the fifth
intercostal space, near the apex.
34. The S1 heard on auscultation is caused by closure of the mitral and tricuspid valves.
35. To maintain package sterility, the nurse should open a wrappers top flap away from the body,
open each side flap by touching only the outer part of the wrapper, and open the final flap by grasping
the turned-down corner and pulling it toward the body.
36. The nurse shouldnt dry a patients ear canal or remove wax with a cotton-tipped applicator
because it may force cerumen against the tympanic membrane.
37. A patients identification bracelet should remain in place until the patient has been discharged
from the health care facility and has left the premises.
38. The Controlled Substances Act designated five categories, or schedules, that classify controlled
drugs according to their abuse potential.
39. Schedule I drugs, such as heroin, have a high abuse potential and have no currently accepted
medical use in the United States.
40. Schedule II drugs, such as morphine, opium, and meperidine (Demerol), have a high abuse
potential, but currently have accepted medical uses. Their use may lead to physical or psychological
dependence.
41. Schedule III drugs, such as paregoric and butabarbital (Butisol), have a lower abuse potential than
Schedule I or II drugs. Abuse of Schedule III drugs may lead to moderate or low physical or
psychological dependence, or both.
42. Schedule IV drugs, such as chloral hydrate, have a low abuse potential compared with Schedule III
drugs.
43. Schedule V drugs, such as cough syrups that contain codeine, have the lowest abuse potential of
the controlled substances.
44. Activities of daily living are actions that the patient must perform every day to provide self-care
and to interact with society.
45. Testing of the six cardinal fields of gaze evaluates the function of all extraocular muscles and
cranial nerves III, IV, and VI.

186

46. The six types of heart murmurs are graded from 1 to 6. A grade 6 heart murmur can be heard with
the stethoscope slightly raised from the chest.
47. The most important goal to include in a care plan is the patients goal.
48. Fruits are high in fiber and low in protein, and should be omitted from a low-residue diet.
49. The nurse should use an objective scale to assess and quantify pain. Postoperative pain varies
greatly among individuals.
50. Postmortem care includes cleaning and preparing the deceased patient for family viewing,
arranging transportation to the morgue or funeral home, and determining the disposition of
belongings.
51. The nurse should provide honest answers to the patients questions.
52. Milk shouldnt be included in a clear liquid diet.
53. When caring for an infant, a child, or a confused patient, consistency in nursing personnel is
paramount.
54. The hypothalamus secretes vasopressin and oxytocin, which are stored in the pituitary gland.
55. The three membranes that enclose the brain and spinal cord are the dura mater, pia mater, and
arachnoid.
56. A nasogastric tube is used to remove fluid and gas from the small intestine preoperatively or
postoperatively.
57. Psychologists, physical therapists, and chiropractors arent authorized to write prescriptions for
drugs.
58. The area around a stoma is cleaned with mild soap and water.
59. Vegetables have a high fiber content.
60. The nurse should use a tuberculin syringe to administer a subcutaneous injection of less than 1 ml.
61. For adults, subcutaneous injections require a 25G 1 needle; for infants, children, elderly, or very
thin patients, they require a 25G to 27G needle.
62. Before administering a drug, the nurse should identify the patient by checking the identification
band and asking the patient to state his name.
63. To clean the skin before an injection, the nurse uses a sterile alcohol swab to wipe from the center
of the site outward in a circular motion.
64. The nurse should inject heparin deep into subcutaneous tissue at a 90-degree angle (perpendicular
to the skin) to prevent skin irritation.
65. If blood is aspirated into the syringe before an I.M. injection, the nurse should withdraw the needle,
prepare another syringe, and repeat the procedure.
66. The nurse shouldnt cut the patients hair without written consent from the patient or an
appropriate relative.
67. If bleeding occurs after an injection, the nurse should apply pressure until the bleeding stops. If
bruising occurs, the nurse should monitor the site for an enlarging hematoma.
68. When providing hair and scalp care, the nurse should begin combing at the end of the hair and
work toward the head.
69. The frequency of patient hair care depends on the length and texture of the hair, the duration of
hospitalization, and the patients condition.
70. Proper function of a hearing aid requires careful handling during insertion and removal, regular
cleaning of the ear piece to prevent wax buildup, and prompt replacement of dead batteries.
71. The hearing aid thats marked with a blue dot is for the left ear; the one with a red dot is for the
right ear.
72. A hearing aid shouldnt be exposed to heat or humidity and shouldnt be immersed in water.
73. The nurse should instruct the patient to avoid using hair spray while wearing a hearing aid.

187

74. The five branches of pharmacology are pharmacokinetics, pharmacodynamics,


pharmacotherapeutics, toxicology, and pharmacognosy.
75. The nurse should remove heel protectors every 8 hours to inspect the foot for signs of skin
breakdown.
76. Heat is applied to promote vasodilation, which reduces pain caused by inflammation.
77. A sutured surgical incision is an example of healing by first intention (healing directly, without
granulation).
78. Healing by secondary intention (healing by granulation) is closure of the wound when granulation
tissue fills the defect and allows reepithelialization to occur, beginning at the wound edges and
continuing to the center, until the entire wound is covered.
79. Keloid formation is an abnormality in healing thats characterized by overgrowth of scar tissue at
the wound site.
80. The nurse should administer procaine penicillin by deep I.M. injection in the upper outer portion of
the buttocks in the adult or in the midlateral thigh in the child. The nurse shouldnt massage the
injection site.
81. An ascending colostomy drains fluid feces. A descending colostomy drains solid fecal matter.
82. A folded towel (scrotal bridge) can provide scrotal support for the patient with scrotal edema
caused by vasectomy, epididymitis, or orchitis.
83. When giving an injection to a patient who has a bleeding disorder, the nurse should use a smallgauge needle and apply pressure to the site for 5 minutes after the injection.
84. Platelets are the smallest and most fragile formed element of the blood and are essential for
coagulation.
85. To insert a nasogastric tube, the nurse instructs the patient to tilt the head back slightly and then
inserts the tube. When the nurse feels the tube curving at the pharynx, the nurse should tell the
patient to tilt the head forward to close the trachea and open the esophagus by swallowing. (Sips of
water can facilitate this action.)
86. Families with loved ones in intensive care units report that their four most important needs are to
have their questions answered honestly, to be assured that the best possible care is being provided, to
know the patients prognosis, and to feel that there is hope of recovery.
87. Double-bind communication occurs when the verbal message contradicts the nonverbal message
and the receiver is unsure of which message to respond to.
88. A nonjudgmental attitude displayed by a nurse shows that she neither approves nor disapproves of
the patient.
89. Target symptoms are those that the patient finds most distressing.
90. A patient should be advised to take aspirin on an empty stomach, with a full glass of water, and
should avoid acidic foods such as coffee, citrus fruits, and cola.
91. For every patient problem, there is a nursing diagnosis; for every nursing diagnosis, there is a goal;
and for every goal, there are interventions designed to make the goal a reality. The keys to answering
examination questions correctly are identifying the problem presented, formulating a goal for the
problem, and selecting the intervention from the choices provided that will enable the patient to reach
that goal.
92. Fidelity means loyalty and can be shown as a commitment to the profession of nursing and to the
patient.
93. Administering an I.M. injection against the patients will and without legal authority is battery.
94. An example of a third-party payer is an insurance company.
95. The formula for calculating the drops per minute for an I.V. infusion is as follows: (volume to be
infused drip factor) time in minutes = drops/minute

188

96. On-call medication should be given within 5 minutes of the call.


97. Usually, the best method to determine a patients cultural or spiritual needs is to ask him.
98. An incident report or unusual occurrence report isnt part of a patients record, but is an in-house
document thats used for the purpose of correcting the problem.
99. Critical pathways are a multidisciplinary guideline for patient care.
100. When prioritizing nursing diagnoses, the following hierarchy should be used: Problems associated
with the airway, those concerning breathing, and those related to circulation.
101. The two nursing diagnoses that have the highest priority that the nurse can assign are Ineffective
airway clearance and Ineffective breathing pattern.
102. A subjective sign that a sitz bath has been effective is the patients expression of decreased pain
or discomfort.
103. For the nursing diagnosis Deficient diversional activity to be valid, the patient must state that hes
bored, that he has nothing to do, or words to that effect.
104. The most appropriate nursing diagnosis for an individual who doesnt speak English is Impaired
verbal communication related to inability to speak dominant language (English).
105. The family of a patient who has been diagnosed as hearing impaired should be instructed to face
the individual when they speak to him.
106. Before instilling medication into the ear of a patient who is up to age 3, the nurse should pull the
pinna down and back to straighten the eustachian tube.
107. To prevent injury to the cornea when administering eyedrops, the nurse should waste the first
drop and instill the drug in the lower conjunctival sac.
108. After administering eye ointment, the nurse should twist the medication tube to detach the
ointment.
109. When the nurse removes gloves and a mask, she should remove the gloves first. They are soiled
and are likely to contain pathogens.
110. Crutches should be placed 6 (15.2 cm) in front of the patient and 6 to the side to form a tripod
arrangement.
111. Listening is the most effective communication technique.
112. Before teaching any procedure to a patient, the nurse must assess the patients current
knowledge and willingness to learn.
113. Process recording is a method of evaluating ones communication effectiveness.
114. When feeding an elderly patient, the nurse should limit high-carbohydrate foods because of the
risk of glucose intolerance.
115. When feeding an elderly patient, essential foods should be given first.
116. Passive range of motion maintains joint mobility. Resistive exercises increase muscle mass.
117. Isometric exercises are performed on an extremity thats in a cast.
118. A back rub is an example of the gate-control theory of pain.
119. Anything thats located below the waist is considered unsterile; a sterile field becomes unsterile
when it comes in contact with any unsterile item; a sterile field must be monitored continuously; and a
border of 1 (2.5 cm) around a sterile field is considered unsterile.
120. A shift to the left is evident when the number of immature cells (bands) in the blood increases
to fight an infection.
121. A shift to the right is evident when the number of mature cells in the blood increases, as seen in
advanced liver disease and pernicious anemia.
122. Before administering preoperative medication, the nurse should ensure that an informed consent
form has been signed and attached to the patients record.

189

123. A nurse should spend no more than 30 minutes per 8-hour shift providing care to a patient who
has a radiation implant.
124. A nurse shouldnt be assigned to care for more than one patient who has a radiation implant.
125. Long-handled forceps and a lead-lined container should be available in the room of a patient who
has a radiation implant.
126. Usually, patients who have the same infection and are in strict isolation can share a room.
127. Diseases that require strict isolation include chickenpox, diphtheria, and viral hemorrhagic fevers
such as Marburg disease.
128. For the patient who abides by Jewish custom, milk and meat shouldnt be served at the same
meal.
129. Whether the patient can perform a procedure (psychomotor domain of learning) is a better
indicator of the effectiveness of patient teaching than whether the patient can simply state the steps
involved in the procedure (cognitive domain of learning).
130. According to Erik Erikson, developmental stages are trust versus mistrust (birth to 18 months),
autonomy versus shame and doubt (18 months to age 3), initiative versus guilt (ages 3 to 5), industry
versus inferiority (ages 5 to 12), identity versus identity diffusion (ages 12 to 18), intimacy versus
isolation (ages 18 to 25), generativity versus stagnation (ages 25 to 60), and ego integrity versus
despair (older than age 60).
131. When communicating with a hearing impaired patient, the nurse should face him.
132. An appropriate nursing intervention for the spouse of a patient who has a serious incapacitating
disease is to help him to mobilize a support system.
133. Hyperpyrexia is extreme elevation in temperature above 106 F (41.1 C).
134. Milk is high in sodium and low in iron.
135. When a patient expresses concern about a health-related issue, before addressing the concern,
the nurse should assess the patients level of knowledge.
136. The most effective way to reduce a fever is to administer an antipyretic, which lowers the
temperature set point.
137. When a patient is ill, its essential for the members of his family to maintain communication about
his health needs.
138. Ethnocentrism is the universal belief that ones way of life is superior to others.
139. When a nurse is communicating with a patient through an interpreter, the nurse should speak to
the patient and the interpreter.
140. In accordance with the hot-cold system used by some Mexicans, Puerto Ricans, and other
Hispanic and Latino groups, most foods, beverages, herbs, and drugs are described as cold.
141. Prejudice is a hostile attitude toward individuals of a particular group.
142. Discrimination is preferential treatment of individuals of a particular group. Its usually discussed
in a negative sense.
143. Increased gastric motility interferes with the absorption of oral drugs.
144. The three phases of the therapeutic relationship are orientation, working, and termination.
145. Patients often exhibit resistive and challenging behaviors in the orientation phase of the
therapeutic relationship.
146. Abdominal assessment is performed in the following order: inspection, auscultation, percussion &
palpation.
147. When measuring blood pressure in a neonate, the nurse should select a cuff thats no less than
one-half and no more than two-thirds the length of the extremity thats used.
148. When administering a drug by Z-track, the nurse shouldnt use the same needle that was used to
draw the drug into the syringe because doing so could stain the skin.

190

149. Sites for intradermal injection include the inner arm, the upper chest, and on the back, under the
scapula.
150. When evaluating whether an answer on an examination is correct, the nurse should consider
whether the action thats described promotes autonomy (independence), safety, self-esteem, and a
sense of belonging.
151. When answering a question on the NCLEX examination, the student should consider the cue (the
stimulus for a thought) and the inference (the thought) to determine whether the inference is correct.
When in doubt, the nurse should select an answer that indicates the need for further information to
eliminate ambiguity. For example, the patient complains of chest pain (the stimulus for the thought)
and the nurse infers that the patient is having cardiac pain (the thought). In this case, the nurse hasnt
confirmed whether the pain is cardiac. It would be more appropriate to make further assessments.
152. Veracity is truth and is an essential component of a therapeutic relationship between a health
care provider and his patient.
153. Beneficence is the duty to do no harm and the duty to do good. Theres an obligation in patient
care to do no harm and an equal obligation to assist the patient.
154. Nonmaleficence is the duty to do no harm.
155. Fryes ABCDE cascade provides a framework for prioritizing care by identifying the most important
treatment concerns.
156. A = Airway. This category includes everything that affects a patent airway, including a foreign
object, fluid from an upper respiratory infection, and edema from trauma or an allergic reaction.
157. B = Breathing. This category includes everything that affects the breathing pattern, including
hyperventilation or hypoventilation and abnormal breathing patterns, such as Korsakoffs, Biots, or
Cheyne-Stokes respiration.
158. C = Circulation. This category includes everything that affects the circulation, including fluid and
electrolyte disturbances and disease processes that affect cardiac output.
159. D = Disease processes. If the patient has no problem with the airway, breathing, or circulation,
then the nurse should evaluate the disease processes, giving priority to the disease process that poses
the greatest immediate risk. For example, if a patient has terminal cancer and hypoglycemia,
hypoglycemia is a more immediate concern.
160. E = Everything else. This category includes such issues as writing an incident report and
completing the patient chart. When evaluating needs, this category is never the highest priority.
161. When answering a question on an NCLEX examination, the basic rule is assess before action.
The student should evaluate each possible answer carefully. Usually, several answers reflect the
implementation phase of nursing and one or two reflect the assessment phase. In this case, the best
choice is an assessment response unless a specific course of action is clearly indicated.
162. Rule utilitarianism is known as the greatest good for the greatest number of people theory.
163. Egalitarian theory emphasizes that equal access to goods and services must be provided to the
less fortunate by an affluent society.
164. Active euthanasia is actively helping a person to die.
165. Brain death is irreversible cessation of all brain function.
166. Passive euthanasia is stopping the therapy thats sustaining life.
167. A third-party payer is an insurance company.
168. Utilization review is performed to determine whether the care provided to a patient was
appropriate and cost-effective.
169. A value cohort is a group of people who experienced an out-of-the-ordinary event that shaped
their values.
170. Voluntary euthanasia is actively helping a patient to die at the patients request.

191

171. Bananas, citrus fruits, and potatoes are good sources of potassium.
172. Good sources of magnesium include fish, nuts, and grains.
173. Beef, oysters, shrimp, scallops, spinach, beets, and greens are good sources of iron.
174. Intrathecal injection is administering a drug through the spine.
175. When a patient asks a question or makes a statement thats emotionally charged, the nurse
should respond to the emotion behind the statement or question rather than to whats being said or
asked.
176. The steps of the trajectory-nursing model are as follows:
177. Step 1: Identifying the trajectory phase
178. Step 2: Identifying the problems and establishing goals
179. Step 3: Establishing a plan to meet the goals
180. Step 4: Identifying factors that facilitate or hinder attainment of the goals
181. Step 5: Implementing interventions
182. Step 6: Evaluating the effectiveness of the interventions
183. A Hindu patient is likely to request a vegetarian diet.
184. Pain threshold, or pain sensation, is the initial point at which a patient feels pain.
185. The difference between acute pain and chronic pain is its duration.
186. Referred pain is pain thats felt at a site other than its origin.
187. Alleviating pain by performing a back massage is consistent with the gate control theory.
188. Rombergs test is a test for balance or gait.
189. Pain seems more intense at night because the patient isnt distracted by daily activities.
190. Older patients commonly dont report pain because of fear of treatment, lifestyle changes, or
dependency.
191. No pork or pork products are allowed in a Muslim diet.
192. Two goals of Healthy People 2010 are:
193. Help individuals of all ages to increase the quality of life and the number of years of optimal
health
194. Eliminate health disparities among different segments of the population.
195. A community nurse is serving as a patients advocate if she tells a malnourished patient to go to
a meal program at a local park.
196. If a patient isnt following his treatment plan, the nurse should first ask why.
197. Falls are the leading cause of injury in elderly people.
198. Primary prevention is true prevention. Examples are immunizations, weight control, and smoking
cessation.
199. Secondary prevention is early detection. Examples include purified protein derivative (PPD),
breast self-examination, testicular self-examination, and chest X-ray.
200. Tertiary prevention is treatment to prevent long-term complications.
201. A patient indicates that hes coming to terms with having a chronic disease when he says, Im
never going to get any better.
202. On noticing religious artifacts and literature on a patients night stand, a culturally aware nurse
would ask the patient the meaning of the items.
203. A Mexican patient may request the intervention of a curandero, or faith healer, who involves the
family in healing the patient.
204. In an infant, the normal hemoglobin value is 12 g/dl.
205. The nitrogen balance estimates the difference between the intake and use of protein.
206. Most of the absorption of water occurs in the large intestine.
207. Most nutrients are absorbed in the small intestine.

192

208. When assessing a patients eating habits, the nurse should ask, What have you eaten in the last
24 hours?
209. A vegan diet should include an abundant supply of fiber.
210. A hypotonic enema softens the feces, distends the colon, and stimulates peristalsis.
211. First-morning urine provides the best sample to measure glucose, ketone, pH, and specific gravity
values.
212. To induce sleep, the first step is to minimize environmental stimuli.
213. Before moving a patient, the nurse should assess the patients physical abilities and ability to
understand instructions as well as the amount of strength required to move the patient.
214. To lose 1 lb (0.5 kg) in 1 week, the patient must decrease his weekly intake by 3,500 calories
(approximately 500 calories daily). To lose 2 lb (1 kg) in 1 week, the patient must decrease his weekly
caloric intake by 7,000 calories (approximately 1,000 calories daily).
215. To avoid shearing force injury, a patient who is completely immobile is lifted on a sheet.
216. To insert a catheter from the nose through the trachea for suction, the nurse should ask the
patient to swallow.
217. Vitamin C is needed for collagen production.
218. Only the patient can describe his pain accurately.
219. Cutaneous stimulation creates the release of endorphins that block the transmission of pain
stimuli.
220. Patient-controlled analgesia is a safe method to relieve acute pain caused by surgical incision,
traumatic injury, labor and delivery, or cancer.
221. An Asian American or European American typically places distance between himself and others
when communicating.
222. The patient who believes in a scientific, or biomedical, approach to health is likely to expect a
drug, treatment, or surgery to cure illness.
223. Chronic illnesses occur in very young as well as middle-aged and very old people.
224. The trajectory framework for chronic illness states that preferences about daily life activities
affect treatment decisions.
225. Exacerbations of chronic disease usually cause the patient to seek treatment and may lead to
hospitalization.
226. School health programs provide cost-effective health care for low-income families and those who
have no health insurance.
227. Collegiality is the promotion of collaboration, development, and interdependence among
members of a profession.
228. A change agent is an individual who recognizes a need for change or is selected to make a
change within an established entity, such as a hospital.
229. The patients bill of rights was introduced by the American Hospital Association.
230. Abandonment is premature termination of treatment without the patients permission and without
appropriate relief of symptoms.
231. Values clarification is a process that individuals use to prioritize their personal values.
232. Distributive justice is a principle that promotes equal treatment for all.
233. Milk and milk products, poultry, grains, and fish are good sources of phosphate.
234. The best way to prevent falls at night in an oriented, but restless, elderly patient is to raise the
side rails.
235. By the end of the orientation phase, the patient should begin to trust the nurse.
236. Falls in the elderly are likely to be caused by poor vision.

193

237. Barriers to communication include language deficits, sensory deficits, cognitive impairments,
structural deficits, and paralysis.
238. The three elements that are necessary for a fire are heat, oxygen, and combustible material.
239. Sebaceous glands lubricate the skin.
240. To check for petechiae in a dark-skinned patient, the nurse should assess the oral mucosa.
241. To put on a sterile glove, the nurse should pick up the first glove at the folded border and adjust
the fingers when both gloves are on.
242. To increase patient comfort, the nurse should let the alcohol dry before giving an intramuscular
injection.
243. Treatment for a stage 1 ulcer on the heels includes heel protectors.
244. Seventh-Day Adventists are usually vegetarians.
245. Endorphins are morphine-like substances that produce a feeling of well-being.
246. Pain tolerance is the maximum amount and duration of pain that an individual is willing to endure.

194

You might also like